Download as pdf or txt
Download as pdf or txt
You are on page 1of 95

~!5n:~~u~!

:'n~~ PRE RMO


c.,.. cw. Programmes ('PCCP)
"'-"foundation CLASS VIII TO X
www.pccp.resonance.ac.ln

CONTENTS ·
TOPIC PAGE NO.

1. Number Theory 01-16

2. EquaOon 17-40

3. Sequence & Serles 41-60

4. Geometry 61 • .7 0

5. Combinatorics 71 • 88
PREFACE ·
An Important Note to Parents and Students ·

Oea·r.Sludent,
If you are reading this, it means you are serious about performing in your PRMO Exam. To help you
achieve your potential, this booklet is designed in a way which is highly beneficial for students. Let's
see how to use the different components of the booklet :
• Theory Part : The theory part has been designed with perfect bl1::nd of solved examples, text and
impo~ant notes. At relevant checkpoints, in theory, exercises have. bee~ inserted to enhance the
reading experience of a student.

• Exercise 1 : This exercise is Competitive Level Exercise (CLE). After going through the theory and
thus·absorbing the important concepts, students are ready to implement their !earning in competitive
level questions. These questions are in accordance to the level being asked in the National Level
Competitive Examinations especially PRMO. These questions are must for all the students to
,stren_gthen _their concepts.

• Exercise 2 : This exercise consists of questions from previous year papers of PRMO and RMO. This
.. e?(ercise is included for students to determine their current proficien~y l~vel of that chapter with respect
to the competitive examination.

After reading the theory and completing the exercises, a student should be ab:~ to have a conceptual
framework and problem solving aptitude in that particular chapter. For best results, all exercises
should be solved In a fair notebook and all the solutions should be maintained so that when
time of revision comes, this notebook proves to be a handy one.

''You have to dream before your dreams come true"


Pre-foundation Career Care Programmes (PCCP) Division

Every effort has been taken to make our study material error free, however any suggestion to
improve is welcome in this regard. ·
Number Theory / - - - - - - - - - - - - - - - - - - - - - - - - - ~ •

I Number Theory I
DIVISIBILITY :

An integer a -¢ 0 divides b, if there exists an integer x such that b = ax, and thus, we write as a I b {read a
· divides b). This can also be stated as bis divisible by a or a is a divisor of b orb is a multiple of a. If a does
not divide b we write as a I b.

Properties of Divisibility
1. a I band b I c a Ic
2. a I b, a I c a I {b + c), and a I {b. c)
3. a I b, a I {b + c) a Ic
4. a I b, a I (b • c) aIc
5. a I b and a I c a I {kb± le) for all k, I e z
6. a I b and b I a a =±b
7. a Ib b = 0 or I a Is I b J. In particular if a I b where a> 0, b > 0, then a< b
8. a Ib a I be for any integer c
9. a I b iff ma I mb where m-¢ 0

Notes:
1. (x + y) I (x2n+1 + y2n+1) \/ n E No
Proof:
For n = 0 it is obvious, for n 2: 1, we have
(x2n+1 + y2n•1) = (x + y)(x2n _ x2n·1y + x2n-2 y2 - ... + y2")

2. {x - y) I (x" - y") ':/ n e N


Proof:
For n = 1 it is obvious, for n 2: 2, we have
x" - y" = (x - y)(x"-1 + xn-2 y + x"-3 y2 + .. .+ y"-1)

GCD and LCM:


The greatest common divisor of two positive integers a and b is the great- est positive integer that divides
both a and b, which we denote by gcd(a, b), and similarly, the lowest common multiple of a and b is the
least positive say that a and b are relatively prime if gcd(a, b) = 1. For integers a,. a2,
................ , an, gcd{a1, a2, ... , an) is the greatest positive integer that divides all of a,, a,, . .. , an,
and lcm{a1, a2, ••• , an) is defined similarly.
Useful Facts

For all a, b, gcd{a, b) · lcm{a, b) = ab.

• For all a, b, and m, gcd(ma, mb) =m gcd(a, b) and lcm(ma, mb) =mlcm(a, b).
a b) gcd{a b)
• If d I gcd(a, b), then gcd( d'~ = d '

In particular, if d =gcd{a, b), then gcd{a/d, b/d) =1; that is, aid and bid are relatively prime.
• If a I be and gcd{a, c) = 1, then a I b.
• For positive integers a and b, if d is a positive integer such that d I a,
d I b, and for any d', d' I a and d' Ib implies that d' Jd, then d = gee {a,b). This is meraly the
assertion that any common divisor of a and b divides gcd {a, b)

g R ill C°'pcrate Office: CG Tower, A--46 & 52, IPIA, Near City Mall, Jhalawar Road, Kola (Raj.}-324005
r. Edu~u~~C'n~D,';;,~ Websile : www.resonance.ac.ln E-mail : contact resonance.ac.ln
Toll Free : 1800 258 5555 CIN: U80302RJ2007PLC024029
Number Theory / , - - - - - - - - - - - - - - - - - - - - - - - - - - ~ •
If a, a2 · · ·a0 is a perfect k power and the a1 are pairwise relatively prime, then each a1 l:i a
1h

th
perfect k power.
• Any two consecutive integers are relatively prime.

EUCLID DIVISION LEMMA (EDL) :

For any positive integer a and integer b, there exist unique integers q and r such that
b = qa + r and O r < a, with r = 0 iff a I b.

EUCLID DIVISION ALGORITHM (EDA) :

If a, b, q, rare integers such that a = be, + r then common factor of a and b is also common factor of
band r

Useful Facts :

(i) For any positive integers a and b there exist integers x and y such that ax + by= gcd(a,
b).Furthermore, as x and y vary over all integers, ax + by attains all ultiples and only multiples of
gcd(a,b).

Proof. Let S be the set of all integers of the form ax+by, and let d be the least positive element of S. By the
division algorithm, there exist integers q and r such that a = qd + r, 0 r < d. Then

r =a - qd = a - q(ax + by) =(1 - qx)a - (qy)b, so r is also in S. But r < d, so r = 0 ==> d I a.

and similarly, d I b, sod I gcd(a, b). However, gcd(a, b) divides all elements of S, so in particular

gcd(a,b) Id==> d = gcd(a,b).

(ii). The positive integers a and b are relatively prime iff there exist integers x and y such that ax+by = 1.

(iii) For any positive integers a,, a2, . . . , an, there exist integers x1, x2, ••• , Xn, such that

a, X + a2X2 +· · •+an Xn = gcd(a, , 82, . .. , an),

(iv). Let a and b be positive integers, and let n be an integer. Then the equation ax + by == n has a
solution in integers x and y iff gcd(a,b) I n. If this is the case, then all solutions are of the form

( x, y) =( x0 + t.~, Yo - t.~) where d = gcd(a, b), (x0 , y 0 ) is a specific solution of ax + by= n, anci t

is an integer.

Proof. The first part follows from (i) For the second part, as stated, let d = gcd(a, b), and let (x 0 , y0 ) be a

specific solution of ax + by = n, So that axo + by0 = n. If ax + by = n, then ax + by - ax 0 - byo

= a(x - x 0 ) + b(y - Yo)= 0, or a(x - Xo) = b(Yo - Y), and hence (x - x 0 }~ = (y 0 - y }!: Since aid and
d d

b/d are relatively prime, b/d must divide x - x0 , and a/d must divide Yo - y.Let x - x0 = tb/d and

Y0 - y = ta/d. This gives the solutions described above.

I
·
medu~o~!?.C'11 i;,'!J,';;,~
® Corporate Office: CG Tower, A-46 & 52, IPIA, Near City Mall, Jhalawar Road. Kola (Raj.)-324005
1,, _.. _
W ebsite : www.resonance.ac.ln I E-mail : contar.l@reson,nr.e ar. .
_ ___ :._!"'I Fri,e : 1800 258 5555 j CIN U.1!0102RJ2007PLC(l?.4_ ~ _
Number Them" / -------------- _,----

. Example 1.

Let a and b be natural numbers and let q and r be the quotient and remainder respectively when
a 2 + b2 is divided by a + b. Deter.;iine the number q and r if q 2 + r = 2000. .
Sol. a2 +b 2 =q(a+b)+r
................................ (i)
2
+ r = as
2000Os r <(a+ b)
2
q = 2000- r
q 2 $ 2000
q :s; 44 ... (ii)
(a+ b)2
as - - - s (a 2 + b2 )
2 .
(a + b)2 :s; 2(a2 + b 2)
from
2
(Q
and (ii)
a + b < 45(a + b)
2
2(a + b7 ) < 90(a + b)
(a+ b) < 90
so r < 90
q
2
= 2000 - r > 2000 - 90 1910 =
q2 > 1910
q > 43 ... (iii)
From (ii) & (iii) q = 44
r = 64

Example 2.
A number n is called multiplicatively perfect if the product of all the positive divisors of n is n 2 '.
Determine the number of positive multiplicatively perfect numbers less than 100. ·
Sol. All multiplicativel y perfect numbers have exactly 4 distinct positive divisors, or 1. So, we must look
for numbers that are either - 1
a product of two distinct primes
a cube of a prime
Numbers satisfying one of these condition less than 100 are : 1, 6, 8, 10, 14, 15, 21, 22, 26, 27, 33,
34, 35, 38, 39, 46, 51, 55, 57, 58, 62, 65, 69, 74, 77, 82, 86, 87, 91, 93, 94, 95. There are 33 of
these

Self Practice Problems :

1. Find all 3-digit numbers which are the sums of the cubes of their digits.

Ans. 153,370, 371,407

The Fundamental Theorem of Arithmetic.


Every integer greater than 1 can be written uniquely in the form p~• p;• ..•. .p~ where the Pi are
distinct primes and the e1 are positive integers.

Note : There exist an infinite number of primes.


Proof. Suppose that there are a finite number of primes, say p1' p , •• • • Pn· let N = p p • • ·pn + 1. By the
2 1 2
fundamental theorem of arithmetic, N is divisible by some prime p. This prime p must be among the
Pi, since by assumption these are all the primes, but N is seen not to be divisible by any of the p
1
contradiction.

Ell R Corporate Office: CG Tower, A-46 & 52. IPIA, Near City Mall, Jhalawar Road, Kota (Raj.)-324005
- Edu~~el:1~/:!';;,~ 1-=Toll
W-:-:
eb"=s""'
lte~:~www
..,.,,..,.c;'
.r~es'="=on
Free : 1800 258 5555 CIN:
,...,a"=n'="
cec':.ac
~ .ln-'-'-:"E'e-
•m-"'a""ll":",c-
co'="nc.c
ta"""
ct-res
~ on"'an
="-'
ce"'
.ac
= .ln.;___ _
U80302RJ2007PLC024029
Number Theory / , - - - - - - - - - - - - - - - - - - - - - - - - - - •
Greatest integer function or step up function :
The function y = f (x) = [x] is called the greatest integer function where [x] equals to the greatest
integer less than or equal to x. For example :
(3.2] = 3; [- 3.2] = - 4
for - 1 s x < 0 ; [x] = - 1 ; for Os x < 1 (x] = 0
for 1 s x < 2 ; [x] = 1 ; for 2sx<3 [x] = 2 and soon.

Properties of greatest integer function :


(a) x- 1 < (x] s x (o) [x±m] = (x]±m iff mis an integer.
O; if x is an integer
(c) (x] + [y] s (x + y] s (x] + [y] + 1 (d) (x] + [- x1=
[ - 1 otherwise

Exponent of prime p In n! (Legender's theorem):

Let p be a prime number, n be a positive integer and Let EJn) denote the exponent cf the prime p in
the positive integer n. Then,

Ep(n!) = [~]+[;2 ] + [ ;3 ]+ ..... + [ ; ]

where s is the largest positive integer such that p's n < p••1

Example 3.
192
If (i2°°- 2 .31 + 2") is the perfect square of a natural number, then find the sum of digits cf 'n'
Ans. 18
Sol. 2 192(28 - 31) + 2" 2192.225 + 2' = m2
Z' = (m - 2 •15)(m + 296.15)
96

now let m - 296. 15 = 2a and m + 2.96.15 = 2'1"


Hence: 297• 15 = 2a-~ - 2'1
97 4
2'1(2~ - 1) = 2 (2 - 1)
a= 97 j3 = 4
2" = 2 2· ~
=
n 2a + j3 =198
-1- 2q 2q + 1 . I
p = - -- = - - not poss1b e
2-Sq Sq-2
Example4.
Find the number of quadruplets of positive integers (a, b, C. d} satisfying the following relations:
1 s a s b s c s d and ab + cd = a + b + c + d + 3
Ans. 4
Sol. ab - a - b + 1 + cd - c - d + 1 = 5
(a - 1)(b - 1) + (c - 1)(d - 1) = 5
case-1

Hence LHS 8
If follows that
a-1 = 0, 1
Case-1 a - 1 = 0
(c-1)(d-1)=5

R Aesonance• Corporate Office: CG T CNIM, A~6 & 52, IPIA. N - City Mall. Jhalawar Road. Kota (Raj.}-32-'005
(Ill Educalng fot .,._ _ , _ Website : www.resonance.ac.in E-mail : contact onanc:e.ac..in
Toll Free : 1800 258 5555 CIN: U80302RJ2007Pl.C024029
Number Theory /r----•--------------------•
c - 1 = 1 and d - 1 = 5 hence c = 2, d = 6
Now b - 1 can be 0, 1 (c b)
b = 1, c = 2, d = 6, a = 1
b = 2, c = 2 d = 6, a = 1
(a, b, c, d) = (1, 1,2,6) and (1,2,2,6)
Case-7 a-1 = 1
Now b - 1 can be 1, 2
but b - 1 = 2 not possible
b-1 = 1
(c-1)(d-1)=4
c -1 = 2 and d - 1 = 2 or c - 1 = 1 and d - 1 = 4
c = 3 and d = 3 c=2 d= 5
(a, b, c, d) = (2, 2, 3, 3) (2, 2, 2, 5)
Self Practice problems:
1. A three digit number is equal to the sum of the factorial of their digits. If the sufJ1. of all such three
digit numbers is ).. then find the sum of digit of)..

(2007)!
2. ( )' is an integer & n e N, then find maximum value of n.
2007
Answers.
1. 10 2. 9
Modular Arithmetic
For a positive integer m and integers a and b, we say that a is congruent to b modulo m
if m I (a - b), and we denote this by a = b modulo m, or more commonly a = b (mod m).Otherwise,
a is not congruent. to b modulo m, and we denote this by a= b (mod m) (although this notation is
not used often). In the above notation, m is called the modulus, and we consider the integers
modulom.

Useful Facts :

(i) If a = band c = d (mod m), then a+ c = b +. d (mod m) and ac = bd ..


(mod m).
.
Proof. If a=b and c = d (mod m), then there exist integers k and t , such that a = b + km and' c = d + Im.
Hence, a + c = b + d + (k + t)m, so
a + c = b + d (mod m). Also,
2
ac = bd + dkm + blm + ktm
= bd + (dk + bl + ktm)m,

so ac = bd (mod m)

(u. )F.or a1 . tegers n, n = {O}i


in
1
,mod4){ifn
2
. iseven.
.
n od. If IS

2
(iii) For all integers n, n "' l~)(mod8)
1 ifn= 1 (mod2)
1:;~: ~:=:~
(iv)' If f is a polynomial with integer coefficients and a = b (mod m), then f(a) = f(b)
(modm).

B Aesonance Corporate Office: CG T~r. A--46 & 52, IPIA, Near Ctty Mall, Jhalawar Road. Kola (Raj.)-324005
- Eclucdngfo,- 1_ , _ ~W;.,;;ebs~tte:...:;...;www":-=-:''r.;:es,:;on:..:;an::,'
=' =',ce:.:;.ac::;::,:.ln!.J.,=E~-m;:,:•:;:.ilc-=
: c,.,o:;:.nt,.,,_act~~res:::,.on,e:.:.,:a:,::nce=
.ac~-!!.!ln'-_
Toll Free : 1800 258 5555 CIN: UB0302RJ2007PLC024029
Number Theory /,__________,:___ _ _ _ _ _ _ _ _ _ _ _ _ _ _ __ _
Example 5. ' .'
Eleven pira\es find a treasure ch~st. When they split up the coins in it. they find that there are 5
coins left. They throw one pirate' overboard and split the coins again, only to find ~hat there are 3
coins left over. So, they throw another pirate over and try again. This time, the coins split evenly.
What is the least number of coins there could have been?
Sol. Let y be the number of coi ns in the chest. From the problem, we know that y "' 5 (mod 11 ), Y ,. 3
(mod 10}, and y .. 0 (mod 9). Combining these gives us that y .. 423 (mod 990), so the answer is
423
Example~
Let S denote the set of all 6-tuples (a, b, c, d, e, f) of positive Integers such that
a'+ b' + c' + d'. + e2 = f2. Consider the set ·
T = (abcdef: ('i!b, c,- d, e, f) e S}.
Find the greatest common divisor of all the me,nbers of T.
Ans. 24 · ·
Sol. We show that the required gcd is 24. Consider an element (a,d,c,d,e,f) e S. We have
a 1 + b2 + c' + d' + e2 = f2.
We first observe that not all a, b, c, d, e can be odd. Otherwise, we have a' "' b' - c> - d2 = .e2 .. 1
(mod 8) and hence f2 "' 5 (mod 3) which is impossible because no square can be congruent to 5
modulo 8. Thus at least one of a, b, c, d, e is even. '
Similarly if none of a, b, c, d, e is divisible by 3, then a' = b' "'c' ,. d',. e',. 1 (mod 3} and nence f2 .;
2 (mod 3) which again is impossible because no square is congruent to 2 modulo 3. Thus 3 divides
abcdef.
There are several possibilities for a, b, c, d, e.
Case -1 Suppose one of them is.even.and the other four are odd; say a is even b, c, d, e are odd.
Then b' + c' + d' + e2 - 4 (mod 8) .which agaio gives that 4Ia and 21 f so that 8Iaf. It follows that
8Iabcdef and hence 24labcdef. •,.
Case-2 Suppose a, b, are even anA c,-d, e are odd. Then c' + d' + e2 = 3 (mod 8) . Since a' + b' .. 0
or 4 modulo 8, it follows that f2 ,. 3 or 7 ·(mod 8) which impossible. Hence this case does not arise.
Case-3 If three of a, b, c, d, e are even a·nd two odd,· then 8labcde f and hence 24labcdef.
Case-4 If four o' a,b,c,d,e are even, then again 8labcdef and 24Iabcdef. Hence again for any six
tuple (a, b, c, d, e, f) in S, we observe that 24Iabcde f. Since
1' + 12 +1 2 + 2' + 3' = 42 .
We see that (1 , 1, 1, 2, 3, 4) e Sand hence 24 e T . Thus 24 is the gcd of. T.
Self Practice problems:
1. What j:t-the smallest positive integer t such that there exist integer n 1, n2 ,
4000
•••••• .• n1 with
(n,3 + n/ + n/ + ...... + n/ = 4000 }. · ·
. . 71
7
2. What are the last two digits of 7 ?
Ans. 1. 4 -· 2. 43

Fennat's theorem, Euler's theorem


Fennat's Little Theorem (FLT). If pis a prime, and p does not divide a, then aP--
1
= 1 (mod p).
Euler's Theorem. If a is relatively prime to m, then a.im>= 1 (mod m).

Proof. Let a 1, a2 , . .. , a+(m) be the positive integers less than m that are relatively prime tom. Consider
the integers ac11 , aa2, . .. , aa+(m)• We claim that they are a permutation of the original «m) integers
111. modulo m. For each i, aai is also relatively prime to m, so aai = ak for some k. Since

aa1 =aa1, a1• = ai (mod m), each a, gets taken to a different ak under multiplicatio:1 by a, so
indeed they are permuted. Hence,
a1a2 • • • a«m) = (aa,)(aa 2) • • • (aa«m) • a4>(m)a,a2 · · · a«m) =) 1 = (mod m)

n Corporate Office: CG Tower, A-46 & 62, IPIA, Neer City Mall, Jhalawar Road, Kota

Fl l"'lteSOnanCe
Educotln9 fo,botlwlomorr-
Website : www.reaonance.ac.ln E-malt : contact resonance.ac.ln
f--='C-""_:C.::..~~='7=~~;-<::;':'='2';:~=.~=':===--
Toll Free : 1800 258 5555 CIN:
Number Theory / - - - - - - - - - - - - - - - - - - - - - - - - - - •
Wilson's Theorem: (p - 1)! = -1 (mod p)

Useful Facts:
Note that if (p - 1)! * -1 (mod p) then pis not a prime, hence it is a test to determine whether Pis
prime or not
Example 7.
· Show that if a and b are relatively prime positive integers, then there exist integers m and n such
that am + bn = 1 (mod ab).

Sol. Let S =am + b" where m =q,(b) and n =«a). Then by Euler's Theorem, S = b«a) = 1 (mod a),

or S - 1 = 0 (mod a), and S = a«b) 1 (mod b),

or S - 1 = 0 (mod b). Therefore, S - 1 = O, or S = 1 (mod ab).


Example 8.

For all positive integers i, let S; be the sum of the products of 1, 2, ... , p-1
taken i at a time, where pis an odd prime. Show thats,= S2 =· ·· = Sp- 2 = 0 (mod p).
Sol. First, observe that
1 2
(x - 1)(x - 2) • • • (x - (p - 1))= xP-- - S1x P-- + S2x p-3 - • • • - Sp - 2x + Sp--1
p--1
This polynomial vanishes for x = 1, 2, . .. • p -1. But by Fermat's Little Theorem, so does x -1
modulo p. Taking the difference of these two polynomials, we obtain another polynom_ial of degree
p - 2 with p-1 roots modulo p, so it must be the zero polynomial, and the resuh follows from
comparing coefficients.

Self Practice Problems:

1. Show that if n is an integer greater than 1, then n does not divide 2" - 1.

Sol. Let p be the least prime divisor of n. Then gcd(n, p-1) =1 , and by Corollary 2.2, there exist integers
x and y such that nx + (p - 1)y = 1. If p I (2" - 1).

then 2 = 2nx+(p- 1)Y_= (2"t(2p- 1)Y =1 (mod p) by Fermat's Little Theorem, contradiction. Therefore,

p + (2" - 1) n - (2" - 1).

Arithmetic Functions
There are several important arithmetic functions, of which three are pre-sented here. If the prime
factorization ofnumber
prime to n, the n > 1 isofp~•p;• .p:• ,
.....of n isthen the number of positive integers less than n, relatively
divisors

~{n) =(1- P~J(1-


P:}{1-P:}
p~•-1 p;.-1••• .p:•- (p -1)(P2 -1} ...
1
1 (pk -1)
the number of divisors of n is
i:(n) = (e, + 1) (e2 + 1) .... (ek+1),

and the sum of the divisors of n is


Road, Kola (Raj.}-324005
R
a, Aesonancell)
EduudnglotbottortomO<TOW Corpo_rat~
Webs1le . www.resonance.ac.ln & 52,I ._IPIA,
Office: CG Tower, A-46E•ma1 Nearresonance.ac.ln
contact City Mall. ••
Jhalawar Toll Free : 1800 258 5555 CIN: U80302RJ2007PLC024029
Number Theory /,--------------------------1--
1 1
o(n)= + p~•- + ... +1)~;2 +p;2- + .... +1) .... +p:• - 1
+ ...... +1)

=(p~••1-1) (p~2•1·-1} ....(P=k+1 _1)


P1 -1 P2 -1 Pk -1
Also, cp (1 ), t (1 ), and o(1) are defined to be 1. We say that a function f is multiplicative
if f (mn) = f (m) f (n) for all relatively prime positive integers m and n, and f (1) =1 (otherwise,
f (1) = 0, which Implies that f (n) = Ofor all n).

(I) The functions it,, 1 , and o are multiplicative.


Hence, by laking the prime factorization and evc:iuating at each prime power, the formula above are
found easily.
(II) If (m,n)=1 then ip(mn)= cp (m) it,(n)
Proof : We have to choose elements :'l)m 1 to mn which are coprime tom as well as to n
1 m+1 2m+1 ...... (n-1 )m+1
2 m+2 2m+2 ...... (n-1)m+2

Let A= k m+k 2m+k ...... (n-1)m+k

m 2m 3m ......nm
1
No of element in k h sowwhich are coprime to n are ip(n) as (m,n)=1
We have choosen k such that (k,m)=1 as we (By proposition(ii))
are trying to find those element in k1h row which are also prime tom
1
As (k,m}=1 so all elements of k h row are coprime tom. As (k,m) = 1 so all elements of k1h row are
th
coprime to m. This means all the it,(n) elements taken from k row are coprime to n & so in each row
starting with
k 1,k 2, ...... ,k«m> we have it>(n) elements coprime to mn
total =it> (m) cp (n) cp (mn)= cp (m) it, (n)

(iii) If n 1 , n2, .. ,nk are mutually prime then cp (n1n2 .... nk)= cp (n,) it> (n2) .... it, (nk)

(iv) If n = p~•p;2......p:• is unique prime factorisation of n then cp (n)=n ( 1- ; J( 1- p: ). ... ( 1- p: )

1 1
Proof: cp(n)= it> (P1 P~)=it>(Pnit>(P~)=(pf- Pr )(p~- P~- )=P~P~(1-:J(1- p: )

(v) If p is prime then cp (pk) = pk - pk-, where k e N


Proof : It is true for k= 1
For k>1, numbers from 1 lo pk .which are not prime to pk are p(1 ), p(2), ........,p(pk- 1). These are
pk- 1 in number

:. ip (pk) = pk-pk-1 = pk ( 1 _ i)
Example 9.
7
Find number of natural numbers less than 10 which have exactly 77 divisors.
Ans. 2
10 6
Sol. N = 2 3 has 77 divisors and it is less than 107
6 10
N = 2 3 has 77 divisors and it is less than 107

Ifill R <1> Corporate ornce: CG Towor, A-46 & 52, IPIA, Noar City Mall, Jholownr Rood, Kota
Edu~~S,!;l~D,S.2 Website : www.rosonance.ac.ln E-mall : contoc1 resonnnca.ac.ln
Toll Free: 1800 258 5555 CIN : U80302RJ2007PLC024029
_N_u_m_b_er_Th_e_o__.,y'--------~/,--------------------•

• Exercise-1
PART- I
1. When the tens digit of a three digit number abc is deleted, a two digit number ac is formed. How many
numbers abc are there such that abc=9ac + 4c.

2. Consider two positive integer a and b. Find the least possible value of the product ab if
abba is divisible by 2000 ·

3. Find all solutions to aabb = n4 - 6n3, where a and bare non-zero digits, and n is an
integer (a and bare not necessarily distinct). ·

4. Find the least possible value of a +•b, where a, b are positive integers such that 11 divides a + 13b and
13 divides a+ 11b.

5. The sum of all three digit numbers each of which is equal to 11 times the sum of the squares of its
digits is "- . Find the sum of digits of "-·

6. N is 50 digit number (in decimal form). All digits except the 26th digit (from left) are 1. If N is divisible by
13, find the 26th digit. ·

4444
7. Find remainder when 4444 is divided by 9

8. . . .integers appear .1n the 1·1stl2006J


How many positive - - ,..., l2006J where Lx. j represents th e
- - , l .-,2006J
1 2 2006
greatest integer that does not exceed x. ?

9. Find number of positive integer less than 2431 and prime to 2431.

10. Find the smallest natural number n which has last digit 6 & if this last is moved to the front of the
number, the number becomes 4 times larger.

11. Does there exist an !riteger such that its cube is equal to 3n2 +' 3n + 7, n e I ?
,/ I
444
12. Let A be the sum of the digits of the number (4444)4 and B be the sum of the digits of the number A.
Find the sum of the digits of the number B.

13. For how many integers n is J9- (n + 2) 2


a real number?

14. The number of prime numbers less than 1 million whose digit~I sum is 2 is:

15. An eight digit number is a multiple of 73 and 137. If the second digit from left is 7, what is the 6th digit
from the left of the number?
. 15n2 +Sn+ 6
16. The number of natural numbers n for which----- is a natural number is :
n

17. Let A be the least number such that 10A is a perfect square and 35 A Is perfect cube. Then the number
of positive divisors of A is :

I nesonance® t°T'.r
n
Educating for better tomorrow
Corporate Office: CG Tower, A-46 & 52, IPIA, Near City Mall. Jhalawar Road. Kota (Raj.}-324005
Website : www.resonance.ac.ln E-mail : coniacl resonance.ac.ln
01:;;1.:F:::,e:::e-:-
: 7.1s;;:00=2s;;;s;-;s;-;:ss~s;:.;c::;;1N:-;-::";-U;;;8;;:03;;;0~2R~J;;;2:;;'.00~7~P::<:Lc.c:oaa2c:;40"'2;;;:9=~---
_N_u_m_b_e_r_Ti_ Z_,e_o"""ry"'------- ~/.,..----------------- ----•
18. The number of 2 digit numbers having exactly 6 factors is :

19. The number of positive integers 'n' for which 3rr-4, 4n-5 and 5n - 3 are all primes is :

20. The number of positive integral values of n for which (n3 - 8n 2 + 20n - 13) is a prime number is :

21. a, b, care digits of a 3-digit number such that 64a + 8b + c = 403, then the value of a+ b + c + 2013 is

22. N is a five digit number. 1 is written after the 5 digit of N to make it a six digit number, which is three
times the same number with 1 written before N. (If N = 23456 it means 234561 and 123456) . Then the
middle digit of the number N is

. 2 9
23. The sum of all values of integers n for which is also an integer is
n-1
24. The number of natural number pairs (x, y) in which x > y and + = 1 is :
X y

25. The number of positive integer pairs (a, b) such that ab - 24 = 2b is

26. A = (2 + 1) (22 + 1) (24 + 1) ...... (22048 + 1 ). The value of (A + 1 )112048 is

27. The least positive integer n such that 2015• + 2016• + 2017• is divisible by 10
is
15
28. b is an integer is
The number of pairs of relatively prime positive integers (a, b) such that b + 4a

29. The four digit number 8ab9 is a perfect square. The value of a2 + b2 is

30. a, b are positive real numbers such that ..!. + = 1 . The smallest value of a + b is
a b

PART- II ·
4
1. Prove that there exist infinitely many natural numbers 'a' with the property that the number p = n :- is
not prime for any natural number n.

2. For what values of natural numbers n can the product of the numbers n, n + 1, n + 2, n + 4, n + 5 be
equal to the product of remaining ones?
2 1
3. Determine all pairs (x, y) of positive Integers satisfying the equation 1 + 2' + 2 • • = y2.
113 113
4. Let a and b be two positive rational numbers such that (a) + (b) is also a rational number. Prove
113
that (a)113, (b) themselves are rational numbers.

3 3
5. Prove that the only solution in rational numbers of the equation x + 3y + 9z3 - 9xyz = o 15
X =y =Z = 0.
6. If a, b, x, y are integers greater than 1 such that a and b have no common factor except 1 t-'.•".1
x• = / show that x = nb, y = n" for some integer x > 1.

nesonance® Corporate Office: CG Towor, A-46 & 52, IPIA, Near City Mall, Jhalewar Road, Kota (Raj.)-3:?JO<: ;i
Website : www.resonance.ec.ln E-mail : conlacl resonance.ac.in
7

r"'(
Educatlng for better tomorrow 1--r'--'o~1 F""re=-a
1 "" : -=ue-"0""30:::..2:
e'"':'--",e'""oo=2sc::.e:.:.:s""'ss"'s=c"'"1N:..i. PRMO - 10•. 20=01"'p"'L""
......................................................................................................................................................................................................................................:R"'J"' co~2a:.;4=02""'9=:=..c.---
_ N_u_m_b_e_r_Th
_ e_o--'ry
' --____ __/,..- - - - - - - - - - - - - - - - - - - - - •

7. Show that there are an infinite number of primes of the form 4k + 1 and of the form 4k + 3.

8. A natural number n is said to have the property P if whenever n divides a" - 1 for some i nteger a, n2
also necessarily divides a" - 1.
(a) Show that every prime number has property P.
(b) Show that there are infinitely many composite numbers 'n' that possess property P.

9. Find all natural numbers n for which every natural number whose decimal representation has n -
1 digits 1 and one digit 7 is prime.

Prove that there are infinitely many positive integers n such that n(n + 1) can be expressed as a sum of two positive
squares in at least two different ways. (Here a>+ b2 and b' + a2 are considered as the same representation.)

Let O < a, < a2 < · · · < a .... , be mn + 1 integers. Prove that you can select either m + 1 of them no
11 .
one of which divides any other, or n + 1 of them each dividing the following one.

Let m and b be non-negative integers. Prove that m! n!(m + n)! divides (2m)!(2n)!
12.

For every integer


13. 25 3
n prove that the fraction n + cannot be further reduced.
15n+2

14. Let n be a positive integer and p, , p2 ..... Pn be n prime numbers all larger than 5 such that 6 divides
(p/ + p/ + p/ .....+ p/). Prove that 6 divides n.

15. Prove that 220 - 1 is divisible by 41

16. Prove that 53103 + 10353 is divisible by 39.

17. Let< p 1, p2, ··•Pn, ....> be a sequence of primes defined by p, = 2 and for n 1, Pn•1 is the largest
prime factor of p, P2
.............................. ,p" + 1 (Thus P2 = 3, p3 =7). Prove that Pn 5 for any n. *
18. Find the number of solutions in ordered pairs of positive integers (x, y) of the equation + =
X y n'
where n is a positive integer.

19. Prove that 1993 - 1399 is positive integer divisible by 162.

20. Show that product of any n consecutive integers Is always divisible by n!

• Exercise-2
PART-I

1
1. If = - where m and n are positive integers, what is the value of m + n.
+ - 1

[PRMO 2012]

Resonance® Corporate Office: CG Tower, A-46 & 52, IPIA, Near City Mall, Jhalawar Road, Kola (RaJ.}-324005
Website : www.resonance.ac.in E-mail : contact resonance.ac.ln
Educating fOf better lomooow r::
ro1'""1:=:F""
re-=-e.:..,::-; 1e=-=o::': o'-=2:=se="s==s==
s'=-s~c;:;1'::-;
N-': : u"'e"-=0"'3"0'2'=
R"'J2;:.:0""
01,:.::P~L""
c~o2!!!4~0~29='==!__-- -
_N_u_m_b_e_r_T._'h_
eo_ry.;:.__ _ _ _ __ _ , / , - - - - - - - - - - - - - - - - - - - - - •

2. Let p(n) = (n+1) (n+3) (n+5) (n+7) (n+9). What is the largest integer that is a divisor of p(n) for all
positive even integers n? (PRMO 2012)

3. How many non-negative integral values of x satisfy the equation

[i] = [ ~] ? (Here [x] denotes the greatest integer less than or equal to x.

(For example (3.4) = 3 and (-2.3) = - 3). [PRMO 2012)

2
4. What is the sum of the squares of the roots of the equation x - 7(x] + 5 = 0?
(Here [x] denotes the greatest integer less than or equal to x. For example [3.4) = 3 and (-2.3) = - 3).
[PRMO 2012)

5. Let S(M) denote the sum of the digits of a positive integer M written in base 10. Let N be the smallest
positive integer such that S(N) = 2017. Find the value of S(5N + 2017) (PRMO 2013)

6. Let Akbar and Birbal together have n marbles where n > o. Akbar says to Birbal "If I give you some
marbles then you will have twice a~ many marbles as I will have".Birbal says to Akbar" If I give you
some marbles then you will have thrice as many marbles as I will have" What is the minimum possible
value of n for which the above statements are true? (PRMO 2013)

7. To each element of the set S = {1, 2, 3 .... 1000}, a colour is assigned. Suppose that for any two
elements a, b of S, if 15 divides a + b they are both assigned the same colour. What is the maximum
possible number of distinct colours used [PRMO 2013]
3 3 3
8. What is the smallest positive integer k such that k(33 + 4 + 5 _+ 6 ) = a"for some positive integer a and
n with n > 1 ? (PRMO 2013]

9. Let f be a one-to-one function from the set of natural number to itself such that f(mn) = f(m) f(n) for all
natural number~ m and n. What is the least possible value of f(999) ? (PRMO 2014]

10. One morning, each member of manjul's family drank an 8-ounce mixture of coffee and milk. The

amount of coffee and milk varied from cup to cup but never zero. Manjul drank ( r of the total amount

of milk and (fr of the total amount of coffee. How many people are there in manjul's family ?

[PRMO 2014]

11. For how many natural numbers n between 1 and 2014 (both inclusive) is an · an integer?
9999-n
(PRMO 2014)

12. For natural numbers x and y, let (x,y) denote the gcd of x and y. How many pairs of natural numbers
x and y with x s y satisfy equation xy = x + y + (x,y)? [PRMO 2014)

13. Suppose f is a quadratic polynomial ie. a polynomial of degree 2 with leading coefficient 1 such that
f(f(x)+x) = f(x) (x2 + 786x + 439) for all real numbers x. What is the value of f(3) ? [PRMO 2015]

14. What is the greatest possible perimeter of a right angled triangle with integer side lengths if one of the
sides has length 12? [PRMO 2015]

Mesonance®
I
Corporate Office: CG Tower, A-46 & 52, IPIA, Near City Mall, Jhalawar Road, Kola (Raj.}-324005
f"lt Website: www.resonance.ac.ln E-mail : contact resonance.ac.ln
Educating for better tomorrow 1-T!!ol..::l~F~,e~e'-':::..:1s:..:0~0"'2'c'ss""'5;,;55"=s==-c=-"1N="":.L:'U=::8.:,:;03:::0e,,2=RJ"'=2""0'='01:'::!P'LC'=-0;;2""47'o2:'c9==---~
?-
_N_u_m_b_e_r_Th_e_o-'ry_ _ _ _ _ _ -,,/,..---------------------1•
2 2
15. Positive integers a and b are such that a + b = ! + ~. What is the value of a + b ? [PRMO 2015)
· b a

16. Let n be the largest integer that is the product of exactly 3 distinct prime numbers x, Y and 10x + Y
where x and y are digits. What is the sum of digits of n? [PRMO 2015)

17. The digits of a positive integer n are four consecutive integers in decreasing order when read from left
to right . What is the sum of the possible remainders when n is divided by 37 ? [PRMO 2015)

18. For positive integers m and n, let gcd (m,n} denote the largest integer that is a factor of both m and n.
Find gcd (2015! + 1, 2016! + 1) where n! denotes the factorial of a positive integer n. [PRMO 2015)

19. Find the total number of solutions to the equation x2 + y2 = 2015 where both x and y are integers.
[PRMO 2015)

20, For positive integers m and n, let gcd(m, n) denote the largest integer that is a factor of both m and n.
Find the sum of all possible values of gcd(a - 1, a2 + a + 1) where a is a positive integer. [PRMO 2015)

21. Let n!, the factorial of a positive· integer n, be defin~d as the product of the integers 1, 2 ...., n. In words,
2 2
n! = 1x2x ... xn. What is the number of zeros at the end of the integer 102! + 1 f! + 12 ! + ... +99 !
[PRMO 2015)

· 22. a, b, c, d are integers such that ad + be divides each of a, b, c and d.Prove that ad + be= ± 1
. . [PRMO 2016)

23. At .some integer points a polynomial with integer coefficients take values 1, 2 and 3. Prove that there
exist not more than one integer at which the polynomial is equal to 5. [PRMO 2016)

24. The five digit number 2a9b1 is a perfect square. Find the value of ab-1 + ba-1 • [PRMO 2016)

25. Find the number of integer solutions of [ ~O [ ~O ]] = 5 (Here [x] denotes the greatest integer less than
1 1
or equal to x. (For example [3.4) = 3 and [-2.3) = - 3). [PRMO 2016)

26. Integers 1,2,3 ........ n where n > 2, are written on a board. Two numbers m , k such that
1 < m < n, 1 < k < n are removed and the average of the remaining numbers is found to be 17. What is
the maximum sum of the two removed numbers? [PRMO 2017)

27. Suppose an integer r, a natural number n and a prime number p satisfy the equation
7x2 - 44x + 12 = p". Find_the largest value of p. [PRMO 2017]
3
28. Let p, q be prime numbers such that n pq - n is a multiple of 3pq for all positive integers n. Find the least
possible value of p + q. [PRMO 2017)

29. For each positive integer n, consider the highest common factor hn of the two numbers n! + 1 and
(n + 1}!. For n < 100, find the largest value of hn, · [PRMO 2017)

30. Consider the areas of the four triangles obtained by drawing the diagonals AC and BO of a trapezium
ABCD. The product of these areas, taken two at time, are computed. If among the six products so
obtained, two product are 1296 and 576, determine the square root of the maximum possible area of
the trapezium to the nearest integer. [PRMO 2017)

Corporate Office: CG Tower, A-46 & 52, IPIA, Near City Mall, Jhalawar Road, Kota (Raj.)-324005
Website : www.resonance.ac.ln E-mail: contact resonance.ac.ln
Educating for better tomorrow ~To1:;;1Fi'F'=re=e':-:~,s;;-;0;;;0~2?.59~5fi5~5s~Ca-;:IN;:;-:-'-iu';;;e';;:03:;;o'i:i2;:;RJ';';:2~00~1;'-;!P::"LC~o~2:":40::':2~9="-:!!!- --
_N_u_11_1b_e_r_Ti_71_e_o...
ry_ _ _ _ _ _ ~/r---------------------•
31. If a,b,c 4 are integers, not all equal, and 4abc = (a + 3) (b + 3) (c + 3), then what is the value of
a+ b + c? [PRMO 2018]

32. Let a and b natural numbers such that 2a - b, a - 2b and a + b are all distinct squares. What is the
smallest possible value of b? [PRMO 2018]

33. Consider all 6-digit numbers of the form abccba where b is odd. Determine the number of all such 6·
digit numbers that are divisible by 7. (PRMO 2018]

34. The equation 166 x 56 = 8590 is valid in some base b 10 (that is 1,6,5,8,9,0 are digits in base bin the
above equation). Find the sum of all possible values of b 10 satisfying the equation. [PRMO 2018]

35. What is the value of L, (i + j) _ L, (i + i) ? (PRMO 2018]


1ikjS10 1Si<j,;10
l+j• odd l+j•even

36. Let N = 6 + 66 + 666 + ..... + 666 ....66, where there are hundred 6's in the last term in the sum. How
many times does the digit 7 occur in the number N? [PRMO 2018]

37. Determine the sum of all possible positive integers n, the product of whose digits equals n 2 - 15n - 27.
[PRMO 2018]

PART-II

1. Show that there is no integer 'a' such that a2-3a-19 is divisible by 289. [RMO 2009)

2. A natural number n is chosen strictly between two consecutive perfect square. The smaller of these two
squares is obtained by subtracting k from n and the larger one is obtained by adding eto n. Prove that
n - kt is a perfect square. [RMO-2011)

3. Prove that for all positive integers n, 169 divides 21 n2 + 89n + 44 if 13 divides n' + 3n + 51 .
[RMO-2012)
4. Determine with proof all triples (a, b, c) of positive integers satisfying + + = 1, where a is a prime
a b c
number and a 5 b 5 c. [RMO-2012]

5. Let a,b,c be positive integer such that a divides b', b divides C3 and c divides a 3 • Prove that abc divides
(a+ b + c)13• [RMO-2012]

6. Find all triples (p, q, r) of primes such that pq = r + 1·and 2(p2 + q 2 ) = r2 + 1. [RMO-2013)

7. Let a,, b, , c, be natural numbers. We define [RMO-2013)


a2 =gcd(b,, c,), b2 = gcd(c, . a,) , c2 =gcd(a,, b,),
and
=
a, lcm(b2 , c,) , b3 =lcm(c,, a 2
) ,

Show that gcd(b3, c,) = a,.

8. For any natural number n, expressed in base 10, let S(n) denote the sum of all digits of n. Find all
natural numbers n such that n = 2S(n)2. [RMO-2016)

nesanance®
r"'t
Corporate Office: CG Towor, A-46 & 52, IPIA, Near City Mall, Jhalawar Road , Kota (Raj.}-32400S
Website : www.resonanca.ac.ln E-mail : contact resonance.ac.ln
EducaUng for better tomorrow : 1"=e"'='oo;:.:2='s'=='e'='ss='s~s:,:
i-,;,:To1""1"=F::::,e:..:e-" .
: L: = c="1"'-N u=,,e~oJ""o'==2R~Jc:'20='=0':'1P~L"=c~o2="4~02='=9:-===---
_M_u_m_b_e_r_Th_e_o...:ry:___ _ _ _ _ .....; / , - - - - - - - - - - - - - - - - - - - - - - - , •

9. For any positive integer n, let d(n) denotes the number of positive divisors of n; and let ~(n) denote the
number of elements from the set (1, 2, ..... n} that are coprlme to n. [RMO-2017]
(For example, d(12) =6 and ~12) = 4.)
Find the smallest positive integer n such that d(,(n)) = 2017.
3 4 5 7
10. Show that there are infinitely many tuples (a, b, c, d) of natural numbers such that a + b + c = d
[RMO-2018]

om
11. Find all natural numbers n such that 1+ [ En] divides 2n. [RMO-2018]

12. For a rational number r, its *period* is the length of the smallest repeating block in its decimal

t.c
expansion. for example, the number r = 0.123123123 ................ has period 3. If S denotes the set of all rational
numbers of the form r = abcdefgh having period 8, find the sum of all elements in S. [RMO-2018]

po
gs
lo
e.b
or
lst
ia
fic
of
er
rn
co
o ok
.b
w
w
w
Number Theory /

•• Answers
Exercise-1 l
]

PART- I
1. 6 2. 20 3. a= 6, b=5 4. 28 5. 12
6. 3 7. 7 8. 88 9. 1920
10. 153846 11. No 12. 7 13. 7 · 14: 3
15. 7 16. 4 17. 72 18. 16 19.
20. 3 21. 2024 22. 8 23. 8 24. 3
25. 8 26. 4 27. 2 28. 4 29. 52
30. 16

PART - II
2. No value 3. (4, 23) 9. n = 1, 2

18. (2e1 + 1 )(2e2 + 1) · · · (2ek + 1 ), where n = p~•p;• ...p=•


1.
6.
2011
12
Exercise-2 ]

2.
7.
15
8
PART- I
3.
8.
9
3
4.
9.
69 5. 6
24 10. 8
11. 12. 3 13. 2015 14. 84 15. 2
16. 12 17. 217 18. 19. 0 20. 4
21. 24 23. 15 24. 21 25. 50 26. 51
27. 47 28. 28 29. 97 30. 13 31 . 16
32. 21 33. 70 34. 12 35. 55 36. 33

37. 17

PART- II
4. (2.5,30) (2,6,18)(2,7,14) (2,8,12) (2.10,10) (3,4,6) (3,6,9) 6. (2, 3, 5), (3, 2, 5)
2011
8. 50,162,392,648 9. 2

nesonance®
M
Corporate Office: CG Tower, A-46 & 52, IPIA, Near City Mall, Jhalawar Road, Kota (Raj.)-324005
Website : www.res0f1ance.ac.ln E-mail : COf1tact res0f1ance.ac.in
EducaUng for better tomorrow 1N:-::':';';ua:.':o;:;';30;;';;2;;-RJ-;:;2~00~7P;!;:L~C""'o2""40""2~9="'-'-- - -
t-:;::Tol:;;-1-;::Fr=ee:-::'1°'80;;;-0-;;;25'°67ss;;;:s'rs;";c:::':
w
w
w
.b
o
Equations

ok
co
rn
er
of
fic
ia
lst
or
e.b
lo
gs
po
t.c
om
-~q E ' /-------------~-
_ ,-
-
........u_a_t1_o_n_s_______
IEquations I
Intervals:
Intervals are basically subsets of R and are commonly used In solving lnequalllles or In finding
domains. If there are two numbers a, b e R such that a < b, we can define four types of Intervals as
follows :

Name Representation Dlscrlptlon

Open Interval (a. b) {x : a < x < b) I.e. end points are not Included.

Close lnterv al
(x : a s x s b) I.e. end points are also Included. This Is possible only
[a. b)
when both a and bare finite.
Open - Closed Interval (a. b) (x : a < x s b} I.e. a Is excluded and b Is Included.
Close• Open Interval [a, b) (x : a s x < b} I.e. a Is Included and b Is excluded.
Note: (1) The infinite Intervals are defined as follows :
(I) (a,a:i)=(x:x>a} (II) (a, a:i) = {x : x a}
(Iii} (- a:i, b) = {x : x < b} (Iv) (-«>, b) = {x : X :S b}
(v) (- a:i, a:i) = {x : x e R)
(2) x e {1, 2} denotes some particular values of x, I.e. x = 1, 2
(3) If there is no value of x, then we say x e (null set)
General Method to solve Inequalities :
(Method of intervals (Wavy curve method)

Let g(x) =
t
(x-b1 t •(x-b2 2 ---(x-b
n
t• J ... (i)
( (x - a )'' (x - a )'2 - - -( x - an)'•
1 2
Where k 1, k 2 ••••.kn and r,, r 2 ••......•rn e N and b 1, b2 ..•••bn and a,. a2 •••••.an are real numbers.
Then to solve the inequality following steps are taken.
Steps : -
Points where numerator becomes zero are called zeros or roots of the function and where denominator
becomes zero are called poles of the function.
(i) First we find the zeros and poles of the function.
(ii) Then we mark all the zeros and poles on the real line and put a vertical bar there dividing
the real line in many intervals.
(Ill) Determine sign of the function in any. of the interval and then alternates the sign in the
neighboring interval if the poles or zeros dividing the two interval has appeared odd number of
times otherwise retain the sign.
(iv) Thus we consider all the intervals. The solution of the g(x) > 0 is the union of the intervals
in which we have put the plus sign and the solution of g(x) < 0 is the union of all intervals in
which we have put the minus sign.
Example# 1:

Solve the inequality if f(x) =


(x - 2)10 (x + 1)3 ( x - J
.!.2 (x + 8)2
is> 0 or< 0.
x 24 (x-3)3 (x+2)5
Solutlon.
5
{x-2}10 (x+1)3 ( x - -1 ) {x +8)2
2
Let f(x) = the poles and 2eros are 0, 3,- 2,- 1, ..!. ,- 8, 2
x24 (x - 3}3(x + 2)5 2

Aesonance®
EducaUng for better tomorrow
COfJ)orate Office: CG Tower, A-46 & 52. IPIA. Near City Mall. Jhalawar Road. Kola (Raj.)-324005
Website : www.resonance.ac.ln E-mail : contact resonance.ac.ln
Toll Free : 1800 258 5555 CIN: U80302RJ2007PLC024029
s _____.J/,-------------------------,m--
_E_q_u_a_li_o_n_

/+

If f(x) > 0, then x e (- oo, - 8) u (- 8, - 2) u (- 1, 0) u ( o,-i-) u (3, oo)

and if f(x) < 0, then x e (- 2, - 1) u ( f 2) u (2, 3) Ans.

1. Polynomial :
A function f defined by f(x) = a.x• + a0 _ ,x•- 1 + .......+ ci,x + a0
where a0 , a,, a2 , ......, a. e R is called a polynomial of degree n with real coefficients (a. 0, n e W).
If a0, a,, a 2, ....., _a. e C, it is called a polynomial with complex coefficients.

2. Quadratic polynomlal & Quadratic equation :


A polynomial of degree 2 is known as quadratic polynomial. Any equation f(x) = 0, where f is a
quadratic polynomial, is called a quadratic equation. The general form of a quadratic equation is
ax2 + bx + c = O ..•..•. (i)
Where a, b, c are real numbers, a 0.
If a = 0, then equation (i) becomes linear equation.
3. Difference between equation & Identity:
If a statement is true for all the values of the variable, such statements are called as identities. If the
statement is true for some or no values of the variable, such statements are called as equations.
Example : (i) (x + 3)2 = x 2 + 6x + 9 is an identity
(ii) (x + 3)2 = x 2 + Gx + 8, is an equation having no root.
(iii) (x + 3)2 = x 2 + Sx + 8, is an equation having - 1 as its root.
A quadratic equation has exacUy two roots which may be real (equal or unequal) or imaginary.
a x 2 +bx+ c = 0 is:
* a quadratic equation if a 0 Two Roots
* a linear equation if a = 0, b 0 One Root
* a contradiction if a = b = 0, c 0 No Root
* an identity if a=b=c=0 Infinite Roots
If aY 2 + bx + c = 0 is satisfied by three distinct values of' x ', then it is an identity.
Example#2:
2 2 2
(a2 - 1 )x + (a + Sa + 4 )x + (a + 4a + 3) = 0. Find the value of a for which
(i) equation has two roots
(ii) Equation has one real root
(iii) It is an identity
Solution:
(i) a ±1 (ii) a =1 (iii) a =-1
4. Relation Between Roots & Coefficients:
(I) The solutions of quadratic equation, ax2 +bx+ c = 0, (a~ 0) is given by
- b±Jb2 -4ac
x=--~---
2a
2
The expression, b - 4 a c"' Dis called discriminant of quadratic
equation.
(II) If a, p are the roots of quadratic equation,
ax
then2 +bx+c=0 .......(i)
equation (i) can be written as
a(x - a)(x - P) = 0

Corporate Office: CG Tower. A-46 & 52, IPIA. Near City Mall, Jhalawar Road, Kota (Raj.)-324005
Aesonance®
Educating for better tomorrow
Website : www.resonance.ac.ln E-mail : contact resonance ac.in
Toll Free : 1800 258 5555 CIN: U80302RJ2007PLC024029
..._·_ _ _ __,/_ _ _ _ _ _ _ _ _ _ _ _ _n__
-Equat,ons . _,.
or ax 2 - a(a + f3)x + a af3 = 0 ..... (ii)
equ~tions (i) and (ii) are identical,
b
:. by comparing the coefficients sum of the roots, a+ f3 = - - = - coefficien t of x
ffi . t f 2
a coe IcIen o x
c
and product of the roots, af3 = - = constant
.
term
a coefficient of x 2

(iii) Dividing the equation (i) by a, b C


x2 + -x + - = 0
a a
x2 - (-:) x +-; = O x2 -(a + f3)x + af3 = 0

x2 - (sum of the roots) x + (product of the roots)= 0


Hence we condude that the quadratic equation whose roots are a & f3 is 2
x - (a + f3)x + af3 = 0
Example# 3:
If a and p are the roots of ax 2 + bx+ c = O, find the equation whose roots
are a+2 and f3+2.
Solution:
Replacing x by x - 2 in the given ec1uation, the required equation is
a(x - 2)2 + b(x - 2) + c = O i.e., ax2 - (4a - b)x + (4a - 2b + c) = 0.
Exampl e#4:
The coefficient of x in the quadratic equation x2 + px + q = O was taken
as 17 in place of 13, its roots
were found to be - 2 and - 15. Find the roots of the original equation.
Solution :
Here q =(- =
.?) >< (-15) = 30, correct value of p 13. Hence original equation is
x2 + 13x + 30 =Oas (x + 10) (x + 3) = 0
roots are - 10, - 3

Example # 5:
If a is a root of x 2 - 3x - 5 = 0 find the value of a' - 2a3 - 7a2 - Ba
Solution:
Note that given expression is (a2 - 3a -5) (a2 + a+ 1) + 5 hence value
of the expression is 5.
Self practice problem s :

(1) If a, p are the roots of the quadratic equation cx2 - 2bx + 4a = 0 then find
the quadratic equation whose
roots are

(i) a 2, f3 2 (iii) a+1,f3+ 1 (iv) 1+a 1+ f3


(v)
1-a'1 - p

(r+1) 2 2
(2) If r be the ratio of the roots of the equation ax 2 + bx + c =0, show that - b
- =- .
r ac
(3) If roots of the equation x 2 - 10ax - 11 b = 0 are c and d and those of x 2 -
1Ocx - 11d = O are a and b,
then find the value of a+ b + c + d. (where a, b, c, d are all distinct numbers
)
Answers:
(1) (I) cx2 - bx + a = 0 (Ii) ~x2 + 4(b2 - 2ac)x + 16a2 = O
(Ill) cx2 - 2x(b + c) + (4a + 2b + c) = 0 (Iv) (c - 2b + 4a)x2 + 2(4a - c) x + (c + 2b + 4a) = 0
(v) 4acx2 2
+ 4(b - 2ac) x + 4ac = 0
(3) 1210

Reso nance~
EducaUng for btlttr tomolTOW
Corporate Office: CG Tower, A-46 & 52, IPIA, Near City Mall, Jhalawar
Website : www.resonance.ac.ln E-mail : contact resonance.ac.ln
Road, Kola (Raj.}-324005

Toll Free : 1800 258 5555 CIN: U80302RJ2007PLC024029


_E_q_u_a_ti_o_n_s______ , , / ' " - - - - - - - - - - - - - - - - - - - - - - - - 7 •

5. Nature of Roots:
Consider the quadratic equation, ax2 +bx+ c = O having a, pas its roots;
D=b2 -4ac

D=O DaoO
Roots are equal i.e. a= p = - b/2a Roots are unequal
& the quadratic expression can be expressed
as a perfect square of a linear polynomial

a, b, c e R & D > 0 a . b, c e R & D < 0


Roots are real Roots are imaginary a= p + i q, P = p - i q

a, b, CE a & a, b, CE a &
D is square of a rational number D is not square of a rational number
Roots are rational Roots are irrational
.J, i.e. a= p +Jo., J3= p-Ja.
a = 1. b, c e I & D is square of an integer
Roots are integral.

6. Graph of Quadratic Expression :


* the graph between x, y is always a parabola.

r the co-ordinate of vertex are


.
(-J:_ ,__Q_)
2a 4a
* If a > O then the shape of the parabola is concave upwards & if a < O then the shape of the
par2bcla is concave downwards.
y "I~ ax~+ bx + c
(a <O)
vertex (-~.-~)
2il 4i1

* the parabola intersect the y-axis at point (0, c). · ·

* the x-coordinate of point of intersection of parabola with x-axis are the real roots of the
quadratic equation f (x) = 0. Hence the parabola may or may not intersect the x-axis.

Example# 6:
For what values of m the equation (1 + m)x2 - 2(1 + 3m)x + (1 +Sm)= Ohas equal roots.
Solution:
Given equation is (1 + m) x 2 - 2(1 + 3m)x + p +Sm)= 0 .........(i)
Let D be the discriminant of equation (i).
Roots of equation (i) will be equal if D = 0.
or 4(1 + 3m)2 -4(1 + m) (1 + 8m) = 0
or 4(1 -+ 9m 2 + 6m - 1 - 9m - 8m 2 ) = 0
or m 2 - 3m 0 = or, m(m - 3) 0 =
m =O. 3.

Corporate Office; CG Tower, A-46 & 52. IPIA, Near City Mall. Jhalawar Road, Kata (Raj.)-324005
Websll1,; www.resonance.ac.ln E-mail ; contact resonance.ac.ln
·r:-~:
:';i;e : 1800 258 5555 CIN; U80302RJ2007PLC024029
_E_.q_u_a_ti..;.o.;..ns_ _ _ _ ___,,
-/,--------------------------•

Example# 7 : Find all the integral values of a for which the quadratic equation (x - a) (x - 10) + 1 = 0 has
integral roots.
Solution: Here the equation is x2 - (a+ 10)x + 10a + 1 = 0. Since integral roots will always be rational it
means D should be a perfect square.
From (i) D = a2 - 20a + 96.
D=(a-10)2 -4 4=(a-10)2-D
If D is a perfect square it means we want difference of two perfect square as 4 which is
possible only when (a - 10)' 4 and D 0.= =
(a-10)=±2 a=12,8
Ex:,imple # 8 : If the roots of the equation (x - a) (x - b) - k = o be c and d, then prove that the roots of the
+
equation (x - c) (x - d) k = 0, are a and b.
Solution : By given condition (x - a) (x - b) - k = (x - c) (x - d)
or (x - c) (x - d) + k = (x - a) (x - b)
Above shows that the roots of (x - c) (x - d) + k = 0 are a and b.
Example# 9: Determine·:~. such that x2 - 11x + a and x2 -14x + 2a may have a commo,:i factor.
Solution : Let x - a be a common factor of x' - 11x +·a and x2 - .14x + 2a.
Then x = a will satisfy the equations x' -11x +a= O and x2 - 14x + 2a = 0.
a.2 -11a. +a= 0and a.2 -14a. + 2a = O
Solving (I) and (ii) by cross multiplication method,.we get a= 0, 24.
Example# 10 : Show that the expression x' + 2(a + b + c)x .+ 3(bc + ca + ab) will be a perfect square
ifa=b=c. ·
Solution : Given quadratic expression ·will be a
perfect square if the discriminant of its corresponding
equation is zero.
i.e. 4(a + b + c)' - 4.3 (be+ ca+ ab)= 0
or (a + b + c)2 - 3(bc + ca+ ab)= 0
1 ' '
or ((a - b)2 + (b - c)2 + (c - a)')= O
2
which is possible only when a = b = c.

Self practice problems :

(1) For what values of 'k' the expression (4 - k)x2 + 2(k + 2)x + Bk + 1,will be a perfect square?

(2) If (x- a) be a factor common to a 1x 2 + b,x +c and_a2x2 + b,x + c, then prove that
a(a, - a,) = b, - b,.

(3) If 3x' + 2axy + 2y' + 2ax - 4y + 1 can be resolved into two linear factors, Prove that 'a.' is a root
of the equation x 2 + 4ax + 2a2 + 6 = 0.

(4) Let 4x2 ·- 4(a- 2)x + a - 2 = O (a E R) be a quadratic equation .. Find the values of 'o.' for which
(I) Both roots are real and distinct.
(ii) Both roots are equal.
(iii) Both roots are imaginary
(iv) Both roots are opposite in sign.
(v) Both roots are equal in magnitude but opposite in sign.

(5) If P(x) =
ax2 + bx + c, and Q(x) = - ax• + dx + c, ac 0 then prove that P(x) . Q(x) =0 has
atleast two real roots.
Answers.

(1) 0, 3
(4) (I) (- «>, 2) v (3, «>) (ii) a. E {2, 3} (iii) (2, 3) (Iv) (- co, 2) (v) t

Resonance®
Educating for btlter tomorrow
Corporate omce: CG Tower, A-46 & 52, IPIA, Near City Mall, Jhalawar Road, Kola (RaJ.)-324005
Website: www.rasonance.ac.ln E-mail: contact resonance.ac.ln
Toll Free: 1600 258 5555 CIN: U60302RJ2007PLC024029
---'E'"'q_ll_at_ioc..n.;,,;s;___ _ _ _ ~/,...----------------------7•
7. Common Roots:
Consider two quadratic equations, a, x2 + b, x + c, =O & a x 2
2 + b,x + c, =0.
(I) If two quadratic equations have both roots common, then the equations are identical and their
co-efficient are in proportion.

I.e. = .:.!.
a2 b2 C2
(II) If only one root is common, then the common root 'a' will be :
a= c, a2 - c 2 a1 = b1 c 2 - b2 c 1
a, b 2 - a 2 b1 c 1 a 2 - c 2 a,
Hence the condition for one common root is :

(c 1 a 2 -c 2 a,)2 = (a1 b 2 -a2 b1)(b1 c 2 -b2 c 1 )


Note': I~ f(x) = 0 & g(x) = 0 are two polynomial equation having some common root(s) then those common
. root(s) is/are also the root(s) of h(x) a a f(x) + bg (x) = 0.

Example # 11 :
2
If x - ax + b = 0 and x 2 - px + q = 0 have a root in common and the second equation has equal roots,
show that b + q = ap .
2
Solution:
Given equations are : xx 22 -- ax b==00.
px + q
........ (i)
........ (ii)
and
Let a be the common root. Then roots of equation (ii) will be a and a. Let p be the other root
of equation (i). Thus roots of equation (i) are a, p and those of equation (ii) are u, 11..

Now a + p= a
........ (iii)
aP = b ....... (iv)
2a = p ....... (v)
a2 = q ........ (vi)
L.H.S. =b + q =afl + a = a(a + P) 2
..... (vii)
ap (a+A) 2a
and R.H .S. =
2 = ; =a (a+ P) ...... (viii)

from (vii) and (viii), L.H.S. = R.H.S.

Example#12:
If a, b, c e R and equations ax 2 + bx + c =0 and x 2 + 2x + 9 = 0 have a common root, show
that a : b : c = 1 : 2 : 9.
Solution:
Given equations are : x2 + 2x + 9 =0 ........(i)
and ax2 + bx + c =0 ........(ii)
Clearly roots of equation (i) are imaginary since equation (i) and (ii) have a common root, therefore
common root must be imaginary and hence both roots will be common.
Therefore equations (I) and (II) are identical

!: .!: =
2 9
a : b :c"'1: 2 : 9

Aesonance®
EducaUng for better tomorrow
Corporate Office: CO Tower. A-46 & 52, IPIA. Near City Mall, Jhalawar Roarl. Kt'IA (Rai.)-324005
Website : www.resonance.ac.ln E-mail : contact resonance.ac.in
Toll Free : 1800 258 5555 CIN: U80302RJ2007PLC024029
_E_.g.._u-'-a-'ti-'-o'--n_s_ _ _ _ __,/

Self practice problems:


(1) If the equations ax2 + bx + c = O and x 3 + x - 2 = O have two common roots then show that
2a = =
2b c.
· · . a b c
(2) If ax2 + 2bx + c = O and a x 2 + 2b x + c, = 0 have a common root and-, -b , are in A.P.
1 1
a1 1 C1
show that a,, b,, c, are in G.P.

8. Theory Of Equation~ :
If a,. °t. a 3 ..........a 0 are the roots of the equation;
f(x) =a.,x" + a,x""' + a x""' + ... + a..,x + a
2 0
= O where a0 , a,, .... ,a0 are all real & a 0 0 then,
L. a, =-~ , L. a,'¾ =+~ , L,, a, a 2 a 3 -__ 83 , ••••. , a , a 2 a.3 ...........a" =(-1)"8"
ao ao ao ao
Note:
(I) If a is a root of the equation f(x) = 0, then the polynomial f(x) is exactly divisible by (x - a) or (x
- a) is a
factor of f(x) and conversely. '
(ii) Every equation of n" degree (n 1) has exactly n roots & if the equation has more than n roots, it is an
identity.

(ill) If the coefficients of the equation f(x) = O are all real and a + ii} is its root, then a - ii} is also a
root. i.e.
imaginary roots occur in conjugate pairs.
(Iv) An equation of odd degree will have odd number of real roots and an equation of even degree will have
even numbers of real roots.

(v) If the coefficients in the equation are all rational & a + is one of its roots, then a - is also a root
where a, 13 e Q & 13 is not square of a rational number.
(vi) If there be any two real numbers 'a' & 'b' such that f(a) & f(b) are of opposite signs, then
f(x) = 0 must have odd number of real roots (also atleast one real root) between 'a' and 'b '.
' equation f(x)
(vii) Every = 0 of degree odd has atleast one real root of a sign opposite to that of its last term.
(If coefficient of highest degree term is positive).

Example# 13 : ·
If 2x' + 3x 2 + 5x + 6 = 0 has roots· a , 13, y then find a+ 13 + y, al} + l}y + ya and al}y.
Solution:
Using relation between roots and coefficients, we get
3 5 6
,
2,
a + 13 + y = = - al} + l}y + ya = al}y = - = -:- 3.
2 2
Self practice problems :

(1) If 2p'- 9pq + 27r =0 then prove th9t the roots of the equations rx3 - qx2 + px -1 =o are in H.P.
(2) If a, 13, y are the roots o! the equation x' + qx + r = 0 then find the equation whose roots are
(a) 2a + 213 + y, a+ 213 + 2y, 2a + 13 + 2y (b)

(c) (a+ 13)2, (13 + y)2, (y + a)l (d)


Answers :
(2) (a) x>+ qx-r = 0 (b) X3 - qx2 - r 2 = O
(c) x3 + 2qx2 + q 2 x - r2 = 0 (d) x' - 3x'r + (3r2 + q') x - r3 = O

Reson ance®
EducaUng for better tomorrow
Corporate Offk:e: CG Tower, A-46 & S2, IPIA, Near City Mall, Jhalawar Road, Kota (Raj.)-32400S
Webslle : www.resonance.ac.ln E-mail: contact resonance.ac.ln
Toll Free : 1800 2S8 5SS5 CIN: U80302RJ2007PLC024029
_£_.q_u_a_ti_on_s'------~/,------------------------,m-
Example 14:
let u, ll, y are the roots of the equation x3 + 2x2 + 3x + 1 = O. If the value of

( _!_ + _!_ _
131 y3 a3
J...) (J...
aJ
+ _!_ -_!_) + (J...
y3 p3
+ _!_ __!_J
a3 p3 y3
(ia3
+ _!_ __!_) + (J...
y3 p3
+ _!_ -_!_) (_!_ + _!_ -~)
a3 p3 y3 p3 y3 a
is

l. Then find the sum of digit of Ill


Solutlon:

3
X + 2x2 + 3x + 1= 0/;
"--r
find the equation whose roots are a 3, p3, y3
3 2
x -7x +12x+1=0
1
change x -
X

a+b+c=-12
roots are b + c - a, a + c - b, a + b - c
=-12-2a, -12-2b, -12-2c
equation is
3 2
(12 +x) +
12
(12+x) + 7(12+x) +
11
=0
8 4 2
x 3 + 12x2 - 172x - 2152 = 0
Ill= 172

9. Rarige of Quadratic Expression f(x) • ax2 + bx + c.


(I) Range:

If a> 0 ~a ,oo)
f (x)e [-

If a< 0 f(x)e (-oo,- ~a]

Hence maximum and minimum vall1es of the expression f (x) is - ~a in respective cases and it occurs

b
at x =- - (at vertex).
2a

(ii) Range In restricted domain:

Given x e [x,, x 2]
b
(a) If - - ti! [x,, X2] then,
2a
f (x)e [min{f(x1).f(x 2)}, max{f(x1),f(x 2 )})
b
(b) If - - e (x,, x 2) then,
2a

f (x)e [ min { t(x;), f (x2 ), - ~a}, max { t(x1), f (x2), - ~a}]


Corporate Office: CG Tower, A-46 & 52, IPIA, Near City Mall, Jhalawar Road, Kola (Raj.}-324005
Website : www.resonance.ac.ln E-mail : contact resonance.ac.in
Toll Free : 1800 258 5555 I CIN: U80302RJ2007PLC024029
_ E__.q._u_a_ti..;.o_n.:..s_ _ _ _ _ ...,/r---- ----- ---------- ---- - •
10. - Stgn of Quadratic Expressions :
The value of expression f (x) = a x2 + bx + c at x = x is equal to y-co-ordinate of the point on parabola
0
y = a x 2 + bx + c whose x-co-ordinate is x • Hence if the point lies above the x-axis for some x = x 0 ,
0
then f (x0 ) > 0 and vice-versa.

We get six different positions of the graph with respect to x-axis as shown.

(i)
~-\ / Conclusions :

(a) a>O
(b) D>O
(c) Roots are real & distinct.
(d) f(x) > 0 in x e (- oo, a) u (13, oo)
(e) f(x) < 0 in x e (a, P)

(ii)~ (a) a>O


a X

(b) D=O
(c) Roots are real & equal.
(d) -f(x) > 0 in x e R - {a}

(iii) V
- -- -•
(a) a>O

(b) D<O
{c) Roots are imaginary.
(d) f(x) > 0 'v x e R.

(iv) / \ ' (a) a<O

(b) D>O
(c) Roots are real & distinct.
(d) f(x) < 0 in x e (- 00, a) u (13, oo)

(v)n· (e)

(a)

(b)
f(x) > 0 in x e (a, P)

a<O

D =O
(c) Roots are real & equal.
(d) · f(x) < 0 in x e R - {a}

- - - --x
(vi) /\ (a) a<O

(b) D<O
(c) Roots are imaginary.
(d) f(x) < 0 'v x e R.

Resona nce®
Educating for better tomorrow
Corporate Office: CG Tower, A-46 & 52, IPIA, Near City Mall, Jhalawar Road, Kola (Raj.)-324005
Website : www.resonanca.ac.ln E-mail : contact resonanca.ac.ln
Toll Free : 1800 258 5555 CIN: U80302RJ2007PLC024029
_E__.q._u_a_11_·o_n_s_ _ _ _ _ ....., / , . - - - - - - - - - - - - - - - - - - - - - - - - - •

Example# 15: If c < 0 and ax2 +bx+ c = O does not have any real roots then prove that
(I) a- b + c < 0 (ii) 9a + 3b + c < 0.
Solution: c < 0 and D < 0 f(x) = ax2 +bx+ c < 0 for all x e R
f(-1)=a-b+c<O
and f(3) = 9a + 3b + c < 0

Example# 16: Find the range of f(x) = x 2 - Sx + 6.

Solution: 5 24
minimumoff(x)=- ~a atx=-:a =-('.'! ~ ) atx=%
4
maximum of f(x) t0
Hence range is [-¾, t0)
Example# 17 : Find the range of rational expression y = x:-x + 4 if xis real.
X +x+4
2
x -x +4
Solution: y = --=--- (y - 1 )x 2 + (y + 1) X + 4(y - 1) = 0 ......... (i)
2
x +x+ 4
case-I : if y 1, then equation (i) is quadratic in x
and xis real

D <!: 0 (y + 1) 2 - 16(y - 1)2 <!: 0 (Sy - 3) (3y - 5) s 0

ye[¾,%] -{1}

case-II: if y = 1, then equation becomes


2x = x = 0 which is possible as x is real.

Range[¾ , ¾]
X +J
Example#18: Find the range of y = -=----. ·t1 x Is rea.I
2
2x +3x+9
x+3
Solution: y= --=----
2x2 +3x+9.;
2yx2 + (3y - 1)x + 3(3y - 1) = 0 ...... (i)
Case-I : if y 0, then equation (i) is quadratic in x
xis real

D <!: 0
(3y - 1)2 - 24y (3y - 1) 0
(3y-1)(21y+1)SO

ye[-;1' i] -{O}

Case-II : if y = 0, then equation becomes


- - - x = -3 which is possible as x is real

Range y e [- ;1' "j]

Aesonance®
Educating for better tomorrow
Corporate Office: CG Tower, A-46 & 52, IPIA, Near City Mall. Jhalawar Road, Kola (Raj.)-324005
Webslle: www.resonance.ac.ln E•mall : contact resonance.ac.ln
Toll Free ; 1800 258 5555 CIN: U80302RJ2007PLC024029
-..L .;.; .~-
- Equa ions . ~-~ /--- ---- ---- ---1 --
Self practice problems :
(1) If c > o and ax• + 2bx + 3c = O does not have any real roots then prove that
(I) . 4a - 4b + 3c > O (II) a + 6b + 27c > O (Iii) a + 2b + 6c > 0
_J,
2
(2) If f(x) = (x - a) (x - b), then show that f(x) - (a -b) •
4
(3) Find the least Integral value of 'k' for which the quadratic polynomial
(k - 1) x2 + ax + k + 5 > o v x e R.

2
(4) Find the range of the expression x + 34x- 71 , if xis a real.
x 2 + 2x - 7
2
(5) Find the interval in which 'm' lies so that the expression mx + 3x - 4 can take
all real values, x e R.
· · · -4x 2 + 3x+m
(6) Find the value of b for which difference between maximum and minimum value 2
of x - 2bx - 1 in (0, 1]
is 1.

(7) Find all numbers a for each of which the least value of the quadratic trinomial 4x2 2
- 4ax + a - 2a + 2 on
the interva' O s xs 2 Is equal to 3.
Answers :
(3) k=4 (4) (- 00, 5] u (9, 00) (5) m e(1,7)
(6) b =0 (7) a= 1- or 5 + Jio

(I) Conditions for both the roots of f (x) = 0 to be greater than a specified number •x; are b' - 4ac O&
f (x0 ) > 0 & (-b/2a) > x0 •
(II) Conditions for both the roots off (x) = 0 to be smaller than a specified number
'x0 ' are b 2 - 4ac <!: O &
f(x0 ) > 0 & (-b/2a) < X0 .

(Ill) Conditions for a number 'x0 ' to.lie between the roots off (x) = 0 is f (x ) < O.
0

(Iv) (v)

(Iv) Conditions that both roots of f (x) = 0 to be confined between the numbers
x, and x 2 , (x, < x 2) are
b'- 4ac <!: 0 & f (x,) > 0 & f(x,) > 0 & x, < (-b/2a) < x •
2

(v) Conditions for exactly one root of f(x) = 0 to lie In the interval (x,, x 2) i.e.
x, < x < x 2 is f(x,). f(x 2) < 0.

Reson ance®
Educ,dng-for ~., tomorrow
Corporate Office: CG Towe,, A~6 & 52, IPIA, Noa, City Mall, Jhalawar Road. Kola
W ebsite : www.resonance.ac.ln E-mail : contact resonance.ac.ln
(R.,j.)-324005 ,

Toll Free : 1800 258 5555 CIN: U80302RJ2007PLC024029 PRERMO. 27


_E_q_u_a_ti_o_n_s_ _ _ _ _ .J/r---- --------- --------- -7•
Example#19:
Let x' - (m - 3) x + m = 0 (m e R) be a quadratic equation, then find the values of 'm' for which
(a) both the roots are greater than 2.
(b) both roots are positive.
(c) one root is positive and other is negative.
{d) One root is greater than 2 and other smaller than 1
(e) Roots are equal in magnitude and opposite in sign.
(f) both roots lie in the interval (1, 2)
Solution:

(a)
~•l1 1 ,
h-V-.
Condition - I : D 0 (m-3)2-4m~0
(m - 1) (m - 9) 0
m e (- co, 1] v (9, co) .••... (i)
Condition - II : f(2) > 0 4 - (m - 3)2 + m > 0 m < 10 ...... (ii)
b m-3
Condition - m: - - >2 . --> 2 m>7 ......(iii)
2a 2
Intersection of (i). (ii) and (iii) gives m e (9, 10)

(b)

Condition - I D~O m E (-co, 1)v[9,co)


Condition - II f(0) > 0 m>0

Condition - III _ _.!:_>O m-3 >0 m>3


2a 2
intersection gves m e (9, co) Ans.

(c)
iv.
Condition - I f(0) < 0 m <0 Ans.

(d) ~•l1 \ J

Condition - I f(1) < 0 4<0


Condition - II f(2) < 0 m> 10
Intersection gives m E Ans.
(e) sum of roots = 0 m=3
and f(0) < 0 m <0 Ana.

(f)

Condition -I D 0 m e (- co, 1] v (9, co)


Condition - II f(1) > O 1 - (m - 3) + m > O 4 > 0 which Is In.le vm eR
Condition - m f(2) > 0 m< 10

Corporate Olfiee: CG Tower, A-46 & 52. IPIA. Near City MIi~ J h ~ ROlld. Kota (Raj.}-324005
Resonance®
Educating for better tomorrow
Website : www.resonance.ac.l11 E-mail: contact •onance.ac.111
Toll Free : 1800 258 5555 CIN: U80302RJ2007PLC02~29
_________ /~------------------------im-
Equations __,

m -3
Condition - IV 1 < _..!:_ < 2 1<--<2 5< m < 7
2a 2
intersection gives m e Ans.
Example#20:

Find all the values of 'a' for which both the roots of the equation (a - 2)X2 - 2ax + a = 0 lloo
In the interval (- 2, 1),
Solution :
Case-I : f(-2) > 0 4(a - 2) + 4a + a> 0
9a-8 > 0 8
a> -
9
f(1)>0 a-2- 2a +a > 0
- 2 > 0 not possible a e~
Case-II: a-2<0 · a< 2

f(-2) < 0 8
a< -
9
f(1) < 0 aeR
- 2 <..!:_ - < 1
2a
a< i3
D~ O a~0

intersection gives a e [ 0, ¾) .• [ 98)


complete solution a e 0, V {2}

Self practice problems :


(1) Let x
2
- 2(a - 1)x + a - 1 = 0 (a e R) be a quadratic equation, then find the·value of 'a' for which
(a) Both the roots are positive (b) Both the roots are negative
(c) Both the roots are opposite in sign. (d) . Both the roots are greater than 1.
(e) Both the roots are smaller than 1.
(f) ,· One root is small than 1 and the other root is.greater than 1.
(2) Find the values of p for which both the roots of the equation 4x2 - 20px + (25p2 + 15p - 66) = 0 are less
than 2. ·

(3) Find the values of 'a.' for which 6 lies between the roots of the equation x2 + 2(a. - 3)x + 9 = 0.
2
(4) Let x - 2(a - 1)x + a - 1 = 0 (a e R) be a quadratic equa·tion, then find the values of 'a' for
which
(I) Exactly one root lies in (0, 1).
(ii) Both roots lies in (0, 1).
(Iii) Atleast one root lies in (0, 1 ).
(Iv) One root is greater than 1 and other root is smaller than 0.
(5) Find the values of a, for which the quadratic expression ax2 + (a - 2) x - 2 is negative for exactly two
integral values of x.
Answers :
(1) (a) [2, «>) (b) (c) (- «>, 1) . (d) (e) (- «>, 1]
(f) (2. «>)
{2) (-«>,-1) (3) (-oo,-¾)
(4) (I) (- «>, 1) V (2, oo) (II) (Ill) • ( - «>, 1) v (2, «>) (Iv)
(5) [1 , 2)

Resonance®
EducatJng better
for lomorrow
Corporate Otoce: CG Tower, A-46 & 52, IPIA, Near City Mall, Jholowar Road, Kolo (Rnj.)-324005
Website : www.rasonance.ac.ln E-mail : contact resonnnce.ac.ln
Toll Free : 1800 258 5555 CIN: U80302RJ2007PLC024029
_ ....._ _______
Equations ___,/ , - - - - - - - - - - - - - - - - - - - - - - - - ~ •

12. Solving Equations :


(I) Type - I Relation ship between root and coefficient :
(II) Type-II Solve for x, y, z
• (Iii) Type-Ill Miscellaneous

Example# 21 :
Find all real numbers 'r' which there is atleast one triplet (x, y,z) of nonzero real numbers such that
2 2
x y + y2z + z x = xy2 + yz2 + zx2 = rxyz
Ans. re(~. -1] v [3, co)
Sol. Divide by xyz as x,y,z are non zero

~+ r+ ~= r+~+ =r
Z X y Z X y

now assume = a, f =band =c


y Z X

1 1 1
a + b + c= r , abc = 1 and - + - + - = r
a b c
ab + be + ca = r ........... (ii)
now we can write that a, b,c are the roots of the cubic polynomial t3 - rt2 + rt - 1 = 0
(t3 -1) - rt (t - 1) = O 2
(t - 1) (1 + t + 1 - rt) = O (t - 1) (t2 - (r - 1)t + 1) =0
all solutions should be real so discriminant of quadratic should be ;:: 0
(r - 3)(r+1);::0
r e (~. - 1] v [ 3,co) Ans.

Example# 22 :
Find all polynomials whose coefficients are equal to 1 or - 1 and whose all roots are real.
Ans. ±(x + 1) , ± (x - 1)2 ·
2
± (x + x - 1) , ± (x - x - 1)
3 2
± (x + x - x - 1)
3 2
± (x x - x + 1)
3 2
± (x + x - x + 1)
1
Sol. Let the polynomial is p(x) = a,,x" + a,;:_, x,,_ ........ + a1x + ao
let the roots are a, , a2 ........ a,,
2
l:a, = -(a•-1), l:a,~2 = aa-2 l:a,2 = (a••, ) _ 2 (a""2)= 1 _ 2 (a...2)
an an an an an

L.H.S. is always positive so a,...2 has to be - 1 =3


a.

ByA.M ;:: G.M


2 2 2
a 1 + a 2 .....a . ;:: (a,2 o} ..... a/)1/n
n
Case-I n = 1
or p(x) = ±(x+1), ± (x -1)
Case-II
n 2 = 2
= 2
p(x) ax +bx+ c
D = b - 4ac 0 1-4 ac~ 0 ac < 0
2
p(x) = ± (x + x- 1) or p(x)=±(x2-x-1)

Aesonance®
Educating for better tomorrow
Corporate Office: CG Tower. A-46 & 52. IPIA. Near City Mall. Jhalawar Road, Kota (Raj.)-32400S
Website : www.resonanca.ac.ln E-mail : contact resonance.ac.in
Toll Free : 1800 258 5555 CIN: U80302RJ2007PLC024029
_E_,qLu
........a;.;. t1c::·oc.:n::.s_ _ _ _ __,/'_ _ _ _ _ _ _ _ _ _ _ _ _ _ _ _ _ _
___ _ _ __ _

Case-Ill 3
+ bx2 + ex+ d = o·
n = 3j3 + y)2
(a+ ra 2=+x2Eaj3 = (-b)2
let =P{x)
1 - 2 rap = ra2 1 -2raP = 3 rap = -1
ap + Pr + ya = -1 so c = - 1
Now by choosing b, d from {-1, 1) we can conclude only two cubic polynomials with
their negative sign
have real roots
P(x) = ± (x 3 + x2 - x - 1) or P(x) = ± (x3 - x2 - x + 1) 3 2 3
P(x) = ± (X + x - x + 1)

Example# 23 :
If a is a real root of the equation x• - xl + x - 2 = o, prove that (a' ) = 3. (For any
real number a, we
denote by [a) the greatest integer not exceeding a.)
Sol. Suppose a is a real root of the given equation. Then
a• - al + a - 2 = 0.
This gives a•- al+ a ·_ 1 = 1 and hence (a- 1) (a•+ al +1) = 1. Observe that a•+ 2
al +1 2a + al=
a 2(a+2). If -1 ~a< O, then a+ 2 > o, g·iving a 2(a+ 2) > o and hence {a - 1) a• +
al + 1 > 0. This again
gives {a-1) (a•+ al+1) < O.·
The above reasoning shows that for a< O, we have a•- al+ a - 1 = 1 < 0 and hence
cannot be equal
to 1. We condude that a real root a of x• - xl + x - 2 = O is positive (obviously a~
0).
Now using a•- al+ a- 2 = 0, we get
a•= a•-a2 + 2a
Tl')e statement [a') = 3 is equivalent to 3 s a• < 4 .
Consider a'- a 2+ 2a < 4. Since a> 0, this is equivalent t6 a• - al+ 2a2 < 4a. Using
the relation (1 ), we
can write 2a2 - a+ 2 < 4a or 2a2- Sci+ 2 < 0. Treating this as a quadratic, we get
this is equivalent to
<a< 2. Now observe tha'tif a~ 2 then 1 = (a- 1)(a' + al+ 1) 25 which is the impossible
.
If O i, < a!. i,
then 1 = (a - 1)(a'+ a 3 + 1) < 0 which again is impossible. We conclude that <a<
2. J
Similarly a' - a 2+2a 3 is equivalent to a•- al + 2a2 - 3a 0 which is equivalent
to 2a2- 4a + 2 0.
But this is 2(a - 1)2 Owhich is valid. Hence 3 s ci1 < 4 and we get [a•) = 3.

Example# 24 :
Find the number of ordered pair of real numbers (x, y) satisfying the equation

5x (1+-
1
Xz + y2
-J 1
= 12 & Sy ( 1 - -- ) = 4.
X 2 + y2 .
Solution:
2 12 2 42
Now (5x) + (Sy)2 = 2 + ( )2
1
( 1+ -x-2 -: -y-2) 1- -x-2_+_y_z

2 1
putx2 + y = -
I
25 = ~ + ~
t (1+t) 2 (1-t) 2
2s1• -160t3 + 20612 -1601 + 2s = o
2
2s (1 + ~ ) -160 (t+T). 206 = a
1

t+ ! =u 25u2 - 160u + 156 = 0


t

Aesonance®
Educating for btlttr tomorrow
COfl)Ofale Office: CG Tower, A-46 & 52, IPIA. Near City Mell, Jhalawar Road, Kola
Website : www.resonance.ac.ln E•mall : contact resonance.ac.ln
(RaJ.)-32400 5

Toll Free : 1800 258 5555 CIN: U80302RJ2007PLC02402 9


_E_q_u_
a_ti_on_s_ _ _ _ _~ / , - - - - - - - - - - - - - - - - - - - - - - - - - , •

6 26
u=- -
5' 5

t+ ! = and t + ! =
26
t 5 t 5
1
t =5 -
' 5

If x +
2
y2 = i (x, y) =( f-i)
2
and x + y2 = 5 (x, y) = (2, 1)

Example# 25 :
Find the number of triplets (x, y, z) of integers satisfying the equations x + y = 1 - z and x 3 + y3 = 1 - z2
(where z 'I- 1)
Solution:
x3 + y3 + (1- (x + r)J2 = 1
2
(x + y)(x - xy + y + x + y - 2] = O
Ir x + y = 0 then z = 1 so rejected
(x, y, z) = (m, - m, 1)
and x + y 'I- 0
2
x - xy + y2 + x + y - 2 = 0
(2x - y + 1)2 + 3(y + 1)2 = 12
2x - y + 1 = 0, y + 1 = ± 2
=
or 2x - y + 1 ± 3, y + 1 ± 1 =
(x, y, z) = (0, 1, 0), (-2, -3, 6), (1, 0, 0), (0, -2, 3), (-2, 0, 3), (-3, -2, 6)

Example : 26
Let (x + y - z) (x - y + z) = ayz
(y + z - x) (y -z + x) = bzx
(z + x - y) (z - x + y) = cxy.
If abc = (q - a - b - c)' (where q, r e N) find the value of (q + r)
Solution :
2 2 2
Multiply (x + y - z) (x - y + z) (y + z - x) = abc x 2y2z 2
2 2 2 2
(-I:x3 + y2z + yz + z2x + zx + x y + xy2- 2xyz) = abc x 2y2z2.

abc = ( - ~ -
yz
·rZX_ -~+
xy
r(~+1) -2)
y X
............ (1)

Equation (1)
2 2
a= x - y2 - z + 2yz = _ 'j_ z +2
yz yz z y
x2 z
a-2=-
yz
- :t.
z
--y
y2 z
b-2=---
xz X
--zX
z2
c-2= -
xy
--Xy -1
X

abc = {(2-a) + (2-b) +(2-c) - 2)2


abc = (4 - a -b- c)2.

Resonance®
Educating for better tomorrow
Corporate Offlce: CG Tower, A-46 & 52, IPIA, Near City Mall, Jhalawar Road, Kota (RaJ.)-324005
Website : www.resonanco.ac.ln E-mail : contact rosonance.ac.ln
Toll Free : 1800 258 5555 CIN: U80302RJ2007PLC024029
_E_.q_u_a_ti_on_s_____ ~/r---------------------------1•
Example# 27 :
Let P{x) =:= 0 be a fifth degree polynomial equation with Integer coefficients that has atleast one Integral
root. If P{2) = 13 and P(10) = 5, then find the Integral value of 'x' that must satisfy P{x) = 0.
Sol. Let r be an Integer such that p(r) = 0
P(x) = (x - 2) q(x) + P(2)
P(r) = (r- 2) q(r) + 13
q(r) = - 13
r-2
Now, • r - 2 = ± 1, ± 13
r = 3, 1, 15, - 11
P(x) = {x - 10) q(x) + P(10)

P(r) = 0 q(r) =
r-10
r-10=1,-1,5,-5
r=11,9,15,5
Thus r = 15.

Example# 28 :

Find the number of ordered pairs of natural numbers (x, y) satisfying the equation
2
(xy - 1) = (x + 1)2 + (y + 1)2
Solution :
(xy - 1)2 - (x + 1)2 + (y + 1)2
(xy - x - 2) (xy + x) = (y + 1)2
(y + 1) [x(xy - x - 2) - (y + 1)) = O
If y = -:- 1 then x e R
Similarly x = - 1 then y e R (x, y) = (-1, y), (x, L.1)
Case-1 x .. - -1 y" .. -1
x(xy - x - 2) (y + 1) = (y + 1)2
x(xy - x - 2) = y + 1
2 2
x y - x - 2x - y - 1 = 0
y (x - 1)(x + 1) = (x + 1)2
Since x .. -1
x+1 2
y= - - = 1+ - -
x-1 x-1
Now x - 1 = - 1, 2, -2, 1
X = 0, 3, -1, 2 (X .. - 1)
(x. y) = (3, 2) (2, 3), (0, -1)

Resonance®
EducaUng btlttr tomorrow
for
Corporate Office: CG Tower, A-46 & 52, IPlA, Near City Mall, Jhatawar Road, Kota (Raj.)-324005
Website : www.rasonanca.ac.ln E-mail : contact resonanc a.ac.ln
Toll Free : 1800 258 5555 CIN: U80302RJ2007PLC024029
Equations / , - - - - - - - - - - - - - - - - - - - - - - - - - - - - - - - •

• Exercise-1
PART - I : PRE RMO
1. If the two equations ex+ d = O and x 2 - ax+ b = O have one common root and the second equation
x2 -
has equal roots, then 2 (b + d) =
2. If x 2 + 3x + 5 = 0 and ax2 + bx + c = O have a common root and a, b, c e N, then the minimum value of
(a+ b + c) is

3. x2 + x + 1 is a factor of ax3 + bx2 +ex+ d = O, then the real root of above equation is (a, b, c, d e R)

4. If roots of equation 2x' - 3x3 + 2x2 - 7x - 1 = Oare a, p, y and 6 then value of La+a 1 is equal to
5. If two roots of the equation X3 - px2 + qx - r = Oare equal in magnitude but opi,.:isite in sign, then:
6. If a, p, y are the roots of the equation xJ + px2 + qx + r = " then the value of

(a -p~)(P -Y~)(r -:p) is

7. If the inequality (m - 2)x2 + 8x + m + 4 > o is satisfied for all x e R, then the least integral value of m is:
8. The real values of 'a' for which the quadratic equation 2x2 - (a3 + 8a - 1) x + a2 - 4a = 0 possess roots
of opposite sign is given by:

9. If a, pare the roots of the quadratic equation x2 - 2p (x - 4) - 15 = 0, then the set of values of p for
which one root is less than 1 & the other root is greater than 2 is:

10. If a, p are the roots of quadratic equation x 2 + p x + q = 0 and y, 6 are the roots of x2 + px - r = O,
then (a - y) . (a - 6) is equal to :
11. Each root of the equation ax2 + bx + c = 0 is decreased by 1. The quadratic equation with these roots is
x2 + 4x + 1 = 0. The numerical value of b + c is _ _ __

12. .!.
x 'y' z are distinct real numbers such that x + y = y + .!.Z = z + .!.X . The value of x y 2.' ::: _
2 2
_ _,

13. A hare sees a hound 100 m away from her and runs off in the opposite direction at a speed of 12 KM
an hour. A minute later the hound perceives her and gives a chase at a speed of 16 KM an hour. The
distance at which the hound catches the hare (in meters) is _ _ __

14. a and b are positive integers such that a2 + 2b = b2 + 2a + 5. The value of b is _ _ _ _.

15. a~ o, b o The number of real number pair (a, b) which satisfy the equation a•+ b' =(a ... b)' is
16, The value of + is= _ _ .

17. If a, b, c, d satisfy the equation a+ 7b + 3c + 5d = 0, Ba + 4b + 6c + 2d = - 16, 2a + 6b + 4c + Bd =16


Sa+ 3b + 7c + d =-16then the value of (a+ d) (b + c) = _ _ .

18. The combined age of a man and his wife is six times the combined ages of their children. TINO years
ago their united ages were ten limes the combined ages of their children. Six years hence their
combined age will be three times the combined age of the children. The number of children they have is

19. If (x + 1)2 = x, the value of 11x3 + Bx2 + Bx- 2 is

Corporate ornce: CG Tower, Ar46 & 52, IPIA, Near City Mall, Jhalawar Road, Kota (Raj.}-324005
Ill EducaUng
a, Aesonance4)
for belier lomorrow
Website : www.resonance.ac.ln E-mail : contact resonance.ac.ln
Toll Free : 1800 258 5555 CtN: U80302RJ2007PLC024029
Equations / , - - - - - - - - - - - - - - - - - - - - - - - - - - - - - - •

a4 b4 c4 .
20. If a= 2012. b = -1005, c = -1007, then the value of - - + - - + - - + 3abc 1s
b+c c+a a+b

21. If one root of x + ,,/b + x = is 2012, then a possible value of (a, b) is

22. 1
a and b are the roots of the quadratic equation x2 + NC - -- = O where x is the unknown and,. is a real
2')..2
parameter. The minimum value of a•+ b• is:
23. The remainder when the polynomial x + x, + x•+ x27 + x•' +x mis divided by x2 -1
24. If x, Y are positive real numbers satisfying the system of equations x 2 + y = 336, Y2 + x = 112.
y
then x + equals
25. If a, b, care positive integers ~uch that a2 + 2b2-2ab = 169 and 2bc- c2= 169 then a+ b + c is:
26. P = 20082007 - 2008; Q = 20082 + 2009. The remainder when P is divided by a is
27. · The number of integer values of a for which x2 + 3ax + 2009 = 0 has two integer roots is :
28. The sum of the fourth powers of the roots of the equation x3 - x 2 - 2x + 2 = 0 is

29. 1 1 1
If a, b, c are real ; a ;e 0, b .. 0, c,. O and a + b + c ;t O and - + - + - = - - -
a b c a+b+c
then (a+ b) (b + c) (c + a)=

30. The roots of the equation x 5 - 40x• + Px3 + Qx2 + Rx + S = 0 are in geometric progression. The sum of
their reciprocals is 10. Then ISi is equal to :
31. The number of solutions (x, y) where x and y are integers, satisfying 2x2 + 3y2 + 2x + 3y = 10 is :

p q r . a b c P2 q2 r2 .
32. If -+-+-= 1 and -+-+- = Othen the value of - + - + - 1s :
a b c p q r a2 b2 c 2
33. A cubic polynomial Pis such that P(1) = 1, P(2) = 2, P(3) = 3 and P(4) = 5. Then P(6) is

3 · 3 1 1 3 1
34. Which of the following is the best approximation to (2 - 1>(3 - ).....( OOO - )
3 3
· (2 + 1)(3 + 1) ..... (10003 + 1)
3
35. Given that (1-x) (1 +x+x2 + X + x•) = :; and x is a rational number. Then 1 + x + x• + x3 + x4 + xsis :

1
36. Solve the equation 3x• - 1Ox' + 4x2 - x - 6 = 0 one root being +
2
5
37. Find the smallest integral x satisfying the inequality x- > o.
2
X + 5x-14
38. Find integral x's which satisfy the inequality x 4 - 3x3 - x + 3 < o.
39. Find the largest integral x which satisfies the following inequality
2
(x + 1)(x -3) (x - 5)(x - 4)2 (x - 2) < O ·
2 3
40. Given 3x + x = 1, find the value of 6x - x 2 - 3x + 2010.

If _!_ _ _!_=4, find the value of


2 x+ 4 xy- 2 y.
41.
X )I X-y-2xy
7 3
42. Let P(x) = ax + bx + ex -5, where a, b, care-constants. Given P(-7) = 7, find the value of P(7).

Corporate Office: CG Tower, A-46 & 52, IPIA, Near City Mnll. JholowM Ro.'X!, K<>1:1 (R:lj,)-~~ t.'I.~
Website : www.resonance.ec.ln E-mail : contact resonanca.nc.in
Toll Free : 1800 258 5555 CIN: U80302RJ2007PLC024029 PRERM0- 35 "
Equations / , - - - - - - - - - - - - - - - - - - - - - - - - - - - - - - - •

43. If xy = a, xz = b, yz = c and abc O, find the value of x 2 + y2 + z2 in terms of a, b, c.


I I I 13
44. Find the number of positive integers x satisfying the equation - + - - + - - =-
x x+1 x+2 12
45. Solve the following equation : (x - 1)(x - 2)(x - 3)(x - 4) = 15
2
46. Solve the following equation : x -
3 .5 x + 1-5 _ o
x 2 -x-6 -

PART - 11 : RMO
2 2
1. Show that the expression (ax - b)(dx-c) will be capable of all values when xis real, if a - b and
(bx-a)(cx-d)
2 2
c - d have the same sign.

2. If by eliminating x between the equations x 2 + ax + b =o and xy + t (x + y) + m =O, a quadratic in y is


formed whose roots are the same as those of the origin.al quadratic in x. then prove that either a = 2l
and b = m or b + m = al.
2
2
3. If the roots of the equation (1-q + P ) x + p(1 + q)x + q(q - 1) + .e.:_ = 0 are equal, then show that
. 2
p2 = 4q
2
4. If each pair of the three equatior,s x - p,x + q, = 0, x 2 - P2C
2
+ qz = 0, x - p3x + q 3 = 0, have common
root, prove that p~ + p~ + p~ + 4(q1 + q 2 + q3 ) = 2(P2PJ + PJP1 + P1P2)

5. Solve the system


x sin a + y sin 2a + z sin 3a = sin 4a
x sin b + y sin 2b + z sin 3b = sin 4b
x sin c + y sin 2c + z sin 3c = sin 4c

6. If a, p, y are the roots of the equation x3 - 2 2


x + 4 = 0, then form an equation where roots are a+ p + y2,
p + a2 + y2, y + a2 + p2.
3
7. If a, p, y are the roots of the equation x + ex + d = 0 then form an equation whose roots are
py + p + y · ay + a + y pa + P+ a
p+y-a' a+y -p ' p+a-y
3 2
8.
4
Determine all real values of the parameter 'a' for which the equation 16x - ax + (2a + 17) x - ax + 16 = o
has exactly four distinct real roots that form a geometric progression
4 3 2
9. Consider the equation x5 + 5AX - x + (A.a- 4)x - (8A. + 3) x + (A.a - 2) = 0 where A., a e R
(i) Determine a such that the given equation has exactly one root independent of A..
(ii) Determine a such that the given equation has exactly two roots independent of A..

10. Find all real numbers 'r' for which there is atleast one triplet (x, y,z) of nonzero real numbers such that
2
x 2y + y2z + z2x =xl
+ yz + zx
2
rxyz =
11. Find all polynomials whose coefficients are equal to 1 or - 1 and whose all roots are real.

12. If a, p, y are the roots of the equation x' + 2x2 + 3x + 1 = 0, then form an equation wh9se roots are
_!_ + _!_ - J..... J... + _!_ - _!_ . J... + _!_ _ _!_ .
f}' y3 al • al y3 p3 • a3 pl y3

13. =
If a is a real root of the equation x 5 - X3 + x - 2 O, then prove that [a•] =3. (For any real number a. we
denote by [a) the greatest integer not exceeding a.)

Corporate Office: CG Tower, A-46 & 52, IPIA. Near City Mall. Jhalawar Road, Kola (Raj.)-324005
111 Aesonance~ Website: www.resonance.ac.in E-mail : contact resonance.ac.in
r&, EducaUng for better tomorrow Toll Free: 1800 258 5555 CIN: U80302RJ2007PLC024029
Equations / ,---- ----- ----- ----- ----- ----- -1•
14. The three equations
x + y +z=3,
x + y3 + z 3 = 15 and .
3
4
x + y4 + z 4 = 35 .
has a real solution x, y, z for which x 2 + y2 + z2 < 10. Find the value of (x5 + y5 + z 5)
15. Determine all real solutions of the given equation where p Is real number ,Jx2 - p + 2 Jx2 -1 = x

16. Find the real numbers (x, y) that satisfy the eq'oation
x/ = 15x22 + 17xy
2
+ 15y 2
x2y = 20x + 3y
17. Find the solution of equations
(3x + y) (x + 3y) Jxy =- 14 and
(x + y) (x2 + 14xy + y2) = 36 where x, ye R
18. Find all solutions (x 1, x 2,x3 .........Xn) of the equation
1 + J_ + x, + 1 + -'-(X 2 +--'-1)
+ ..............
+ (X1 + 1))(X2 + 1) .....(Xn-1+ 1) = O
...........................
X1 X1X2.......1_+--'1)"'-
)(:....X.:..
X1X2 X3 X1X2 ......Xn
19. The roots x 1 , x2, x 3 of the equation x 3 + ax + a = 0, where a is a non-zero real, satisfy
x~ x~ x~ .
- + - + - = -8. Find x,, X2, X3.
X2 X3 X1
". :.. Solve the given equations :
x+y+z = 0
3 3
X + y3 + z = 18
x 7 + y7 + z 7 = 2058 where x,y,z e R

• Exercise-2 ]
PART- I: PREVIOUS ASKED QUESTION FOR PRE RMO
1. Let Sn = n 2 + 20n + 12 where n is a positive integer. What is the sum of all possible values of n for
which Sn is a perfect square? [PRMO-2012]
2. Let x 1, x 2, x 3 be roots of equation x 3 + 3x + 5 = 0. What is the value of the expression

( x, + ;J ( X2 + x:) ( X3 + x:) ? [PRMO-2012]


2
How many integer pairs (x, y) satisfy x + 4y2 - 2xy- 2x - 4y - 8 = 0 ?
3. [PRMO-2012]
4. II is given that· the equation x 2 + ax + 20 = 0 has integer roots. What is the sum of all possible
values of a? [PRM0-2013]
5. Three real numbers x, y, z are such that x 2 + 6y = - 17, y2 + 4z = 1 and z 2 + 2x = 2. What is the value of
x 2 + y2 + z
2
[PRMO-2013]
6. Let f(x) = x 3 - 3x + b and g(x) = x 2 + bx - 3 where b is a real number. What is the sum of all possible
values of b for which the equations f(x)= 0 and g(x) = 0 have a common root ? [PRMO-2013]
7. What is the smallest possible natural number 'n' for which the equation x 2 - nx + 2014 = 0 has integer
roots. [PRMO-2014]
8. Natural numbers k, l, p and q are such that if a and b are roots of x 2 - kx + t = 0 then a + and b +
b a
are roots of x 2 - px + q = 0. What is the sum of all possible values of q? [PRMO-2014]
9. Let x,, x2, X3, ....... X201• be real numbers different from 1 such that x, + x + .. ... + x
2 2014 = 1 and
• 2 2 2 '
...!L +___:_L_+ ........ =I. What Is the value of ---1_+ ___:_L_+ ........ [PRMO-2014]
1-x1 1-x2 1-x201 4 1-x1 l- x 2 1-x2014
Corpbrale Otnce: CG Tower, A-46 & 52, IPIA, Near City Mall. Jhalawar Road, Kola (Ra).)-324005
Website : www.resonance.ac,ln E-mail : contact resonance.ac.ln
Toll Free : 1800 258 5555 CIN: U80302RJ2007PLC024029
Equations / , - - - - - - - - - - - - - - - - - - - - - - - - - - - - - - - , •

10. If real numbers a, b, c, d, e satisfy a + 1 = b + 2 = c + 3 = d + 4 = e + 5 = a + b + c + d + e + 3 then find


the value of a2 + b2 + c2 + d2 + e2 [PRMO-2014)
11 .
2
=
The equations x - 4x + k 0 and x2 + kx - 4 =O where k is a real number have exactly one common
root. What is the value of k. [PRMO-2015]
12. Let a, b and c be real numbers such that a - 7b + 8c =4 and 8a + 4b - c =7. What is the value of
a2 - b 2 + c2• [PRMO-2015]
2 2
13. Let a, b and c be such that a + b + c =O and p = _a_
2
_ + _b_z_ + c is defined. What is
2a +be 2b2 +ca 2c 2 +ab
the value of P . [PRMO-2015]
2 6
14. Suppose x - x + 1 is factor of 2x - x 5 + ax• + x 3 + bx 2 - 4x - 3. Find a - 4b [PRMO-2015]
33 22 11 3 2
15. Let R(x) be the remainder upon dividing x ' +x + x +x + 1 by the polynomial x' + x + x + x + 1.
Find R(1) + 2R (2) + 3R(3). [PRMO-2015]
16. Let P(x) = (x - 3)(x - 4)(x - 5). For how many polynomials Q(x), does there exist a polynomial R(x) of
degree ,:, such that P(Q(x)) = P(x)R(x)? [PRMO-2015]
17. For how many pairs of odd positive integers (a, b), both a, b less than 100, does the equation
2
x +ax+ b = O have integer roots? [PRMO-2015]
2
18. Find the sum of all those integers n for which n +20n+15 is the square of an integer. [PRM0-2015)
2 2
19. Let a and p be the roots of equation x + x - 3 = 0. Find the value of the expression 4p - a 3•
- [PRMO-2016]
20. 3
= 3
=
Let x + ax + 10 0 and x + bx2 + 50 0 have two roots in common. Let P be the product of these
3
common roots. Find the numerical value of P , not involving a, b. [PRMO-2016]
21. For real numbers x and y, let M be the maximum value of expression [PRMO-2016)
3 2
x'y + x y + x y + x y + xy2 + xy3 + xy' , subject to x + y = 3. Find [M] where[.]= G.I.F.
22. Between 5pm and 6pm, I looked at my watch mistaking the hour hand for the minute hand and the
minute hand for the hour hand, I mistook the time to be 57 minutes earlier than the actual time. Find
the number of minutes past 5_when I looked at my watch. [PRMO-2016 (Delhi)]

23. Suppose a, b are positive real numbers such that ala+ = 183 , + = 182. Find ~(a+ b).
. . 5
[PRMO-2017]
2 2
24. Let a, b be integers such that all the roots of the equation (x + ax + 20)(x + 17x + b) = 0 are negative
integers. What is the smallest possible value of a+ b? [PRMO-2017]

25. In a class, the total numbers of boys and girls are in the ratio 4 : 3. On one day it was found that 8 boys
and 14 girls were absent from the class and that the number of boys was the square of the number of
girls. Whal is the total number of students in the class? [PRMO-2017]
2 2 2
26. If the real numbers x, y, z are such that x + 4y + 16z = 48 and xy + 4yz + 2zx = 24. what is the value
2
of x + y2 + z2? [PRMO-2017)
2
27. Suppose 1,2,3 are the roots of the equation x' + ax + bx = c. Find the value of c. [PRMO-2017)
2
28. Determine the s~m of all possible positive integers n, the product of whose digits equals n - 15n - 27.
[PRMO-2017)
2
29. Suppose a b are integers and a + b is a root of x + ax + b = 0. What is the maximum possible
2
values of b ? [PRMO-2017]
2 2 2
30. Integers a b, c satisfy a+ b - c = 1 and a + b - c = -1 . What is the sum of all possible values of
2 2 2
a +b +c ? [PRMO-2018]

Corporate Office: CG Tower. A_.6 & 52. IPIA, Near City Mall, Jhalawar Road. Kota (Raj.}-324005
Ill
-
~esonance®
Educ.Ung for bttltr tomorrow
Website : www.resonance.ac.ln E-mail : contact resonance.ac.ln
Toll Free : 1800 258 5555 CIN: U80302RJ2007PLC024029
Equanons / , - - - , - - - - - - - - - - - - - - - - - - - - - - - - - - - - •

PART - II : PREVIOUSLY ASKED QUESTION OF RMO


1. Find three distinct positive integers with the least possible sum such that the sum of the reciprocals
of
any two integers among them is an Integral multiple of the reciprocal of the third integer. [RMO-2010
]
2. For any natural number n, expressed In base 10, let S(n) denote the sum of all digits of n.
Find all
natural numbers n such that n3 = 8S(n)3 + 6nS(n) + 1. [RMO-2010]
3. Let f(x) = x3 + ax2+ bx + c and g(x) = x3 + bx2+ ex + a, where a, b, c are integers with c 6= 0. Suppose
that the following conditions hold: [RMO-2013)
(a) f(1)=0;
(b) the roots of ~(x) = 0 are the squares of the roots of f(x) = 0.
Find the value of a20 3 + b2013 + c2013.

4. Suppose that m and n are integers such that both the quadratic equations x2 + mx - n =
2 0 and
x -, mx + n = 0 have integer roots. Prove that n is divisible by 6. [RMO-2013]
5. Find all positive real numbers i<, y, z such that [RMO-2014)
1 1 1 1 1 1
2x - 2y + - = - - , 2y-2z+ -=-- 2z-2x +-= - ·- .
Z 2014 X 2014 y 2014
6. Let P(x) = x2 + ax + b be a quadratic polynomial with real coefficients. Suppose there are real numbers
s.. t such that P(s) = t and P(t) = s. Prove that b - st is a root of the equation x2 + ax + b - st= 0.
[RMO-2015]
7. Find all integers a,b,c such that a 2 = be+ 1, b2 = ca+ 1. [RMO-2015)

8. Let P(x) = x2 + .!. x ;


b and Q(x) = x2 + ex + d be two polynomials with real coefficients such that
2
P(x) Q(x) = Q(P(x)) for all real x. Find all the real roots of P(Q(x)) = 0. [RMO-2017]

Ii Answ ers ]
• Exercise-1 l :
PART - I : PRE RMO
1. ac 2: 9 3. (a - b)la 4. -3 5. pq = r
3
(r + 1)
6. - -r-2 7. 5 8. 0<a<4 9. (-oo,i)
10. - (q + r) 11. 0 12. 13. 1200 meter. 14. 3
15. no real value 16. 17. -16 18. 3 19.
20. 0 21. (4024, 2012) 22. 23. 6x 24. 20
25. 39 26. 4032066 27. 6 28. 9 29. 0

30. 32 31. 4 32. 333


33 16 34.
500
63 2
35. 36. 3 - - - 37. -6 38. 2 39. -2
32 ' 3' 2

40. 2009 41. 2/3 42. -17

11 Resonance~
Educ1llng lor bttlor tomorrow
Corporate Office: CG Tower, & 52, IPIA, Near City Mall, J halawar Road, Kola (Raj.)-324005
Website : www.resonance.ac.ln E-mail : contact resonance.ac.l n
Toll Free : 1800 258 5555 CIN: U80302RJ2007PLC024029
Equations /r--------------------------•

43.· 44.

45.
5±../21
46. 1/2
2

PART - 11 : RMO
5. x = 2 (cos a + cos b + cos c) + 8(cos a cos b cos c)
Y = - 2 - 4(cos a cos b + cos c + cos b cos c)
z = 2(cos a + cos b + cos c)
6. 7. 8dx3 - 2 2
4x (c + 3d) + 2x (3d + 2c2 - 2cd)- (d + c 2 + d 2 - 2cd) = 0

8. 170 64
9. (I) a=- - (ii) a=-3
5
10. re(~. -1) u [3,'oo)

11. ±(x + 1) , ± (x - 1)
2
± (x + x - 1) , ± (x 2 - x - 1)
3 2
± (x + x - x - 1)
3
± (x - x2 - x + 1)
± (x3 + x 2 - x + 1)

12. z 3 + 12z2 - 172z - 2152 = 0 14.

16. (X, y) = (19, 45) & (0, 0) 17.

18. x1 = -1 for atleast one i is the solution where i e {1,2,3, .....n}

19. {x1.~-.x3} = {-2, 1 - Js, 1 + Js} all its permutation


20. (X, y, Z) = (2, -1, - 3) , (2, - 3, -1} (1-3,-2) (1,-2,-3), (3,-1,-2) , (3,-2,-1)

a Exercise-2 ]
PART - I : PREVIOUS ASKED QUESTION FOR PRE RMO
- 29
1. 16 2. 3. 6 4. 0 5. 14
5
6. 0 7. 91 8. 4 9. 0 10, 10

11. 3 12. 13. 14. 6 15. 0

16. 22 17. 0 18. -40 19. 19 20. 500

21. 36 22. 12 min. 23. 73 24. 25 25. 42

26. 21 27. 36 28. 17 29. 81 30. 18

PART- II : PREVIOUSLY ASKED QUESTION OF RMO


1. 2, 3, 6 2. 17 3. - 1 5. X =y =Z = 2014
7. (1,1,0) (-1, -1, 0}, (-1, 1, 0), (1 , -1, 0), (-1, 0, +1), (+1, 0, -1), (0, 1, -1), (0, -1, 1)

8. ..!2' -1
Corporale ornce: CG Tower, A-46 & 52, IPIA, Near City Mall, Jhalawar Road, Kola (Raj.)-324005
Ill Aesonanceil)
rA, Educating for !Miter lomonow Website : www.resonance.ac.ln E-mail : contact resonance.ac.ln
Toll Free : 1600 256 5555 CIN: UII0302RJ2007PLC024029

Sequence and Series


w
w
w
.b
o ok
co
rn
er
of
fic
ia
lst
or
e.b
lo
gs
po
t.c
om
Sequence & Series / , - - - - - - - - - - - - - - - - - - - - - - - - - - - - , •

I Sequence & Series I


"1729 is a very Interesting number; it is the smallest number expressible as the sum of two cubes In two d~lerent ways." ...... S.Ramanujan

Sequence:
A sequence is a function whose domain is the· set N of ·natural numbers. Since the domain for
every sequence is the set N of natural numbers, therefore a sequence is represented by its
range. If f : N R, then f(n) = t., n e N is called a sequence and is denoted by
t,. t,....................} = {t.J
{f(1 ), f(2), f(3), ...............} = {t,,
Real sequence :
A sequence whose range is a subset of R is called a real sequence.
e.g. (i) 2, 5, 8, 11 , ........................
(ii) 4, 1, - 2, - 5, .......................
Types of sequence :
On the ba:;is of the number of tenns there are two types of sequence.
(i) Finite sequences : A sequence is said to be finite if it has finite number of tenns.
(ii) Infinite sequences : A sequence is said to be infinite if it has infinitely many tenns.

Example# 1:
(-2)"
Write down the sequence whose nth term is
(-1)" +2
Solution:

Lett = (-2 )"


n (-1)" +2
put n = 1, 2, 3, 4, .............. we get
4 16
t, = -2, t, = 3 '13 = -8, t, =3
. -8 16
so the sequence Is - 2, , ,
3 , .......
Serles :
By adding or subtracting the tenns of a sequence, we get an expression which is called a series.
If a,, a 2, a3 , ........a. is a sequence, then the expression a, + a 2 + a 3 + ...... + a. is a series.
e.g. (i) 1 + 2 + 3 + 4 + ..................... + n
(ii) 2+4+8+ 16+ .................
(iii) - 1 + 3 - 9 + 27 - ......._...
Progression :
The word progression refers to sequence or series - finite or infinite

Arithmetic progression (A.P.) :


A.P. is a sequence whose successive tenns are obtained by adding a fixed number 'd' to the preceding
terms. This fixed number 'd' Is called the common difference. If a is the first tenn & d the common
difference, then A.P. can be written as a, a+ d, a+ 2 d,....... ,a+ (n -1) d, ........
e.g. - 4, - 1, 2, 5 ...........

nesonance® C(l(Jlorate Otnce: CG Tower, A-46 & 52, tPIA, Near Clly Mall, Jhalawar Road, Kola (Rej.}-324005
n
Educating for better tomorrow
Website : www.resonance.ac.ln E-mail : contact resonanca.ac.ln
: ""1e'c'oo~2se=s~ss.;;.;s=c"'1N.!.J:c.::u""eo:.:.:3::.:02""R""J::..:20.:.=o:::::1P""L"'c""o""24""0::..:29=='----
1-T;:.;.o1""1=-=F""ree~
Sequence & Series /,.._ _ _ _ _ _ _ _ _ _ _ _ _ _ _ _ _ _ _ _ _ _ _ _ _
__

n111 term of an A.P.:


Let 'a' be the.first term and 'd' be the common difference of an A.P., then
t. = a + (n - 1) d, where d = t. - 10 _ 1
Example# 2:
Find the number of terms in the sequence 4, 7, 10, 13, ....... ,82.
Solution:
Let a be the first so 82d=be
term and + (ncommon
4 the - 1)3 difference n = 27
a = 4, d = 3
The sum of first n terms of an A.P.:
If a is first term and d is common difference, then sum of the first n terms of AP is
n
s.=
2 [2a+ (n-1)d]
n .
=
2 [a+ l) =1n'(•;1) , for n 1s odd. (Where l is the last term and '(T) is the middle term:)
Note : For any sequence {t.,}, whose sum of first r terms is S,, r"' term, t, = S, - S,_,.
Example# 3:
If in an A.P., 3rd term is 18 and 7 term is 30, then find sum of its first 17 terms
Solution:
Let a be the first term and d be the common difference
a+ 2d = 18
a+ 6d = 30
d=3,a=12
17
s = - [2 >< 12 + 16 >< 3] = 612
17 2 .

Example#4:
Find the sum of all odd numbers between 1 and 1000 which are divisible by 3
Solution:
Odd numbers between 1 and 1000 are
3, 5, 7, 9, 11, 13, ·-·- 993, 995, 997, 999.
Those numbers which are divisible by 3 are
3, 9, 15, 21, ----- 993,999
They form an A.P. of which a= 3, d = 6, l = 999 :. n = 167

S= [a+ t ) = 83667
2
Example# 5:
The ratio between the sum of n term of two A.P.'s is 3n + 8 : 7n + 15. Then find the ratio between their
12th term
Solution :
~ = (n/2)[2a+(n-1)d) = 3n+8 a+{(n-1)/2}d = 3n+8
--or --(i)
S." (n/2)[2a'+(n-1)d1 7n+15 a'+(n-1)/2d' 7n+15
T a+11d
we have to find T::, = a'+ 11d'
= =
choosing (n - 1)/2 11 or n 23 in (1 ),
T, 2 a+11d _ 3(23)+8 77 _ 7
wegetT, 2 '-a'+11d' (23)><7+15 176 16

I ~esonance®
Educating for better tomorrow
Website : www.resonance.ac.ln E-mail : contact resonance.ac.ln
Toll Free : 1800 258 5555 CIN: U80302RJ2007PLC024029
Sequence & Series / , - - - - - - - - - - - - - - - - - - - - - - - - - -

Example#6:
If sum of n terms of a sequence Is given by Sn = 3n2 - 4n, find its 50"' term.
Solution:
Let In is n"' term of the sequence so In= Sn - Sn_,.
= 3n2 - 4n - 3(n - 1 )2 + 4(n - 1) = 6n - 7
so t50 = 293.

Self practice problems :

(1) Which term of the sequence 2005, 2000, 1995,' 1990, 1985, ............. is the first negative term

(2) For an A.P. show that tm + '7n•m = 2 tm•n


(3) Find the maximum sum of the A.P. 40 + 38 + 36 + 34 + 32 + ..............
(4) Find the sum of first 16 terms of an A.P. a,, a2 , a3 ...........
If it is known that a, + a. + a7 + a,0 + a, 3 +a,.= 147
Answers. (3) .420 (4) 392
(1) 403
Remarks:
(i) The first term and common difference can be zero, positive or negative (or any complex number.)
(ii) If a, b, care in A.P. 2 b =a+ c & if a, b, c, dare in.AP. a+ d = b + c.
(iii) Three numbers in A.P. can be taken as a - d, a, a+ d ; four numbers in A.P. can be taken as a - 3d,
a -d, a+ d, a+ 3d; five numbers in A.P. are a -2d, a ...'.'d, a, a+ d, a+ 2d ;
six terms in A.P. are a- 5d, a-3d, a -d, a+ d, a+ 3d, a+ 5d etc.
(iv) The sum of the terms of an A.P. equidist_a nt from the beginning & end is constant and equal to the sum
of first & last terms.
(v) Any term of an A.P. (except the first) is equal to half the sum of terms which are equidistant from it.
an= 1/2 (an-k + an•k), k < n. Fork= 1, an= (1/2)(an_1+ an.1 ); Fork= 2, a~= (1 /2)(an-2 + an.2 ) and so on.
(vi) If each term of an A.P. is increased, decreased, multiplied or divided by the same non-zero number,
then the resulting sequence is also an AP.

(vii) The sum and difference of two AP's is an AP.


Example # 7 : .
The numbers t (12 + 1), 12 and 6 are three consecutive terms of an·A.P. If t be real, then find the the next
two term of A.P.
Solution: -.
2b =a+ c -12 = 13 + t + 6
or P + 12 + I + 6 = 0
or {I + 2) (12 - t + 3) = 0
12 - t + 3 0* t = -2
the given numbers are - 10, - 2, 6, which are in an A.P. with d = 8 . The next two numbers are 14, 22
Example #8:
5
If a,, a2 , a3 , a,. a 5 are in A .P. with common difference * 0 , then find the value of}>,, when a,= 2.
,. 1

Solution :
As a,, a 2 , a 3 , a,, a5 are In A.P., we have a, + a5 =a,+ a.= 2a3 •
6
Hence La,
,_, = 10.

Corporate Office: CG Tower, A-46 & 52, IPIA, Near City Mall, Jhalawar Road, Kola (Raj.)-324005
Website : www.resonance.ac.ln E-mail : conlacl resonance.ac.ln
S1•qm•nc,· (\: St•rit•s / , - - - - - - - - - - - - - - - - - - - - - - - - - - - - -

Example #9:

If a(b + c). b(c+o). c(o + b) are In A.P.• prove that 1. , 2.. 1. are also In A.P.
a b c
Solutlon :
·: a(b + c). b(c+a), c(a + b) are In A.P.
=> subtract ob + be + ca from each
- be, -ca. - ab are In A.P.
divide by --abc
1 1 1
- , -b , - are in A.P.
8 C

Example# 10 :
If a+b ,b, b+c arelnA.P.thenprovethat2. ,b 1
1-ab 1-bc 8
I AP
are n · · c
Solution:
a+b b+c .
- b , b, - - are in A.P.
1-a 1-bc
b- a+b = b+c -b
1-ab 1-bc
2 2
-a(b +1) = c(1+b )

1-ab 1- bc
=> -a + abc = c - abc
a+ c = 2abc
divide by ac
1
2-+1. = 2b - , b,
a
are in A.P.
C a C

ArlthmetJc mean (mean or average) (A.M.) :


If three tenns are in A.P. then the middle term is called the A.M. between the other two, so if a. b,
care in A.P., bis A.M. of a & c.
.. A_ a, +a2 +a, + ....+a.
A.M. for any n numbers a,, a2 , .••, an 1s, - n

n-ArtthmetJc means between two numbers:


If a, bare any two given numbers & a, A,,~ .... , ~. bare in A.P., then A 1, A 2 , ... are the
n A.M.'s between a & b.
A, =a+~.~=a+2(b-a) , ...... ~=a+n(b-a)
n+1 n+1 n+1

Note : Sum of n A.M.'s inserted between a & bis equal ton times the single A.M. between a & b

i.e. f
'• I
A,= nA, where A is the single A.M. between a & b i.e. A= a+b
2

Example # 11 :
If a. b. c,d,e, fare A. M's between 2 and 12, then find a+ b + c + d + e + f.
Solution:
2 12
Sum of A.M.• = 6 single A.M. = 6( + ) = •2
2
Sequence & Series / ,---------~
Example# 12:
Insert 10 A.M. between 3 and 80.
Solution:
H~re 3 is the first term and 80 is the 12"' term of A.P. so 80 =3 + (11 )d d =7
so the series is 3, 10, 17, 24, ........, 73, 80
required means are 10, 17, 24, ........ , 73.
Self practice problems :
(5) There are n A.M.'s between 3 and 29 such that 6th mean : (n - 1)th mean : : 3 : 5 then find the
value of n.
. an+3 + bn+3
(6) For what value of n, - - - - , a~ bis the A.M. of a and b.
an+2 + bn+2
Answers.
(5) n = 12 (6) n =-2
Geometric progression (G.P.) : ..
G.P. is a sequence of numbers whose first term is non zero & each of the succeeding terms is equal to the
preceeding terms multiplied by a constant. Thus in a G.P. the ratio of successive terms is constant. This
constant factor is called the common ratio of the series & is obtained by dividing any term by that which
immediately preceeds it. Therefore a, ar, ar2, ar3, ar4, ........................... is a G.P. with 'a' as the first term & 'r' as
common ratio.
1 1 1 1
e.g. (i) 2, 4, 8, 16, ....... (ii) 3 ' 9. 27. 81' .......
Results:
(i) n th term of GP= a 1"'""1
(ii) Sum of the first n terms of
GP
-1) ·

!
a(r"
_ - - - , r~1
Sn - r-1
na , r =1
(iii) Sum of an infinite terms of GP when Ir I < 1. When n <Xl, r" 0 if Ir I < 1·therefore,

S., =
1
~r (/r/ < 1)
Example# 13:
1
The n" term of the series 3, ,1,- is - - , then find
243 ·
Solution:

3.[
1 )n-l = 1
n = 13
243
Example# 14 :
The first term of an infinite G.P . is 1 and any term is equal to the sum of all the succeeding terms.
Find the series.
Solution:
Let the G.P. be 1, r, r 2, f'J, .........
r2 1
given condition r= r= -
1-r 2·
Hence series is 1, ,i ,4 , 8 , ............... oo

Corporate Office: CG Tower, A--46 & 52, IPIA, Near City Mall, Jhalawar Road. Kola (Raj.}-324005
Resonance®
Educating for better tomorrow
Webs resonance.ac.ln
Toi F
Sequence & Series / , - - - - - - - - - - - - - - - - - - - - - - - - - - - - - i l J
-

Example # 15
its first term :
In a G.P, .• T 2 + T5 = 216 and T. : T, = 1 : 4 and all terms are integers. then find
Solution :
3
ar (1 + 1"3) = 216 and ar 5 = .!
ar 4
::)
when r = 2 then 2a(9) = 216::) a= 12
when r = - 2. then - 2a (1-8) = 216
216 108 .
8 =
14 7 .
= which Is not an integer.

Self practice problems :


is 728.
(7) Find the G.P. if the common ratio of G.P. is 3. n" term is 486 and sum of first n terms
common ratio of G.P.
(8) If x. 2y. 3z are in A.P. where the distinct numbers x. y. z are in G.P. Then find the
S and that of the terms
(9) A G.P. consist of 2n terms. If the sum of the terms occupying the odd places is 1
occupying the even places is S 2 • then find the common ratio of the progression.
in pairs Is 156. Find the
(10) If continued product of three number in G.P. is 216 and sum of there product
numbers.
Answers.
1 (10) 2.6. 18
(7) 2. 6. 18. 54. 162. 486 (8) (9)
3 s,
Remarks: .•
If a. b. care in G.P.::) b = ac. in general if'a,. a,. a1 • a ........... a._, • a. are in G.P
2
(i)
then a,a. = a 2a0 _ , = a, a. _2 = ..........................

Any three consecutiv e terms of a G.P. can be taken as ~.a. ar.


(ii) r

Any four consecutiv e terms of a G.P. can be taken as. • ar. ar3.
(iii) r r
same non-zero
If each term of a G.P. be multiplied or divided or raised to power by the
(iv) quantity. the resulting sequence is also a G.P ..
ly. then
If a,. a,. a ........ . and b,. b • b........... are two G.P·s with common ratio r, and r2 respective
3 2
(v)
the sequence a,b,. a 2b2 • a1b1 ............ Is also a G.P. with common ratio r, r2•

If a,. a,. a 3........... are in G.P. where each a, > o. then log a,. loga, . loga ............are in AP. and its
(vi)
converse is. also true.

Example # 16 : ,
the new numbers are in
Three numbers form an increasing G.P. If the middle number is doubled. then
A .P. The common ratio of G.P. is :
Solution:
Three number in G.P.are ~ . a. ar then ~. 2a ar are In A.P. as given.
r r

2(2a) = a (r+n or r 2 - 4r + 1 =0

or
or r =2 + as r > 1 for an Increasing G.P.
Sequence & Series /,---------------------------j•
Example # 17 .
The sum of an infinite geometric progression is 2 and the sum of the geometric
progressio n made from
the cubes of this infinite series is 24. Then find its first term and common ratio
:.
Solution :
Let a be the first term and r be the common ratio of G.P.
a a3
1-r = ' 1-r3 = 24 ' -1 < r < 1

Solving we get a = 3, r = -~
2
cABmple# 18

Express 0.423 in the form of .e_ , (where p, q e I, q 'F 0)


q
Solution:
4 a 4 23 419
= -+- =-+ - = -
10 1-r 10 990 990

Example # 19 :
Evaluate 9 + 99 + 999 + ........... upto n terms.
Solution:
Let S = 9 + 99 + 999 + .......... upto n terms.
= [9 + 99 + 999 + .......}
= [(10 - 1) + (10 2 -1) + (103 -1) + ........ + upto n terms]

=[10+102 + 101 + ...........+ 10" -n) = ( 10<1~"- 1) n)

Geometric means (mean proportion~!) (G.M.):

If a, b, care in G .P., bis called as the G.M. of a & c.


If a and care both positive, then b = .Jac
and if a and care both negative, then b = -./ac
.
b 2 = ac, therefore b = ./ac : a > 0, c > 0.
n--Geometrlc means between a, b :
If a, bare two given numbers & a, G 1, G , •• : •. , Gn, bare in G.P.. Then
2
G 1, G 2• G 3 ....... Gn are n G.M.s between
1••.•.•••
Gna=&a(b/a)nln•
b. 1
G = a(b/a)11n• 1• G = a(b/a)2ln•
1 2
Note : The product of n G.M.s between a & bis equal to the nth power of the single
G.M. between a & b
i.e. [I
,.,
G, = ( ./ab)" = Gn • where G is the single G.M. between a & b.

Example #20
Between 4 and 2916 are inserted odd number (2n + 1) G.M·s. Then the (n + 1)th
G.M. is
Solution:
4. G,.G 2• ••••••• G ••, ..... G •• G .. ,. 2916
2 2
G,,., will be the middle mean of (2n +1) odd means and it will be equidistan t from
1st and last
term
4.G•• ,. 2916 will also be in G.P.
G!., = 4 >< 2916 = 4 >< 9 >< 324 = 4 >< 9 >< 4 >< 81
G.,, = 2 >< 3 >< 2" 9
= 108.
Reso nance®
Educating for better tomorrow
Corporate Office: CG Tower. A-46 & 52. IPIA. Near City Mall. Jhalawar Road. Kola
Website : www.resonance.ac.ln E-mail : contact
res onance.ac.ln
(Raj.)-324005

Toll Free : 1800 258 5555 CIN: U80302RJ2007PLC024029


Sequence & Series /~-- ---- ---- ---- ---- ---- ---- m-
Self practice problems :
an•I +bn+'
(11) Find the value of n so that - - - may be the G.M. between a and b.
a• +bn
(12) If a= 111 .......... 1, b = 1 + 10 t 102+ 102+ 10" and c= 1 + 10' + 1010 + ..... + 10"',
then prove that
55
(I) 'a' is a composite number (II) a= be.
Answers.
1
(11) n=--
2
Harmonic progression (H.P.)
A sequence is said to be in H.P if the reciprocals of its terms are in A.P.. If the sequence
a,. 82· •.....
an is in H.P. then 1/a1, 1/a , •••• , 1/an is in A.P.
2
Note : (I) Here we do not have the formula for the sum of the n terms of an H.P.. For H.P.
whose first
term is a and second term is b, the nth term is~ = ab
b + (n- 1)(a- b)
· . 2ac a a- b
(ii) If a, b, care in b = - - or - = - - .
a+c c b-c

(iii)
.
If a, b, care in A.P.
a-b
-- = -
a
b-c a
(Iv) If a, b, care in G.P. a-b = _!
b
b-c
Harmonic mean (H.M.):
If a, b, care in H.P., bis called as the H.M. between a & c, then b = 2ac
a+c
If a , a ••••••••• an are 'n' non-zero numbers then H.M. 'H' of these numbers is given
1 2
by

.! n a, _!_+
= .!·[_!_+ a 2 ....... +_!_]
an
H
Example # 21
The 7th term of a H.P. is _!_ and 12th term is_!_ . find the 20th term of H.P.
10 25
Solution :
Let a be the first term and d be the common difference of corresponding A.P.
a+6d=1 0
a+ 11d = 25
5d = 15
d = 3, a =-8
T =a+19d =-8+19 x3 =-49
20

1
20 term of H.P. =
49
Example # 22
Insert 4 H.M between~ and
· 4 19
2-.
Solution :
Let 'd' be the common difference of corresponding A.P..

Aeso
EducaUng
nanc e~ Co,porate Otflc:e: CG Tower,
Website : www.resonance.ac.ln
& 52, IPIA, Near City Mall, Jhalawar Road, Kola (Raj.}-324005
E•mall : contact resonance.ac.ln
for better tomorrow Toll Free : 1800 258 5555 CIN: U80302RJ2007PLC024029
Sequence & Series / ,--------------~IIIL _
_,---

19 4
so d = = 1.
5

-
H1
1 4
= - + 1 =-
3
7
3
or H=I
' 7

...!..:i+2=~ 3
or H2 = ·1 0
H2 3 3
1 4 13
- = - +3=- or H =
3
2_
Hl 3 3 13
4 16
= - + 4=- or H = 2_
H, 3 3 ' 16.
Example # 23 .
, 2 d 12
Find the largest positive tenn of the H.P., whose first two tenn are an
5 23
Solutlon.
5
The corresponding A.P. is
2,
23
....... or
30
, 23 , 16 • 9 · 2 ' -5
12
12 12 12 12 12 12
The H.P. is 12 12 12
30 ' 23 ' 16 ' 9 '2 ' - 5 ' ......
Largest posI·1·Ive tenn = -12 = 6
2
Self practice problems :

(13) If a, b, c, d, e are five numbers such·that a, b, care in A.P., b, c, dare in G.P. and c, d, e are in H.P.
prove that a, c, e are in G.P.

(14) If the ratio of H.M. between two positive numbers 'a' and 'b' (a> b) is to their G.M. as 12 to 13, prove
that a : b is 9 : 4.

(15) a,b,careinH.P.thenprovethat b+a + b +c =2


b- a b -c
(16) If a, b, c, d are in H.P., then show that ab + be + cd = 3ad

Arlthmetlco-geometrlc series :
A series, each tenn of which is formed by multiplying the corresponding tenns of an AP. & G.P. is
called the Arithmetico-Geometric Series. e.g. 1 + 3x + Sx2 + 7x3 +.....
Here 1, 3, 5, ... are in AP. & 1, x, x 2, x3· ... . are in G.P..
Sum of n terms of an arithmetlco,ceometrlc series:
Let Sn= a + (a + d) r + (a + 2 d) r2 + .... + [a+ (n - 1)d] rn-1, then

a dr(1- r"-' ) [a +(n- 1)d]r"


Sn=-+ ----~- ra!1 .
1-r (1-r)2 1-r

Sum to Infinity: If Ir I < , & n oo , then Limit r" = 0 and Limit n.r" :: o
n "'"'
a dr
S,.= - + - - 2 .
1- r (1-r).

Aesonance®
Educating for bttter tomorrow
Caporale Ofllce: CG Tower. A-46 & 52, IPIA, Near City Mall. Jhalawar Road, Kola (Raj .)-324005
Website: www.resonance.ao.ln E-mail : conlacl resonance.ac.ln
Toll Free : 1800 258 5555 CIN: U60302RJ2007Pl-C024029
Sequence & Series /

Example# 24 :
2
The sum to n terms or the series 1 + 5 -4n -
+ 1 \I + 9 ( -
4n -
+ 1 ) + 13 ( -
4n -
+ 1)' + ...... la •
( 4n-3J 4n-3 4n-3
Solution
4n+ 1
Let x= - - , then
4n-3
-4 1
1-x= - - , - -
4n-3 1- X 4
_ x_=_(4n+1)
1-x 4
S = 1 + Sx + 9x2 + ....... + (4n - J)x,._,
Sx = x + Sx 2 + ........ (4n - 3)x•
S - Sx = 1 + 4x + 4x2 + ......... + 4x,._' - (4n - 3)x•.
4x
S(1-x) = 1 + - [1-xn-')-(4n -3)x•
1-x
1 4 4 3
S = - - [1 + - x" -(4n-3)x"] =- ( n- )[1-(4n+ 1)+ (4n- 3)x" -(4n-3)x"J "'n (4n - 3).
1-x 1-x 1-x 4
Example # 25 :

Find sum to infinite terms or the series 1 + 2C~ ) + 3u~r + 4 U~J + ...

Solution.

Let x = (;~)

Let S = 1 + 2x + 3x2 + 4x3 + ..... ........ (i)


xS = x + 2x2 + 3x3 + ......... .......(ii)
(i) - (ii) (1 - x) S = 1 + x + x 2 + x3 + .........

or S=-1-
(1 - x)2

;~r
1
S = = 100
(1-
Example# 26
Evaluate : F + 22x + 32x 2 + 4 2x3 •••• ••• upto infinite t6rms for I x I < 1.
Solution :
Let ...(i)
xs = 12 x + 2 2x2 + 32x 3. . ................ co ...(ii)
(i)-(ii)
(1 - x) s - 1 + 3x + 5x2 + 7x3 + ........
1 2x
(1-x)s= - - + - - 2
1-x (1-x)
1 2x
, +
s= - - -
(1- x) (1-x)3
1-x + 2x
s= 3
(1-x)
1+ X
s=--
3 '
(1- X )
nesonance® Corporate Office: CG Tower, A-46 & 52, IPIA, Near City M,11, Jhal1w.r Road, Kot1 (Rlj.)-324005
r'( Website : www.rnonance.ac.ln E•m•II : contact rasonance.ac.ln
Educat1119 for better tomorrow t-;'To1-::1"=F--re'-e--:-::':e:-::00~2s==e"=s:'
, -:'ss=-=s?:=:c~1N.;.,:.,:,u'="'eo='=3'='
02:::Rc-:J"'20'"'0~1P~L"=c'='o2""'4f:02::c;9,.=,;-==---
Sequence & Series / , - - - - - - - - - - - - - - - - - - - - - - - - - - - , •

Self practice problems :

4 4
(17) If 4 + + d + + Zd
2
............ = 1, then find d.
5 5

(18) Evaluate : 1 + 3x + 6x2 + 10x3 + ...... upto infinite term, where I x I < 1.

(19) Sum ton terms of the series ,: 1 + 2 (1+~) + 3 (1+~J + .....

Answers.

(17)
64
(18) (19) n2
5

Relation between means :

(i) If A, G, H are respectively AM., G.M., H.M. between a & b both being positive, then G 2 = AH
(i.e. A, G, Hare in G.P.) and A ~ G H.
Example# 27
8
The A.M. of two numbers exceeds the G.M. by 2 and the G.M. exceeds the H.M. by- ; find the
5
numbers.
Solution.
Let the numbers be a and b, now using the relation

G2 =AH=(G+2) ; A=10

i.e. ab = 64 also a + b = 20
Hence the two numbers are 4 and 16.
Results:
(i) I
r. 1
(a,± b,) = I
r=1
a,± I
r• 1
b,.

(ii) :t k a,= I k a,.


' . 1 r"' 1
n

(iii) L k = k + k + k + ...............n times= nk; where k is a constant.


,.,
(Iv) t 1
r=1+2+3+ ...........+n= n(~+ )

(v) I
,. ,
r' = 12 + 22 + 32 + ...........+ n2 = n (n+1) (2n+1)
6

(vi) I
, . 1
r3 = 13 + z3 + 33 + ........... + n3 = n2 (n+1)2
4
Example# 28
Find the sum of the series ton terms whose n•h term Is 3n + 2.
Solution.

s. = tT. = :E(3n + 2) = 31:n + l:2 = 3(n + 1) n n (


+ 2n = 3n + 7)
2 2

n nance®
I Educating for better tomorrow
Corporate Orrico: CG Tower, & 52. IPIA, Near City Mall, Jhnlnwnr Rood. Kol11 (RIIJ.)·324005
Website : www.reaonance.ac.ln E•mall · contact rt110 anco.ftr.,ln
Toti Freo : 1800 258 5555 CIN: U80302RJ2007PLC024021l
Sequence & Series /r---------------------------•
Example#29

T 0 = k3 + 3•, then find i ; r•.

Solution:
..,
2 2
i;T. =:tk3 + :t3k = (n(n+1) ) + 3(3" -1) = ( n(n + 1) ) + (3" _ 1)
••t k• 1 k• 1 2 3-1 2 2

Method of difference for finding n"' term :

Let u,, u2, u, ........ be a sequence, such that u2 - u,. u, - u2, .......... is either an A.P. or a G.P . then nth
term u. of this sequence is obtained as follows
S = U1 + U2 + U3 + .......... + U0 •••••••• •••••••• (i)

s= u, + u, + ··········· + u..., + u. . .............. (ii)


(i)- (ii) u. = u, + (u2 - u,) + (u, - u2) + .......... + (u. - u...,)

Where the series (u 2 - uJ+ (u, - u2 ) + ......... + (u. - u...,) is

either in A.P. or in G.P. then we can find u•. So sum of series S = i; u,


,.,
Note : The above method can be generalised as follows :
Let u,, u,. u3, • •• •••• •• be a given sequence.
The first differences are t:.,u,, t:.,u2, t:.,u,, ....... where t:.,u. = u2 - u,, t:.,u2 = u3 - u2 etc.
The second differences are 6 2u,, t:. 2u,, 6 2u3 , ••• •••• , where 6 2u1 = 6 1u2 -61u1, 6 2u2 = 6 1u3 - t:.,u 2 etc.
This process is continued untill the k" differences t:._u, , t:.,u2 , •••••••• are obtained, where the k"
differences are all equal or they form a GP with common ratio different from 1.
Case - 1 : The k" differences are all equal.
In this case the n" term, u. is given by
u. = a0 n• + a,n...' + .... + a, , where a0 , a,, .... , a, are calculated by using first · 'k + 1' terms of the
sequence.
Case • 2 : The k" differences are in GP with common ratio r (r -,, 1)
The n" term is given by u. = ).. r--' + a0 n..., + a, n...2 + :··:· + a._,

Example# 30
Find then" term of the series 1, 3, 8, 16, 27, 41, .........

Solution.
s = 1 + 3 + 8 + 16 + 27 + 41 + ..... T. ..... (i)
s= 1 + 3 + 8 + 16 + 27 ......T ..., + T. .. ...(ii)
(i) - (ii)
T.= 1 + 2+ 5+ 8+ 11 + ....... (T0 -T0 _ 1 )
1
T =1+("- ) [2x2+(n-2)3) =~ [3n2 -5n+4)
• 2 2

Example# 31
Find the sum ton terms of the series 5, 7, 13, 31, 85 + .....
Solution.
Successive difference of terms are In G.P. with common ratio 3.
T. = a(3)•-• + b

Aesonance®
Educating for better tomorrow
Co,por1te Office: CG Tower, A--48 & 52, IPIA, Near Clly Mall. Jhalawar Road,
Website: www.rasonanca.ac.ln E-mail : contact resonanca.ac.ln
(Raj.}-324005

ToU Free : 1800 258 5555 CIN: U80302RJ2007PLC024029


·sequence & Series / , - - - - - - - - - - - - - - - - - - - - - - - - - - •

a+b=S
·3a + b= 7 a= 1, b = 4
T. = 3•- 1 + 4
s. =ET.= 1:(3•-• + 4) = (1 + 3 + 3 2 + ..... + 3•- 1) + 4n
1
[3•+ Bn-1)
2
Method of difference for findings.
If possible express r"' term as difference of t...:o terms as t, = ± (f(r) - f(r ± 1)). This can be explained
with the help of examples given below.
t, = f(1) - f(O),
I, = f(2)- f(1 ).

t. =f(n)-f(n-1)
s. = f(n)-f(O)
Example #32
Find the su,n of n-terms of the series 2.5 + 5.8 + 8.11 + ..........
Solution.
T, = (3r - 1) (3r + 2) = 9r2 + 3r - 2
n n n n

s. = L), =9 L), + 3 I:r - I:2


r:1 r:1 ,~1 r:1

:g (n(n+1~2n+1))+ 3 (n(n +1))_ 2n


2
= 3n(n + 1)2-2n

Example#33
1 1 1
Sum to n terms of the series + + -----+
(1 + x)(1 + 3x) (1 + 3x)(1 + Sx) (1 + 5x)(1 + ~x)
Solution.
Let T, be the general term of the series

T = 1
' [1+ (2r-1)x ][1 + (2r + 1)x]

_ 1 [(1+(2r-+'-1 )x)-(1+(2r-1)x)]-
So T, - 2x (1+(2r-1)x}(1+(2r+1)x ) -

s.: I: T,: T, +Tl+ T3 + ......... + T.

= 2x [ 1:x - (1+(~+1)x)]= (1+x)[1+n(2n+1)x]

Example# 34
1 1 1
Sum to n terms of the series
1.4.7 + 4.7.10 + 7.10.13 + ...........
Solution.

T- 1 _1[ 1 1 ]
.- (3n-2)(3n+1)(3n+4) - 6 (3n-2)(3n+1) (3n+1)(3n+4)

nesonance®
r"'(
Corporate Office: CG Tower, A-46 & 52, IPIA, Near C ity Mall, Jhalawar Road, Kola (Raj.}-324005
Website : www.resonance.ac.ln E-mail : contact resonance.ac.ln
Educating for better tomorrow 1-T'--'-o1.=::1=F,;;;:ee-'-:"""1""'soo"--"---=2=,.:ss""s=c.s""ss=c"'"1Na...:u-=-a~o""30""'2'""RJ=2=00=1"'"pL""c=o2=4""02=9=""'-'----
Sequence & Series / ------------- _,--- -

= i [(
/4 - 4\ )+( 4\- 7.~o)+ ..... + (3n-2) (3n+1)

= i[¾ (3n+1) ~3n+4 )]

Examp le# 35

Find the general term and sum of n terms of the series


1 + 5 + 19 + 49 + 101 + 181 + 295 + .........

Solution:
The sequence of difference between successive term 4, 14,
30, 52, 80 .....
The sequence of the second order difference is 10, 16, 22,
28, ..... dearly ii is an A.P>
so let nth term
T =an'+ bn2+en+ d
0

a+b+ c+d =1 .•..(i)


8a + 4b + 2c + d = 5 ... ~i)
27a + 9b + 3c + d = 19 ..(iii)
64a + 16b + 4c + d = 49 ... (iv)
from (i), (ii), (iii) & (iv)
a = 1, b = -1, c = 0, d = 1
2
n(n +1)) _ n(n+1) (2n+1) + n= n(n2-1 ) (3n+2)
s0 =I:(n' - n2 + 1 ) = +n
( 2 6 12

Self practice problems :

(25) Sum to n terms the following series


3 5 7
(I) - -2+ -2- 2 +-2-2 + ..........
12 .2 2 .3 3 .4
(ii) 1 + (1 + 2) + (1 + 2 + 3) + (1 + 2 + 3 + 4) ........

(iii) 4 + 14 + 30 + 52 + 82 + 114 + .........

"
(26) If LT,= (n + 1)(n + 2)(n + 3) then find " 1 r-
Answers.
2 n(n+1) (n+2)
2n+n
(25) (i) (ii) (Iii) n(n+ 1)2
2 6
(n + 1)
n3 +3n
(26)
2(n+1) (n+2)

Resonance®
I Educating for better tomorrow
Corporate Office: CG Tower, A-46 & 52, IPIA, Near Ctty

1 WT'
Mall. Jhalawa r Road, Kota (Raj.}-32 4005
;.;'eb~s;;ije;;'::'" ;wwwi'ii-;;;:.~res~on~anc~e.~ac;;';.ln~E';;-m2;a~II:::::c~on~t~ac;!,!t~r~es~on!!la~n!gce~.ac!!!,:!!.ln:!_
Toll Free: 1800 258 5555 CIN: U80302R J2007PLC024029
__
'
Sequence & Series / ,---------------j--
• ·Exercise-1
PART - I : PRE RMO
1. In the following two A.P.'s how many terms are identical?
2. 5. 8. 11 .... to 60 terms and 3. 5. 7• .... 50 terms
2. The value of 9 113 • 9 119. 91121 ......upto · oo, is-
3. The sum of 10 terms of the series 0.7 + .77 + .777 + ........is-

4. The n"' terms of the series 1 + + 2. +


. 5 52 53
i
+ ...... Is

5. The sumof .m fiIm·t e Ierms of the series


. 5 --
7 +9- -11 . .
- + .....oo Is
3 3 2 33
6. The sum of the series 1.2 + 2.3 + 3.4 + ...... up to 20 terms is

7. L - 1-1 is equal to:


ao

r =2 r
2
-

8. If (12- t,)+ (2' -t,) + ..... + (n' -t.) = J n(n2 - 1), then t. is
x,oo
9. If x > o. then the expression 2 3 200
is always less than or equal
to 1+x+x +x + +x ·· ·
10. Given the sequence a. ab. aab. aabb. aaabb.aaabbb ...... upto 2004 terms. the total number of limes
a·s and b·s are used from 1 lo 2004 terms are :
11. A sequence a0 .a,.a,.ar ·····•a .. is defined such that a0 =a, =1 and a.,, =(a,,_,_a.) + 1 for n ~1. Which of
the following is true ?
12. The first two terms of a sequence are O and 1. The n., terms T. = 2T,,_, - T,,_2 • n 3. For example
= = =
the third term T3 2T2 - T, 2 - 0 2. The sum of the first 2006 terms of this sequence is :
13. Consider the following sequence : a,= a, = 1. a, = 1+ minimum {a.,. a14} for i > 2. Then a,_=
1 1 1 ·1
14. The sum of ,[1
2 1+ 1 2
+ --~--=
+ 2./3
+ ----,~-= + +
4./3 + 3~ ····· 2sffe +
is

1
15. If f(x) + f(1-x) is equal to 10 for ~II real numbers x then f ( -- ) + f(.2...) + f(2-) + ...+ f(~) equals
. . . . 100 •, .. 100 100 100

16. Consider the sequence 4.4,8,2.0,2.2,4.6.0 .... where the nth term is the units place of the sum of the
previous two terms for n 3. If ~.is the sum ton terms of this sequence then the smallest ·n• for which
s.> 2010 is:
17. For some natural number •n·, the sum of the fist •n· natural numbers is 240 less than the sum of the first
(n + 5) natural numbers. Then n itself is the sum of how many nat!,Jral numbers starting with 1.
18. An arithmetical progression has positive terms.' The ratio of the difference of the 4 th and 8th term to
15th
th
term is and the square of the difference of the 4 and the 111 term is 225. Which term of the series
15
is 2015?
19.
The 12 numbers a,. a, ......... a,. are in arithmetical progression. The sum of all these numbers is 354.
Let P = a, + a, + .......... + a 12 and Q = a, + a3 + ...... + a,,. If the ratio P : Q is 32 : 27. the common
• difference of the progression Is
n nanc:e Corporate Office: CG Tower. A..C6 & 52, IPIA. Near City M;ill. Jhalawar Road. Kola (Raj.}-324005
t"'CeSO - Website: www.rMonance.ac.ln E-mail : contact resonance.ac.ln - - •
EducaUng for bettertomorr Toll free : 1aoo 25a 5555 CIN: U80302RJ2007PLC024029
Sequence & Series / - - - - - - - - - - - - - - - - - - - - - - - - - - - •

20. Each term of a sequence is the sum of its preceding two terms from the third term onwards. The
second term of the sequence is -1 and the 10th term is 29. The first term is _ _ .
21. n is a natural number. It is given that (n + 20) + (n + 21) + .....+ (n + 100) is a perfect square. Then the
least value of n is _ _ _ __
22. In a G.P. of real numbers. the sum of the first two terms is 7. The sum of the first six terms is 91. The
sum of the first four terms is _ _ _ __
23. In a potato race, a bucket is placed at the starting point, which is 7 meter from the first potato. The
other potatoes are placed 4 m a part in a straight line from the first one. There are n potatoes in
the line. Each competitor starts from the bucket, picks up the nearest potato, runs back with it,
drops in the bucket, runs back to pick up the next potato, runs to the bucket and drops it and this
process continues till all the potatoes are picked up and dropped in the bucket. Each competitor ran a
total of 150 m . The number of potatoes is. _ _ __
24. The coefficient of the quadratic equation ax2 + (a + d)x + (a + 2d) = O are consecutive terms of a

positively valued, increasing arithmetic sequence. Determine the least integral -,·3lue of such that the
a
equation has real solutions.
25.
Find the sum of integers from 1 to 100 that are divisible by 2 or 5
26.
The sum of three numbers in A.P. is 27, and their p_roduct is 504, find them.
27.
Three friends· whose ages form a G.P. divide a certain sum of money in proportion to their ages. If they
do that three years later, when the youngest is half the age of the oldest, then he will receive 105
rupees more that he gets now and the middle friend will get 15 rupees more that he gets now, then find
the ages of the friends
28. 5 4 3 2
40x + ax + bx +ex+ d = 0 are in GP. If sum of reciprocals of the roots
The root; of the equation x -
is 10, then find !cl and ldl
29.
Let T" denotes the nth term of a G.P. with common ratio 2 and (log 2(Iog3(Iog512 T 100))) = 1. If three sides

of a ::"-=1ngle ABC are the values of (T 1 + T 2 ), T2 and T 3 then area of the triangle is v'2160 , where N is
N
10
a positive integer. Find the remainder when N is divided by 2 .

30. 2 2 2
2
If a, b, care in A.P. and if (b - c) x + (c - a) x + a - b =0 and 2(c + a) x 2 + (b + c)x =0 have a common
root, then show that a , c , b are in A.P.

31 . Along a road lies an odd number of stones placed at intervals of 10 m. These stones have to be
assembled around the middle stone. A person can carry o;,iy one stone at a time. A man carried out the
job starting with the stone in the middle, carrying stones in succession, thereby covering a distance of
4.8 km. find the number of stones
2 2
32. a, b, c are positive real numbers forming a. G.P. If ax + 2bx + c = 0 and dx + 2ex + f = O have a
common root, then prove that d/a, e/b, fie are in A.P. ,. ,
Determine all pairs (a, b) of real numbers such that 10; a, b, ab are in arithmetic progressiun.
33.

34. If . 1+-+1
12 2
1
2
1
2 + 1+ 2 + 2 + 1+
3
1 FH1·
4
1
(1999)
2
1
- - 2 ~ + ... + 1+--- + - - - =x-- thenfindthevalu eofx
(2000)2 X' .
1 .

35. If n is any positive integer, then find the number whose square is 111 ........ 1 - 222 ........ 2
2n times n limes

5 7
36. Find the sum of infinite terms of the series : + - - + - -- + . . :. . + ........
2.4 2.4.6 2.4.6.8 2 4 8 10
37. If S, , S2 , S3 , ....... , S2n are the sums of infinite geometric series whose first terms are respectively
1 , 2 , 3 , ........ , 2 n and common ratio are respectively ,
11 1 S 12 + S22 + ........ + s220 _ 1 •
, , ........... 2n + 1 • fiind the value of
23
11111 Resonan ce Co,porate Office: CG Tower, A-46 & 62, IPIA, Near City Mall, Jhalawar Road, Kota (Raj.}-324005
a1 Edueatl119forbettertomon- Website : E•m•II : contact
Toll Free : 1800 258 5556 CIN: U80302RJ2007PLC024029
Sequence & Series / , - - - - - - - - -r,a,--
-
2 2 2 2 2 2
38. Find the Sum of series 1 + (1 + 2 ) + (1 + 2 + 3 ) + ..... upto n terms

39.
__Let a1 i(
= ¾) -
11
, j eN. If b = af + a 1 , sum of the i~finite series fonned by b/s Is (10 + a ) where
1

is equal to ( [ ] represent greatest integer funciion)


40. The sequence 9, 18, 27, 36, 45, 54, ..... consists of successive multiples of 9. This sequence is then
altered by multiplying every other term by -1, starting with the first term, to produ~e the new se~uence
-9, 18, -27, 36, -45, 54, ......... If the sum of the first n terms of this new sequence 1s 180, determine n.
41. As shown in the figure, the five circles are tangent to one another consecutively and to the lines L,
and

rrxxx r
Lz. If the radius of the largest circle is 18 and that of the smaller one is 8, If radius of the middle circle
is r, then find the value of r/2 .

L,
42. The arithmetic mean of the nine numbers in the given set {9, 99, 999, ....... 999999999} is a 9 digit
number N, all whose digits are distinct. Then which digit does not appear in number
43. Let a sequence whose n th term is {aJ be defined as··
. 1
a, = - and (n - 1)an_ 1 =;.{n + 1)an for n 2 th(!n find Lim Sn
2 · .

44. If x =
- +-
12 2232 10012
+ - + ........ + - - , y
12. . 2 2 .3 2
= - + - + - + ........ +
2
1001 , then
[ I
x-y is equal to
1 3 5 2001 3 5 7 2003 10
(where (.] denotes greatest integer function)
45. Let K is a positive integer such that 36 + K, 300 + K, 596 + K are the squares of three consecutive
tenns of an arithmetic progression. Find K.

PART - II : RMO
1. Show that all the terms of a strictly increasing AP. of positive integers cannot be prime numb!:'rs.
2. The first tenn of an A.P. is equal to first term of a G.P. and first term of an H.P.. Further the last terms of
all these three sequences are also equal, moreover these three sequences also ghave equal number of
1
terms. If A, G, Hare the r hterm of A.P., G.P., H.P., respectively then show that A ~ G H
. . 1
3. Let sequence {an} be defined as below_.
·.'
a, =1, when n 1, an+1
. =an+ - , then show that 12 < a 75 < 15

4. Show that for all a> 1 there exists ·an infinite sequence of positive real numbers x,, xi,
......................................................................................,
. such that Xn+2 = Jax•• , - xn for all n 1

5. A stricUy increasing sequence of positive integers a,, a2, a3, ..... has the property that for ever positive
integer k, the subsequence a2k-1, a2k, a2k•1 is geometric and the subsequence a 2k, a 2k.,, a2k•2 is
· arithmetic. Suppose that a, 3 = 2016. Find a 1.
1
6. Consider the sequence an+1 = -
1-an
-, n 1 .Given that a, =.!2 find sum of first 100 terms of this

sequence
111 1 1 1 1 1
7. Find the value of
1 +J2+s2° + + . +9972 + 9992 _10022_ 1D<M2 _10062 __ ···· -20002
1 1
1+ 2+2+2+ 1 1 1
........ + --2 + - - 2
2 3 4 999 1000
8. A geometrical progression consists of 37 positive Integers. The first and the last terms are relatively
prime numbers. Prove that the 19 h terms of the progression In the 18'h power of some positive Integer.
1

n ·
Corporate ornce: CG Tower, A-46 & 52, IPIA, Near Clly Mall, Jhalowor Rood, Kola (Roj.)-324005
rteSOnanc:e Website : www.resonance.ac.ln E-mail : contact resonance.ac.ln
EducaUng for better tomorrow Toll Free : 1800 258 5555 CIN: U80302RJ2007PLC024029
Sequence & Series / r-------------------------•

• Exercise-2
PART - I : PREVIOUS ASKED QUESTION FOR PRE RMO
n 1 99 1
1. Let Sn = L
k•O k + 1+ k
. What is the value of } : - - - ?=
n•1Sn + Sn.\
[PRMO 2013)

2- Let m be the smallest odd positive integer for which 1 + 2 + ......... + m is a square of an integer and let
n be the smallest even positive integer for which 1 + 2 + ........ + n is a square of an integer. What is the
value of m + n? [PRMO 2013)
3. What is the maximum possible value of k for which 2013 can be written as sum of k ccnsecutive
positive integers ? [PRMO 2013)

4. The first term of a sequence is 2014. Each succeeding term is the sum of the cubes of the digits of the
previous term. What is the 20141h term of the sequence? (PRMO 2014)

5. · A sequence of positive (a,. a2 , ... a.) is called good if a1 = a, + a2 + ... + a1-, for all 2!> i !> n. What Is the
maximum possible value of n for a good sequence such that a. = 9216? (PRMO 2015)

..,
2015 k(lc.•1)
6. Find the sum S = }: (-1 )- - k 2
[PRMO 2015)

7. A new sequence is obtained from the sequence of positive integers 1, 2, ........., by deleting all the
perfect squares. What is the 2015-th term from the beginning of the new sequence?
[PRMO 2015)
8. Let E(n) denote the sum of the even digits of n. For example, E(1243) = 2 + 4 = 6. What is the value
of E(1) + E(2) + E(3) + .... + E(100) ? [PRMO 2015)
1
9. Consider the sequence 1, 3, 3, 3, 5, 5, 5, 5, 5, 7, 7, 7, 7, 7, 7, 7, ..... and evaluate its 2016 h term.
[PRMO 2016Delhl]
10. Find the value of 1he expression
4 2 4 2
(3' + 3 2 + 1).(5' + 5 2 + 1).(7' + 7 2 + 1).(9' + 9 2 + 1).(11 + 11 + 1).(13 + 13 + 1)
2
o•2 2
(2' + 2 2 + 1).( 4 • + 4 2 + 1).(6'.6 2 + 1).(8' + 8 + 1).(1 + 10 + 1).(1 2• + 12 + 1)
When written in lowest form. [PRMO 2016Delhi]

1 1
11. Let S = 1 + - + - + + + ~ - Find [SJ [PRMO 2016WB]
✓ --.,4 --.,99 --.,100

You may use the fact that ( < i){ ..fn + < Jn + 1 for all integers n 1.

12. For positive real numbers x and y, define their special mean to be average of their arithmetic and
geometric means. Find the total number of pairs of integers (x, y), with x y, from the set of number-
s (1, 2, ...,2016), such that the special mean of x and y is a perfect square. [PRMO 2016WB]
40 60
Let u. v, w be real numbers in geometric progression such that u > v > w. Suppose u = v" = w . Find
13.
the value of n. [PRMO 2017]
9

14. Let the sum L n(n + 1)(n


n;
1
+ 2)
written in its lowest terms be E. . Find the value of q - p.
q
1
[PRMO 2017]

15. Suppose x is a positive real number such that {x}, [x] and x are in the geometric progression. Find the
least positive Integer n such that x" > 100. (Here [x] denotes the integer part of x and {x} = x - [xi)
[PRMO 2017]

16. Five distinct 2-digit numbers are In a geometric progression. Find the middle term.
[PRMO 2017]

Resonance Corpo_r al~ Office: CG Tower, A-46 & 52. _IPIA, Near City Mall, Jhalawar Road. Kola (Raj.}-324005
. Ed 11 for better tomorrow Website. www.resonance.ac.ln E-mail . conlacl resonance.ac.ln • •
uca ng Toll Free : 1800 258 5555 CIN: U80302RJ2007PLC024029
Sequence & Series / , - - - - - - - - - - - - - - - - - - - - - - - - - - - •

17. Let N = 6 + 66 + 666 + ..... + 666 .... 66, where there are hundred 6's in the last term in the sum. How
many times does the digit 7 occur in the number N ? [PRMO 2018)

18. What· is the value of L (i + j) - L (i + j) ? [PRMO 2018)


1Sl<js10 1Sk js10
I+ J• odd I+J=even
.J
19. A book is published in three volumes, the pages being numbered from 1 onwards. The page numbers
are continued from the first volume to the second volume to the third. The number of pages in the
second volume is 50 more than that in the first volume, and the number pages in the third volume is
one and a half times that in the second. The sum of the page numbers on the first pages of the three
volumes is 1709. If n is the last page number, what is the largest prime factor of n ? [PRMO 2018)

PART - II : PREVIOUSLY ASKED QUESTION OF RMO


1. Prove that there exist two infinite sequences (a). , , and (b0 ) 0 , , of positive integers such that the
following conditions hold simultaneously : [RMO-2006]
(i) 1 < a, < a2 < a 3 < ...;
(ii) a. < b0 < a2_, for all n 1;
(iii) a. - 1 divides b0 - 1, for all n 1;
(iv) a2_-1 divides b'. -1 , for all n 1.

2. Three nonzero real numbers a, b, c are said to be in harmonic progression if ..!. + ..!. = 3. . Find all three-
a c b ·
term harmonic progressions a, b, c of strictly increasing positive integers in which a = 20 and b
divides c. [RMO-2008)
3. In a book with page numbers from 1 to 100, some pages are tom off. The sum of the numbers on the
x. 1,
Find
define
the number
a. = [[Jn°Jl
remaining pages is 4949. How many pages are torn off?
of all nwh~re
is the [xi
setdenotes
{1, 2, 3, the
...............................
largest integ~r ,2010} not ~xceeding
for whichx,a.>
for aany
[RMO-2009)
..,. (RMO-201
real number
OJ

4. Let a,,
For each .......... na.. be an arithmetic progression of positive real numbers with common differen~e d.
a,,integer
Let
(i) a! a!
+ + ......... + a!n-t= X,

5.

[RMO-2014)
(ii) a: + a! + ...... + a:,= y, and
(iii) a.+ a•• ,=
z.
Express d in terms of x, y, z, n,

6. Let (a 1,a 2 ,a 3 , .•• • .) be a strictly increasing sequence of- positive integers in an arithmetic progression.
Prove that there is an infinite subsequence of- the given sequence whose terms are in a geometric
progression. [RMO-2016)

7. (a) Given any natural number N, prove that there exists a strictly increasing sequence of N positive
integers in harmonic progression.
(b) Prove that there cannot exist a strictly increasing infinite sequence of positive integers which is in
harmonic progression. [RMO-2016)

8. (i) Prove that if an infinite sequence of strictly increasing positive integers in arithmetic progression has
one cube then it has infinitely many cubes.
(ii) Find with justification, an infinite sequence of strictly increasing positive integers in arithmetic
progression which does not have any cube. [RMO-2016)

n Corporate Office: CG Tower, A-46 & 52, IPIA, Near City Malt. Jhalawar Road, Kola (Raj.)-324005
Website : www.resonanca.ac.ln E-mail : contact resonanca.ac.ln
• EducaUng lor ~tt•r tomorrow Toll Free : 1800 258 5555 CIN:
Se9.uence & Series / m--
•• Answers
Exercise-1
l
l
PART - I : PRE RMO

1. 17 3n- 2
2. 3 3. -7 8
( 9 +1-) 4.
81 .

om
1010

5. 27/8 6. 3080 7. ¼ 8. n

t.c
9. 10. 1002 " 1003 a's ind (1002)2 b's 11. 4f a200,
201

po
2005x2006 4
12. 13. 1003 14. 15. 495
2 5

gs
16. 503 17. 9 18. 403 19. 5

lo
20. 3 21. 4 22. 28
23. m=7, n=15. 24. 3059 ' 25. 3050 26. 4, 9, 14

e.b
27. 12, 18,27 28. I d I = 32, I c I = 320 or 29. 0 31 . 31

33. (4, - 2), (%.- 5) 1


3-4. X = 2000 - -- 35. 333 ........ 3
' 2000 n times
lst
n (2n +1) ( 4n + 1)
36. 1/2 37. - 1
3
ia

n(n + 1)2 (n + 2)
38. 39. 40. 40 41. 6
fic

12
42. 0 43. 44. 50 45. 925
of

PART- II : RMO
er

3
5. 504 6. 50 7.
4


rn

Exercise-2 )
co

PART - I : PREVIOUS ASKED QUESTION FOR PRE RMO


ok

1. 9 2. 9 3. 61 4. 370 5. 12
o

6. 0 7. 2060 8. 400 9. 89 10. 61


.b

11. 18 12. 506 13. 48 14. 83 15. 10


w

16. 36 17. 33 18. 55 19. 17


w

PART - II : PREVIOUSLY ASKED QUESTION OF RMO


w

2. (20, 39, 780), (20, 38, 380), (20, 36, 180), (20, 135, 140), (20, 30, 60)

3 4. 22 - 1, 32 - 1, ..... ' 442 - 1 5.


y-x
3. d= -
zn

A Corporate Office: CG Tower, A~6 & 52, IPIA, Near City Mall, Jhalawar Road, Kota (Ra].)-324005
Edu~u~eci11~US.~ Website: www.resonance.ac.ln E•m•II: contact resonance.ac.ln
Toll Free : 1800 258 5555 CIN: U80302RJ2007PLC024029
Geometry
_G_e_om_etry_...__ _ _ _ __,/'"-------------------------1•

I Geometry· I
1. The area of two triangles having equal bases (heights) are In the ratio of their heights (bases).
C

B
Area oft. ADB hi
Area of t.ACB = h2

2. If ABC and DEF are two triangles, then the following statements are equivalent :
BC CA AB
(A) =
LA LO, LB LE, = LC= L F (B)
EF FD DE
AB DE .
(C) -AC =-DF andLA=LD.

. A
Two triangles satisfying any one of these conditions are said to be similar to each other.

B E~E

3, Ap~olonius Theorem : If D is the midpoint of the side BC in a triangle ABC then


AB + AC 2 =
2(AD2 + BD2) 2(AD2 + CO2).
.
= A

4. Ceva's Theorem: If ABC is a triangle, Pis a point in its plane and AP, BP, CP meet the sides BC, CA,
· . BD CE AF
AB in D, E, F respectively then DC , EA. FB = + 1

Conversely, if D, E, F are points on the (possibly extended) sides BC, CA, AB respectively and the
above relation holds good, then AD, BE, CF concur at a point.
Lines such as AD, BE, CF are called Cevians.

F~

B D C

1
Aesonance
EducaUng for better tomorrow
omce: CG Tower,
Website : www.roaonance.ac.ln
& 52, IPIA, Near City Mall, Jhalawar Road, Kola (Raj.}-324005
: contact resonance.ac.ln
Toll Free : 1800 253 5555 CIN: U80302RJ2007PLC024029
I

_G;...e_o_m_e:..:.t""ry'--_ _ _ _ _/ - - - - - - - - - - - - -
--- - - - - - - - - -,m- -
5. Menelaus's Theorem : If ABC is a triangle and a line meets
the sides BC, CA, AB in D, E, F respectively
then BO . CE . AF = _1
DC EA FB
taking direct"ions of the line segments into consideration i.e.,
for example, CD = -OC. Conve~sely if on
the sides BC, CA, AB (possibly extended) of a triangle ABC,
points D, E, F are taken respectively such
that the above relation holds good, then D, E, F are collinea
r.
A

,/
6. Ptolemy's theorem : If ABCD is a cyclic quadrilateral, then
AB. CD+ AD. BC = AC. BO
Conversely, if in a quadrilateral ABCD the above relation is
true, then the quadrilateral is cyclic.
B

7. If two chords AB and CD of a circle intersect at a point O


(which may lie inside or outside the circle),
then AO.OB = CO. OD. Conversely, if AB and CD are two
line segments intersecting at O such that
AO.OB = CO.OD, then the four points A, B, C, D are concycl
ic.
C

8. If OA is tangent to a circle at A from a point O outside the


circle and OCD is any secant of the circle
(that is, a straight line passing through O and intersecting
the circle at C and D), then OA2 = QC.OD.
Conversely, if OA and OCD .are two distinct line segmen
ts such that OA2 = OC .OD, then OA is a
tangent at A to the circumcircle of triangle ACD.

9. If AB Is a line segment in a plane, then set of points P


in the plane such that AP is a fixed ratio
PB
).(,o O or 1) constitu te a circle, called the Appolon
ius circle. If C and Dare two points on AB dividing the
line segment AB in the ratio ). : 1 Internally and externally,
then C and D themselves are two points on

Res onance® Website : Corporate Office: CG Tower, A-46 & 52, IPtA, Near City Mall.

Educating for better tomorrow


Jhatawar Road, Kola (Raj.)-324005
www.resonance.ac.tn E-mail : contact resonance.ac.ln
Toll Free : 1800 258 5555 CIN: U80302RJ2007PLC024029
G e_o_m...;e;.;..try...,__ _ _ ____,,./
__
, - - - - - - - - - - - - - - 1 - -_-, --
--

the circle such that CD is a diameter. Ahio, for any point P on the cirde, PC and PD are internal and
external bisectors of LAPB.

\
B

EXAMPLE-1 .
r
Let ABC be an isosceles triangle with AB = AC and let denote its circumcircie. A point D is on
the arc AB of r not containing C and a point Eis on the arc AC of r not containing B such that AD= C E.
Prove that BE is parallel to AD.
Sol. We note that triangle AEC and triangle BOA are congruent. Therefore AE = BO and hence
L ABE = L DAB. This proves that AD is parallel to BE.

EXAMPLE-2.
Let ABCD be a convex quadrilateral with AB = a, BC = b, CD = c and DA = d. Suppose
a2 + b 2 + c 2 + d2 =ab+ be+ cd + da and the area of ABCD is 60 square units. If the length of one of the
diagonals is 30 units, determine the length of the other diagonal.
Solution.
Given AB = a, BC = b, CD = c and DA= d and a2 + b 2 + c 2 + d2 =ab+ be + cd + da
2 2
=:> 2(a +b +c2 +d2 -ab-bc-cd-da)=O .

=> (a-b}' +(b - c }' +(c-d}' +(d-a)' = 0


=> So a - b =b - c = c - d = d - a = 0
a=b=c=d
Hence ABCD a Rhombus
Area of Rhombus = 60 sq. units
1
d, d2 = 60
2
1
- x30x d2 = 60
2
d2 = 4
So length of the other diagonal is 4 units

EXAMPLE-3.
In a triangle ABC , points D and E are on segments BC and AC such that BO = 3DC and AE = 4EC .
Point P is on line ED such that D is the midpoint of segment EP . Lines AP and BC intersect at point
S. Find the ratio BS/SD.
Solution.
Let F denote the midpoint of the segment AE. Then it follows that OF is parallel to AP . Therefore. in
triangle ASC we have CD/SQ= C FIFA= 3/2. But DC= BD/3 = (BS+ SD)/3. Therefore BS/SD= 7/2.

Example 4.
Three straight lines are drawn through a point P lying inside a triangle ABC, parallel to its sides. The
area of the resulting triangles are 1,4 and 9 sq.units. Find the area of t.ABC in square units.
Sol. Let the lines through P, parallel to BC meet AB and AC at L and M respectively. Similar1y for lines
parallel to AB and ·Ac. Note that triangle PLO, PNM and PRO are similar to the triangle ABC. Let 6 be
the area of t.ABC.
2 2 2
then 1 LP 4 PM and 9 OR
6 -- BC2 ' 6 -- BC2 6 = BC2
Now BC = BO + OR + RC

Resonance®
Educating for better tomorrow
Corporate Office: CG Tower, A--46 & 52, IPIA, Near City Mall, Jhalawar Road, Kota (Raj.}-324005
Website : www.resonance.ac.ln E-mail : contact resonance.ac.ln
Toll Free: 1800 258 5555 CIN: U80302RJ2007PLC024029
_G_e_o_m_e_try
_____ ----J/,-------------------------•
BC = LP + OR + PM
BC = BC + 3BC + 2BC
,Ji. ,Ji. ,Ji.
6
BC= ,Ji. BC

6
"
,Ji.= 6
= 36 sq.cm.

Example 5.
& 0 R
M

Inside • unit square, all isosceles triangles whose base is a side of the square and whose vertex is the
midpoint of the opposite side are drawn. If the area of the octagon determined by the intersection of

these four triangles is .!).. (where).. e N) then find- t~e vaiue of)...
SoluUon.
The octagon is PQRSTUVW
Let O be the centre of PQRSTUVW
0(1/2, 1/2)
P(1/2, 3/4)
R(3/4, 1/2)
=
a is point of intersection of lines 2x + y 2 and x + 2y 2 = 0(2/3, 2/3)
So area of PQRSTUVW = 4 x area of quad. ARQP = 1/24
A L B

Resonances
Educating for btlttr tomorrow
Corporate Office: CG Tower, A-46 & 52, IPIA, Near City Mall, Jhalawar Road, Kola (Raj.)-324005
Website: www.resonance.ac.ln E-mail : contact resonance.ac In
Geometry / ,--------------- _,---

• Exercise-1
PART - I : PREVIOUS ASKED QUESTION FOR PRE RMO
1. A triangle with perimeter 7 has integer side lengths. What is the maximum possible area of such a
triangle?

2. In 6 ABC, we have AC :;: BC = 7 and AB :;: 2. Suppose that D is a point on line AB such that B lies
between A and D and CD :;: 8. What is the length of the i;egment BO?

3. In rectangl.:i ABCD, AB :;: 5 and BC :;: 3. Points F and G are on line .segment CD so that DF :;: 1 and
GC = 2. Lines AF and BG intersect at E. What is the area of 6AEB?

4. ABCD is a square and AB= 1. Equilateral triangle AYB and CXD are drawn such _that X and Y are
inside the square. What is the length of XY?

5. 0 and I are the circumcentre and incentre of 6ABC respectively. Suppose O lies in tho interior of 6ABC
and I lies on the circle passing through B, O and C. What is the magnitude of LBAC in degrees?

6, PS is a line segrn1;nt of length 4 and O is the midpoint of PS. A semicircular arc is drawn with PS as
diameter. Let X be the midpoint of this arc. Q and R are points on the arc PXS such that QR is parallel
to PS and the semicircular arc drawn with QR as diameter is tangent to PS. What is the area of region
QXROQ bounded by the two semicircular arcs?

7. Let AD and BC be the parallel sides of a trapezium ABCD. Let P and Q be the midpoints of the
diagonals AC _and-SD. lf·AD = 16 and BC:;: 20, what is the length of PQ?

8. In a triangle ABC, let H, I and O be the orthocenter, incentre and circumcentre, respectively. If the
points B, H, I, C lie on a cirde, what is the magnitude of I BOC in degrees?

9. Three points X, Y, Z are on a straight line such that XY = 10 and XZ = 3. What is the product of all
possible values of YZ ?

10. Let ABC be an equilateral triangle. Let P and S be point on AB and AC respectively and let Q and R be
point on BC such that PQRS is a rectangle. If PQ = PS and the area of PQRS is , what is the
length of PC?

11. Lei A1, B1, C1, 01 be the midpoints of the sides of a convex quadrilateral ABCD and let A 2, B2 , C2 , 0 2 be
the midpoints of the sides of the quadrilateral•A1B1C1D,. If A 2B2C2 0 2 is a rectangle with sides 4 and 6,
then what is the product of the lengths of the diagonals of ABCD? .

12. Let S be a circle with centre 0. A chord AB, not a diameter. divides S into two regions R1 and R2 such
that O belongs to R2. Let S1 be a circle with centre in R,, touching AB at X and S internally. Let S2 be
circle with centre in R2, touching AB at Y, the circle S internally and passing through the centre of S.
The point X lies on the diameter passing through the centre of S 2 and I YXO = 30°. If the radius of S2 is
100 then what is the radius of S 1?

13. In a triangle ABC with LBCA = 90°, the perpendicular bisector of AB intersects segments AB and AC at
X and Y, respectively. if the ratio of the area of quadrilateral BXYC be the area of triangle ABC Is
13 : 18 and BC:;: 12 then which is the length of AC?

14. Let ABCD be a convex. quadrilateral with perpendicular diagonals. If AB = 20, BC :;: 70 and CD = 90,
then what is the value of DA?

Corporate Office; CG Tower, & 52, IPIA, Near City Mall, Jhalawar Road, Kola (Raj.}-324005
Webslle : www.resonance.ac.ln E-mail: contact resonance.ac.ln ·
Edueatlng for better tomorrow
Toll Free : 1800 258 5555 CIN: U80302RJ2007PLC024029
Geometry / , , - - - - - - - - - - - - - - - - - - - - - - - - - - - - - •

15. In a triangle ABC, X and y are points on the segments AB and AC, respectively, such that AX:XB= 1 : 2
and AY: YC = 2: 1. If the area of triangle AXY is 10 then what is the area of triangle ABC?
'
16 Let ABCD be a convex quadrilateral with LDAB = LBDC = 90°. Let the lncircles of triangle ABO and
BCD touch BO at P and a, respectively with P lying In between B and a. If AD = 999 and PQ = 200,
then what is the sum of the radii of the lncircles of triangles ABO and BOC?

17. Let XOY be a triangle with LXOY = 90°. Let Mand N be the midpoints of legs OX and OY, respectively.
Suppose that XN = 19 and YM = 22. Whal is XY?

18. In a triangle ABC, let I denote the lncentre. Let the lines Al, Bl and Cl intersect the incircle at P, a and
R, respectively. If LBAC = 40°, what Is the value of LQPR in degrees?
A

19. The figure below shows a broken piece of a circi.Jlar plate made of glass.
D

C
A~ B I :::---:::--..
C is midpoint of AB and D is the midpoint of arc AB. Given that AB = 24cm and CD = 6cm, what is the
radius of the plate in centimetre? (The figure is not drawn to scale).

20. A 2 >< 3 recfangle and a 3 >< 4 rectangle are contained within a square without overtapping at any
interior point, and the sides of the square are parallel to the sides of the two given rectangles. What is
the smallest possible area of the square?

21. In rectangle ABCD, AB= 8 and BC= 20. Let P be a point on AD such that L6PC = 90°. If r,, r2 , r3 are
the radii of the incircles of triangles APB, BPC and CPD, what is the value of r, + r2 + r3 ?

22. In acute angled triangle ABC, let D be the foot of the altitude from A and E be the midpoint of BC. Let F
be the midpoint of AC. Suppose LBAE = 40°. If LDAE = LDEF, what is the magnitude of L ADF in
degrees?

23. The circle w touches the circle n internally at P. The centre O of n is outside w. Let XY be a diameter of
n ·which is also tangent to w. Assume PY > PX. Let PY intersect oo at Z. If YZ = 2PZ, what is the
magnitude of LPYX in degrees?

24. Let 6ABC be an equilateral triangle with each side 2-/3. Let P be a point outside the triangle such that
the points A and P lie in the opposite sides of the straight line BC. Let PO, PE, PF be the
perpendiculars dropped on the sides BC, AC and AB respectively where 0, foot of the perpendicular,
lies inside the line segment BC. Let PD= 2. What is the value of PE+ PF.

25. In trapezium PQRS, QR II PS. Let QR= 1001, PS= 2015. Also, let L P= 37° and LS= 53°. Finally, let
X and Y be the midpoints of QR and PS, respectively. Find the length of XY.

26. A square PORS has length of its side equal to 3 + .Js. Let M be the mid-point of the side RS. Also, let
C, be the in-circle oft.PMS and C2 be the circle that touches the sides PQ, QR and PM. Find the radius
of the circle C 2•

Aesonance® Corporate Otnce: CG Tower, A-46 & 52, IPIA, Near City Mall, Jhalawar Road, Kota (Raj.}-324005
Educating for bttter tomorrow t-=W'-:'eb:"::sc-"11=-e'-':www~c:·rc.,,es:;;on:'-:a:'::n'=':c'='-e.:=,ac:-7..l:.;..n..,,E:,.•m:.;.:•::.:11:.,:.:;,;con::::.:.:::ta,,,cl=r~es:::,:o!!!na~nc:::;e~.ac=
.lnc!...__ _
Toll Free : 1800 268 5555 CIN: U80302RJ2007PLC024029
Geometry / , - - - - - - - - - - - - - - - - - - - - - - - - - - - - •

27. Let 6ABC be a triangle with base AB. Let D be the mid-point of AB and P be the mid-point of CD.
Extend AB In both direction. Assuming A to be on the left of B, let X be a point on BA extended further
left such that XA = AD. Similarly, let y be a point on AB extended further right such that BY = BO. Lei
ex cut AC al Q and PY cut BC at R. Lei the sides of MBC be AC= 13, BC= 14 and AB= 15. Whal is
the area of the _pentagon PQABR?

28. - Suppose we wish to cut four e~ual circles from a circular piece of wood whose area is equal to
25n square fl. We want these cirdes (of wood) to be the largest in area that can possible be cut from
the piece of wood. let R ft. be the radius of each of the four new cirdes. Find the integer nearest lo R.

2~. In right-angled triangle ABC with hypotenuse AB, AC = 12, BC= 35. Lei CD be the perpendicular from
C to AB. Let n be the circle having CD as a diameter. Lei I be a point outside MBC such that Al and Bl
are both tangents lo the circle n. Let the ratio of the perimeter of MBI and the length. of Al be min,
where m, n are relatively prime positive integers. Find m + n.

30. Given a rectangle ABCD, detennine two points Kand Lon the sides BC and CD such that the triangles
ABK, AKL and AOL have same area.

31. Two of the Geometry box tools are placed on the table as shown. Determinant the angle LABC.
130•

32. let AD be an altitude in a right triangle ABC with LA = 90° and D on BC: Suppose !hat the radii of the
incirdes of the triangles ABO and ACD are 33 and 56 respectively. Let r be the radius of the incircle of
triangle.ABC. Find the value of 3(r + 7).

33. In triangle ABC right angled at vertex B, a point O is chosen on the side BC such that the circle y
centered at O of radius 08 touches the side AC. Let AB= 63 and BC= 16, and the radius of y be of the
form where m, n are relatively prime positive integers. Find the value of m + n.
n
34. The hexagon OlYMPI has a reflex angle at O and convex at every other vertex. Suppose that
LP = 3.fi. units and the condition ·
=
LO= 10L l = 2LY 5LM = 2LP 10LI =
holds. Find the area (in sq units) of the hexagon.

35. Points G and O denote the centroid and the circumcenter of the triangle ABC. Suppose that
LAGO= 90° and AB= 17, AC= 19. Find the value of BC2•

36. Consider a right-angled triangle ABC with LC = 90°. Suppose that the h~potenuse AB is divided into
=
four equal parts by the points D, E, F.; such that AD DE = EF = FB. If CD + CE2 + CF2 350, find the =
length of AB.
37. Consider a triangle ABC with AB= 13, BC= _14, CA= 15. A line perpendicular to BC divides the interior
of MBC Into two re·gions of equal area. Suppose that the aforesaid perpendicular cuts BC at D, and
cuts MBC again at E. If Lis the length of the line segment DE, find L2•

38. Suppose a circle C of radius .fi. touches the Y-axis at the origin (0, O). A ray of light L, parallel to the
X-axis, reflects on a point P on the circumference of C, and after reflection, the reflected ray
L' becomes parallel to the Y-axis. Find the distance between the ray Land the X-axis.

nesonance® Corporate Office: CG Tower, A-<16 & 52, IPIA, Near City Mall, Jhalawar Roarl. l<ola (Rllj.),J:.'4'lO!i
M Webstte : www.resonance.ac.ln E-mail : contact resonance.ac.in
PRERM0-67
Educating for bttter tomorrow r-:.-Tol::;;1-;:F:-:,e:-:-.,-:--
: :;-;,e;;;o;;-o;:;;2s:-;11-;:57.55:--::5'7'-;;;:c::-clN,-'cu;': ;';e;;:::o:=:30;;:::2;;::Rc;:;J2='=01:'::!P~LC'=o='=2c:'40:c::2~9=""-'---
0'='
Geomelly / , - - - - - - - - - - - - - - - - - - - - - - - - - - - - - - - - . •

39. In a rectangle ABCD, E Is the midpoint of AB , F Is point on AC such that BF Is perpendicular to AC and
FE perpendicular to BO. Suppose BC = Find AB.

40. Let P be an Interior point of a triangle ABC whose side lengths are 26, 65, 78. The line through P
parallel to BC meets AB in K and AC in L. The line through P parallel to CA meets BC In M and BA
In N. The line through P parallel to AB meets CA In S and CB In T. If Kl, MN, ST are of equal
lengths, find this common length.

41, Let ABCD be a rectangle and let E and F be points on CD and BC respectively such that area
(ADE)= 16, area (CEF) = 9 and area (ABF):: 25. What is the area of triangle AEF?

42. Let AB and CD be two parallel chords in a cirde with radius 5 such that the centre O lies between these
chords. Suppose AB = 6, CD = 8. Suppose further that the area of the part of the circle lying between
the chords AB and CD Is (mn + n) / k, where m, n, k are positive Integers with gcd(m, n, k) = 1. What is
the value of m + n + k 7

43. Let o, be a circle with centre O and let AB be diameter of 0 1 • Let P be a point on the segment OB
different from 0. Suppose another circle 0 2 with centre P lies in the interior of 0 1• Tangents are
drawn from A and B to the circle 0 2 Intersecting 0 1 again at A 1 and B 1 respectively such that A 1 and
=
B, are on the opposite sides of AB. Given that A1 B 5, AB 1 15 and OP= 10, find the radius of 0 1• =
44. Consider the areas of the four triangles obtained by drawing the diagonals AC and BO of a
trapezium ABCD. The product of these areas, taken two at time, are computed. If among the six
products so obtained, two product are 1296 and 576, determine the square root of the maximum
possible area of the trapezium to the nearest Integer.

45. In a quadrilateral ABCD, It is given that AB =. AD = 13, BC = CD = 20, BO = 24. If r is ttie radius of the
circle inscribable in the quadrilateral, then what is the Integer closest tor 7

46. Lei ABCD be a trapezium In which AB II CD and AD .l AB. Suppose ABCO has an incirde which
touches AB at Q and CD at P. Given that PC 36 and QB 49,' Find PQ. = =
47. A point pin the interior of a regular hexagon is at distance 8,8.16 units from three consecutive vertices
of the hexagon, respectively. If r is radius of the c_lrcumscribed circle of the hexagon, what is the integer
closest to r ?

48. C
Let AB be a chord of circle with centre 0 . Let be a' point on the cirde such that L ABC = 30" and 0
lies inside the triangle ABC. Let D be a point on AB such that L DCO L DCB 20•. Find the = =
measure of L COO in degrees.

49. In a triangle ABC, the median from B to CA is perpendicular to the median from C to AB. If the median
2 2 2
from A lo BC Is 30, determine (BC + CA + AB )/100.

50. In a triangle ABC, right-angled al A, the altitude through A and the Internal bisector of L A have lengths
3 and 4, respectively. Find the length of the median through A.

nesonance® Corporate Ornce: CG Tower, A-46 & 52, IPIA, Near City Mill, Jhllawar Road, Kola (RaJ.}-324005
Website : www.resonance.ac.ln E-mail : contact
I'"'(
EducaUng for better tomorrow : 1'-ec:" .:oo-"'-2=-=6c=c8c:.: 5::.;;
-T01-=-1:;.:F""re~e"" : u=-e..:..:0=30""'2:..:RJc=c2~00==7"'p""Lc-=o=•o:::.29==.:.,.._--
55=-=5=c1""N'-'-

----
Geometry / - - - - - - - - - - - , -_-
,-

PART - II : PREVIOUSLY ASKED QUESTION OF RMO

.. 1, Let ABC be a right angled triangle with LB= 90° and let BO be the altitude from B on to AC. Draw
DE .l AB and OF .l BC. Lei P, Q, Rand S be respectively the lncenler of triangle DFC, DBF, DEB
and DAE. Suppose S, R, Qare collinear. Prove that P, Q, R, D be on a circle. [RMO-2015)

2. Let ABC be a right-angled triangle with LB ;; go•. Let I be the lncentre of ABC . Draw a line
perpendicular to AI at I. Let It Intersect the line CB at D. Prove that Cl Is perpendicular to AD and prove
that ID;; .jb(b- a) where BC ;; a and CA;; b [RMO-2016) .

3. Let ABC be a triangle with centroid G. Lei the circumclrcie of triangle AGB Intersect the line BC in X
different from B; and the circumcircie of triangle AGC intersect the line BC In Y different from C. Prove
that G is the centroid of triangle AXY. (RM0-2016)

4. Let t,, l2, l3, •••• •• i40 be fourty parallel lines. As shown in the diagram, let m be another line that Intersects
the lines t, to /40 In the points A1, A2, A3, ••••••• ~o respectively. Similar1y, let n be another line that Intersects
the lines t , to t40 in the points B1, B2, B3, .................................... 840 respectively.
Given that A,B, = 1, ~0B40;; 14, and the areas of the 39 trapeziums A 18 1 B2A 2 • A 2 8 2 83A3 ........ AJgB39840
~o are all equal ; then count the number of segments A, B whose length is a positive integer;
where i E {1 , 2,
........................40}

[RMO-2017]

5. Lett.ABC be acute - angled; and let r be its circumcircle, let D be a point on minor arc BC of r . Let
E and F be points on lines AD and AC respectively, such that BE .L AD and OF 1. AC. Prove that EFII
BC if and only if Dis the midpoint arc BC. · [RMO-2017]

Aesonance® Corporate Office: CG Tower. A-46 & 52, IPIA. Near City Mall. Jhalawar Road, Kola (Raj.)-324005
Educadng fo, bttltf tomorrow wwwtti,;;;'.ret1~on"taiinc
r;:;W:;ieb"j:s;;tte;;;-:-:- °'. ~.lnc!....._ _
:ei .ac~.;;:;ln~E-;;:;m:;;a;';;II:;;'c~on~ta~c~t::-;r~et1~on7.a~n~ce~.ac
Toti Free : 1800 258 55557 CIN: U80302RJ2007PLC024029
Answers 2
• Exercise-1
PART- I

om
1. 2. 3 units 3. 12.5 4. ../3-1 5. 60"
4

t.c
3n-4
6. 10. 14

po
7. 2 8. 120· 9. 91
2

gs
11. 208 12. 60 13. 36 14. 60 15. 45

lo
16 799 17. 26 18. 55 19. 15 20. 25

21. 8 22. 40 23. 15


e.b 24. 5 25. 507
or
lst
26. 2 27. 56 28. 2 29. 12753

JBC units from Band Lis (


1 1
ia

30. K is ( - - ) DA units from D


fic

31. 10 32. 216 33. 71


of

34. 9 35. 325 36.20 37. 112 38. 1


er
rn

39. 24 40.Bonus 41.30 42. 75 43.20


co

44. 13 45. (8) 46. (84) 47. (14) 48. 80


ok

49. 24 50.24
o
.b
w
w
w

Aesonance® Corporale Otnce; CG Tower, A--46 & 52, IPIA, Near Clly Mall, Jhalawar Road, Kola (Raj.)-324005
Webslle : www.resonance.ac.ln E-mail ; conlacl resonance.ac.ln
EducaUng for better tomorrow !!!..--
t"T:ro1:ti1"t.Fr;;;:ee:"::-':;1i'ieoo~2sa~siiss~s'f;C~IN;i::~uiieo;-;3;;';02;;:;R'7-J2;;;'00~1~PL;';C~o~24f-Eo2:!:9~<:.
Combinatorics
-,•
Combinatorics/,---------------------------

Icombinatorics I
Factorial notation : ln_or nl is pronounced as factorial n and is defined as
n(n -1)(n - 2) ....... 3 x 2 x 1 if n e N
n• = { ;ifn=O
. 1
Note : n! = n . (n - 1)! ; ne N
Stateme nt of blnomlal theorem :
(a+ b)" = "Co a•b + •c, an- b' + •c, an-2 b2 + ...+ •c, an-r b' + ..... +
0 1 nc. a 0 b"
where n e N
n
or (a+ b)• = L "Cr a"-<br
r•O
General term :
(x + y)" = nco X" y0 + •c, xn-1 Y' + .......... + •c, x~ Y + ..........+•c. x
0 Y'
(r + 1)"" term is called general term and denoted by T,., .
T,., = nC, x~ y

r
Middle term(s) :

(a) If n is even, there is only one middle term, which is( n; 2 term.

(b) If n is odd, there are two middle terms, which are ( n;


1
r and ( n ;1 + 1r terms.

10

Example # 1 : Expand the binomial ( +x J up to four terms


Solution: (;+x r rnr 1
= °C0
1
+ °C1 rnJ X + °C2
1
rnJ'x, + 1°C3 rnJ x3 + ...

Example # 2 : The number of dissimilar terms in the expansion of (1 + x• - 2x 2


) 15 is
(A) 21 (8)31 (C) 41 (D) 61
Solution : (1 - x 2) 30
Therefore number of dissimilar terms= 31.

Example # 3 : Find (I) 15"' term of (2x - Jy)


20
(ii) 4"' ~erm of ( :x _ y J
(I) T,.., = C,. (2x)' (-Jy)" = C,. 2 3" x•.y"
20 20 1
Solution :

(ii) TJ., =,c3(3sx )' (-y)3 =-1cl(¾J x•y3

Example # 4: Find the number of rational terms in the expansion or( 2i + Ji roo
Solution: The general term in the expansion or( 2"i +Ji roo is
T,., = eooc, ( 2i ro-, ( J J~ = 600C, 2
6
~_, i
Resonance® Co,porate Office: CG Tower, A--46 & 52, IPIA. Near City Mall, Jhalawar Road, Kota (Raj.}-32400
5

I EducaUng for better tomorrow IT-To1;:;;1.=F:;:re=e


Websile 7 : www.resona nce.ac.in
: .-.,s;-;;00~25~8~5,;;-55~5~C~IN-;:'
E•mail : contact resonance.a
: ';';U;;;80;:;3~02~Ri';J;:;:20~0'27Pi5L"=cc:'o2~4===0~29~=!
.............................................................................................................................................................................................
c.in
..........................................._-
Combinatorics /r------.----------------------m-

The above term will be rational if exponent of 3 and 2 are integers


600-r r .
It means - - and - must be integers.
3 5
The possible set of values of r is {0, 15, 30,45 ............,600}
Hence, number of rational terms is 41
Example#S: Find the middle term(s) in the expansion of
2)11
(I) (1 + 2x)12 (Ii) ( 2y-y2
Solution: (i) (1 + 2x)12
Here, n is even, therefore midcle term is (1 2; 2}' term.

It means T, is middle te:.n T, = 12c1 (2x)'


y2)11
(II)
( 2y -2
Here, n is odd th~~!fore, midclle terms are (1 12+1}' (1 1; 1 + 1)-,.
&

It means T, & T7 are middle terms

T, = "C5 (2y)' (- J = -2 "C5 y" T7 = " C, (2y)5 ( - J =


11

~
6
y
17

Self practice problems

(1) Write the first three terms in the expansion of ( 2-f)' .

(2) Expand the binomial [ +-;J


(3) Find term which is independent of

80
Xin ( x2- X: r
Ans. (1) 64-64y + - y2 (2)
3
(3) T5 is independent of x.

Ftndln& Remainder :
Example# 6 : Show that 7" + 5 is divisible by 6, where n is a positive integer.
Solution : 7" + 5 = (1 + 6)" + 5 = -CO + -C1 • 6 + -C, . 62 + ....... + -c. 6" + 5.
= 6. c1 + &2. c, + ....... + c•. 6" + 6.
= 61, where A is a positive integer
Hence, 7" + 5 is divisible by 6.
Example # 7 : What is the remainder when 7" is divided by 5.
11
7 = 7.7"' = 7. (49)"' = 7 (50-1)"0
Solution: = 7 [<IOCO (50)"0- <10C1 (50)39 + .......... - '°Cu (50)' + -oc.. (50) 0
]

= 5(k) + 7(where k is a positive integer)= 5 (k + 1) + 2


Hence, remainder is 2.
Example# I : Find the last digit of the number (13)'2•
Solution : (13)' 2 = (169)1 = (170-1)5
= •c. (170)' - •c, (170)5 + ......... - •c, (170)' + •c, (170)0
Hence, last digit is 1

Resonance' Cc,porate Office: CG Tower, A-Ci l 52. IPIA, Near City Mall, Jhalawar Road. Kola (Raj.}-324005
Website: www.resonance.ac.ln E-mail :
Educating 1w Mtllr llfflorTVW : 1;';;8-:::00::'-:2~58~555':'2-5""c~1N':--:~u'-='a03=02='=R;:;J20=01;!:P~L~C02~40~2~9=::!!!...--
t-T:;'ol'7,l=;Fr"'ee~
Combinatorics/
________.._ _,-----

Example-9 : Which number is larger (1.1 )100000 or 10,000 ?


Solutlon : By Binomial Theorem
(1.1 )100000 = (1 + 0.1 )100000 = 1 + 100000c 1 (0.1) + other positive terms
= 1 + 100000 >e 0.1 + other positive terms
= 1 + 10000 + other positive terms
Hence (1.1 ) 100000 > 10,000
Self practice problems :
(1) if n is a positive integer, then show that 6"- 5n - 1 is divisible by 25.
(2) What is the remainder when 3257 is divided by 80 .
5
(3) Find the last digit, last two digits and last three digits of the number (81 )2 ·

(4) Which number Is larger (1.3)2""" or 600


Ana. (2) 3 (3) 1, 01, 001 (4) (1 .3)2000.

Blnomlal theorem for negative and fractlonal Indices :


•_ n(n-1) · n(n-1)(n-2) 3
If n e R, then (1 + x) - 1 + nx + - - x2 + . x + ................
2 3 !
n(n-1)(n-2) ...... (n-r+1) , + • ·
................ + X .•••••••••••••.••••• a:>.
r !
Remarks
(I) The above expansion is valid for any rational number other than a whole number if Ix I< 1.
(ii) When the index is a negative integer or a fraction then number of terms in the expansion of
· (1 + x)• is infinite, and the symbol -C, cannot be used to denote the coefficient of the general
tenn.
(iii) The first tenn must be unity in the expansion, when index 'n' is a negative integer or fraction
1
x"(1+iJ ~x"{1+n. f+n ;n~ ' ( i ) \ ...... } if l i l < 1

(x + y)• =
2

y"(1 + y
x)" = Y"{1+n ._yx+ n 2(n-1)(x)
! y +.... ) If
· Ixy I< 1
The general term in the expansion of (1 + x)• is T = n(n- 1)(n - 2) .........(n - r + 1) x'
(iv) r !

(v) When 'n' is any rational number other than whole number then approximate value of (1 + x)•
is 1 + nx (x2 and higher powers of x can be neglected)
(vi) Expansions to be remembered (lxl < 1)
(a) (1 + x)-1 = 1-x+x2 -x3 + ...:...... +(-1Yx'+ .........a:>
(b}
(c)
=
(1 -x)-1 = 1 + X + X2 +2x3 .t 3......... + X' + .........IX>
1 + 2x + 3x + -4x + ........... + (r + 1)x' + ........... a:>
(1 + x)-2 = 1-2x + 3x2 -4x3 + .......... + (-1Y (r+ 1) x' + ...........00
Example# 10: (d) (1 -the
Prove that
2
x}-coefficient of X' in (1 - x)-" is --1 C,
Solution:

(r + 1 )" term in the expansion of (1 - x)..., can be written as


_ -n(-n-1)(-n-2) .....(-n-r+1) ( )
T 1 - -"---..:..:.._ _:._....:....__....:. -x'
" r I
= (-1)' n(n + 1)(n + 2) ..... (n +r -1) (-xY = n(n + 1)(n + 2)...... (n +r -1) X'
r I r I

= (n - 1)! n(n + 1) ...... (n + r - 1> x' Hence, coefficient of x' is (n + r - 1)! = --•c Proved
(n -1) ·, r I (n -1)1 r I '

Resonance® Corporate OIOce: CG Tower. A-48 & 52. IPIA. Near City Mall, Jhalawar Road. Kota (Raj.)-324005
Website : www.resonance.ac.ln E-mail : contact resonance.ac.ln
Educating for better tomorrow 1-:T;':ol:;;l~Fr°"e'-'e-"
: 1='=e""ooi:-2~s,'fe~s;;ss:'=s~c1~N~
: u=-s~0.!!!30~2~R~J2!.!!0~07::!lP~L!.,!!C0~2~40~2~9~:::!!!.--'~
Combinatorics /r-----------------------------•
Example-11 : If x is so small such that its square and higher powers may be neglected, then find the value of
(1- 2x)"3 + (1 + 5xf312
(9 + x)" 2
2 15x
Solutlon:
(1 - 2x)113 + (1 + sxi-312 1-3 X+ 1-2 - 1 (
-"""----=--- 2 - - x
49 ) ( X)-112
1+-
(9 + x)112 x)112 3 6 9
3(1+ -
9

=-31 (2- 469 x) 18 2 9 6


!
(1-.!..)= (2-~- 49 x) = 1- .!.__ 49 x = 1 - .149x
18 12 ':I~

Self practice problems :


(1) Find the possible set of values of x for which expansion of (3 - 2x)"2 is valid in ascending
powers of x.
2 3
(2) Find the coefficient of x50 in - x
. (1-x)3

Ans. (1) (2) -1173

Meaning of Permutation & Combination :

Fundamental counting principle :


Suppose that an operatio". 0, can be done in m different ways and another operation 0 2 can be done in
n different ways.
(i) Addition rule : The number of ways in which we can do exactly one of the operations 0,, 0 2
ism+ n
(ii) Multiplicatlon rule : The number of ways in which we can do both the operations 0,, 0 2 is mn.
Note : The addition rule is true only when O, & 0 2 are mutually exdusive and multiplication rule is true only
when o, & 0 2 are independent (The reader will understand the concepts of mutual exclusiveness and
independence, in the due course)

Example# 12: There are 8 buses running from Kota to Jaipur and 10 buses running from Jaipur to Delhi. In
how many ways a person can travel from Kola to Delhi via Jaipur by bus?
Solution : Let E, be the event of travelling from Kola to Jaipur & E2 be the event of travelling from Jaipur
to ,
Delhi by the person.
E, can happen in 8 ways and E2 can happen in 10 ways.
Since both the events E, and E2 ·are to be happened in order, simultaneously,
the number of ways = 8 " Hf= 80.

Example# 13 : How many numbers between ·10 and 10,000 can be formed by using the digits 1, 2, 3, 4, 5 if
(i) No digit is repeated in any number. (ii) Digits can be repeated.

Solution: (i) Number of two digit numbers= 5" 4 = 20


Number of three digit numbers = 5 " -4 " 3 = 60
Number of four digit numbers = 5 " 4 " 3 " 2 = 120
Total= 200
(ii) Number of two digit numbers::: 5 " 5 = 25
Number of three digit numbers= 5" 5" 5 = 125
Number of four digit numbers ::: 5 " 5 " 5 " 5 = 625
Total= 775

nesonance® Corporate Office: CG Tower, A-46 & 52, IPIA, Near City Mall, Jhalawar Road, Kola (Raj.)-324005
r""t Website : www.resonance.ac.ln E-mail : contact resonance.ac.in
Educating for better tomorrow i--,T~ol:;;l";;F:::.re"-e'-':'::::,a:;oc;:'0~2~58;:-:5;;5~55~Ca':INc:-:.1cU~8~
03~0~2~R~J2~0~07:,!!Pl!iL~C~02~4~02~9==!..._-
Combinatorics/
_________.._
ra,--
Self Practice Problems :
(1) How many 4 digit numbers are there, without repetition of digits, if each number is divisible
by 5?
(2) Using 6 different flags, how many different signals can be made by using atleast three flags,
arranging one above the other?
Ans. (1) 952 (2) 1920

Arrangements :
If •P, denotes the number of permutations (arrangements) of n different things, taking rat a time. then

•P, = n (n - 1) (n - 2) ..... (n - r + 1) =
. (n-r)!

Example# 14: How many three digit can be formed using the digits 1, 2, 3, 4, 5, without repetition of digits?
How many of these are even?
Solution : Three places are to be filled with 5 different objects.
Number of ways= 5 P 3 = 5 x -4 x 3 = 60
For the 2nd part, unit digit·can be filled in two ways & the remaining two digits can be filled
in 'P2 ways.
Number of even numbers= 2 >< 'P2 = 2-4.
Example# 15 : if all the letters of the word 'QUEST' are arranged in all possible ways and put in dictionary
order, then find the rank of the given word. ·
Solution: Number of words beginning with E = 'P, = 2-4 ··
Number of words beginning with QE = 3P3 = 6
Number of words beginning with QS = 6
Number of words beginning withQT = 6.
Next word is 'QUEST'
its rank is 24 + 6 + 6 + 6 + 1 = 43.

Self Practice Problems :


(1) Find the sum of ail four digit numbers (without repetition of digits) formed using the digits
1, 2, 3, 4; 5.
(2) Six horses take part in a race. In how many ways can these horses come in the first, second
and third place, if a particular horse is among the three winners (Assume No Ties)?
(3) Find the sum of all three digit numbers those can be formed by using the digits. 0, 1, 2, 3, 4.

Ans. (1) 399960 (2) 60 (3) 27200

Combination :
If "C, denotes the number of combinations (selections) of n different things taken rat a time, then
I np
"C = n · = - ' where r n ; n e N and re W .
' r! (n-r)I r!
NOTE : (i) "C, = •c._,
(ii) •c, + •C,_, = • • 1C,
(Iii) •C, = 0 if r t!! {0, 1, 2, 3 ........, n}

nesonance® Corporate Office: CG Tower, A--16 & 52, IPIA. Near City Mall. Jhalewftf Rood, Kot" (Rnj.)-3~,1005
r,; Website : www.resonance.ac.ln E-mail : contacl resonance.ac.ln
EducaUng for better tomorrow ~To1'""1==F"'re=-e':"'1
-' e=-'o~o'-=2'='se:;:;'='55:':5'='5'-=c'='1'i-'N:_.u=-a,:;oc:,:30:;.:,2,:,;R::.::J:;..:20:.::;o.;:7P~La:.;C::::0:;,24:.:.:0::;2:;:_:::;9="----1
-•
Combinatorics/,----------------------------

Example # 16 : There are fifteen players for a cricket match.


(i) In how many ways the 11 players can be selected?
(ii) In how many ways the 11 players can be selected including a particular player?
i i)
(i_ In how many ways the 11 players can be selected excluding two particular players?

Solutlon: (i) 11 players are to be selected from 15


Number of ways= ' 5C,, = 1365.
(ii) Since one player is already included, we have to select 10 from the remaining
14
Number of ways= "C, 0 = 1001 .
13.
(iii) Since two players are to be excluded, we have to select 11 from the remaining
Number of ways = "C,, = 78.

by using the vertices, but


Example # 17: A regular polygon has 20 sidef How many triangles can be drawn
not using the sides?
selected from 17
Solution : The first vertex can be selected in 20 ways. The remaining two are to be
17C - 16 ways.
vertices so that they are not consecutiv e. This can be done in 2

The total number of ways= 20 >< ("C2 - 16)


But in this method, each selection is repeated thrice.
20x(1 7 c 2 -16)
Number of triangles= = 800.
3

three of them if adjacent


Example # 18: 15 persons are sitting in a row. In how many ways we can select
persons are not selected ?
in this order.
Solution : Let P,, P 2 , P 3 , P,, P 5 , P6 , P7 , P,. P9 , P, 0 ,P,,, P,2 , P, 3 ,P,. ,P,5 be the persons sitting
If three are selected (non consecutive) theri 12 are left out.
of ways in
Let P,P,P,P,P ,P,P,P,P,P ,P,P be the left out & q, q, q be the selected. The number
can be placed into the 13 positions between the P's (including extremes) is
which these 3 q's
the number ways of required selection.
Thus number of ways = C3 = 286.
13

MISSISSI PPI?
Example # 19: In how many ways we can select 4 letters from the letters of the word
Solution : M
I II I
ssss
pp .
2
Number of ways of selecting 4 alike letters = C, = 2.
Number of ways of selecting 3 alike and 1 different letters =
2 x 3C, = 6 c,
3
Number of ways of selecting 2 alike and 2 alike letters = , C 2 = 3
Number of ways of selecting 2 alike & 2 different= C, x C2 = 9
3 3

Number of ways of selecting 4 different = = 1 •c,


Total number of ways= 2 + 6 + 3 + 9 + 1 = 21

Self Practice Problems : •


In how many ways 7 persons can be .selected from among 5 Indian, 4 British
& 2 Chinese, If
(1)
atleast two are to be selected from each country ?

In how many ways 6 boys & 6 girls can sit at a round table so that girls & boys
sit alternate?
(2)
chairs?
(3) In how many ways 4 persons can occupy 10 chairs in a row, If no two sit on adjacent
(4) In how many ways we can select 3 letters of the word PROPOR TION ?

Ans. (1) 100 (2) 86400 (3) 840 (4) 36

nesonance® Corporate Omce: CG Tower, A-46 & 52, IPIA, Near City Mall, Jhalawar Road, Kota (RaJ.)-324005
Website : www.resonance.ac.ln E-mail: contact resonance.ac.ln ~=!...,_-
M o ~25~B;";:5:';:-5;;';55~C~INT.':';';U~80;;;;3;;;'02;;;R;';J~20~0~7Piel~Cc:'02='4='-=o'=:29
~To1:T.1=i:F':':re"='e:':-'7-1e;-;;o;;-
Educatlng for better tomorrow
Combinatorics /r-----------------------------i•
Arrangement of n things, those are not all different :
The number of permutations of 'n' things, taken all at a time, when 'p' of them are same & of one type, q
of them are same & of second type, 'r' of them are same & of a third type & the remaining
nI
n - (p + q + r) things are all different, i s - - --
p ! q !r !

Example# 20; In how many ways we can arrange 3 red flowers, 4 yellow flowers and 5 white flowers in a row?
In how many ways this is possible if the white flowers are to be separated in any arrangement?
. . (Flowers of same colour are identical).
Solution.: Total we have 12 flowers 3 red, 4 yellow and 5 white.
12 I
Number of arrangements = · =
27720.
3 ! 4 ! 5 !
For the second part, first arrange 3 red & 4 yellow
. 7 I
Th rs can be done in - - · - = 35 ways
3 I 4 !
Now select 5 places from among places (including extremes) & put the white flowers there.
This can be done in 8 C5 = 56.
The number of ways for the 2"" part= 35 x 56 = 1960.

Example# 21 : In how many ways the letters of the word "ARRANGE" can be arranged without altering the
relative positiOl)S of vowels & consonants?
41
Solution: The consonants in their positions can be arranged in = 12 ways.
2!
The vowels in their positions can be ·arranged in ~: = 3 ·ways

Total number of arrangements= 12 x 3 = 36

Self Practice Problems ;


(1) How many words can be formed using the letters of the word ASSESSMENT if each word
begin with A and end with T?

(2) If all the letters of the word ARRANGE are arranged in all possible ways, in how many of words
we will have the A's not together and also.the R's not together?
Ans. (1) 840 (2) 660

Selectlon of one or more objects


(a) Number of ways in which atleast one object may be selected out of 'n' distinct objects, is
•c1 + •c2 + "C3 + ............... + •c. = 2• -1
(b) Number of ways in which atleast one object may be selected out of 'p' alike objects of one
type, 'q' alike objects of second type and 'r' alike objects of third type, is
(p+1)(q+1)(r+1)-1 ·
(c) Number of ways in which atleast one object may be selected from 'n' objects where 'p' alike of one
type, 'q' alike of second type and 'r' alike of third type and rest n - (p + q + r) are different,
is (p + 1) (q + 1) (r + 1) 2•- c•••">-1

Example# 22: There are 12 different books in a shelf. In how many ways we can select atleast one of them?
Solution: We may select 1 book, 2 books, ........ , 12 books.
: . The number of ways = 12C 1 + 12C2 + ....... + 12C 12 = 212 - 1. = 4095

Example# 23: There are 11 fruits in a basket of which 6 are apples, 3 mangoes and 2 bananas (fruits of same
species are identical). How many ways are there to select atleast one fruit?
Solutlon : Let x be the number of apples being selected
y be the number of mangoes being selected an_d
n $ a ® Corporate Otnce: CG Tower. A-46 & 52. IPIA, Near City Mall, Jhalawar Road. Kota (Raj.}-324005
"e on
Educating better
for
nee Website : www.resonance.ac.ln E-mail : contact resonance.ac.ln
tomorrow · Toll Free : 1800 258 5555 CIN: U80302RJ2007PLC024029
Combinatorics/,-----------------------------•

z be the number of bananas being selected.


Then · x = 0, 1, 2, 3, 4, 5,6
y=0,1,2,3
z=0,1,2
Total number of triplets (x, y, z) is 7 )( 4 )( 3 = 84
Exclude (0, 0, 0) '
Number of combinations = 84 - 1 = 83.
Self Practice Problems

(1) In a shelf there are 6 physics, 4 chemistry and 3 mathematics books. How many combinations
are there if (i) books of same subject are different? (ii) books of same subject are identical?

(2) From 5 apples, 4 mangoes & 3 bananas, in how many ways we can select atleast two fruits of
each variety if fruits of same species are identical ?
Ans. (1) (i) 8191 (ii) 139 (2) 24

Clrcular Permutation :
The number of circular permutations of n different things taken all at a time is (n - 1)!.
1
If clockwise & anti-clockwise circular permuiations ~re considered to be same, then it is (n - )! .
2

Note : Number of circular permutations of n things when p are alike and the rest are different, taken all at a
1
time, distinguishing clockwise and anticlockwise arrangement is (n - >! .
. p!

Example# 24. In how many ways can we arrange 6 different flowers in a circle? In how many ways we
can form a garland using these flowers?
Solution: The number of circular arrangements of 6 different flowers = (6 - 1)! = 120
When we form a garland, clockwise and anticlockwise arrangements are similar. Therefore, the

number nf ways of forming garland = i (6 - 1) ! = 60.

Example#25 In how many ways 6 persons can sit at a round table, if two of them prefer to sit together?
Solution: Let P1 , P2 , P3 , P 4 , P 5 , P6 be the persons, where P1 , P 2 want to sit together.
Regard these person as. 5 objects. They can be arranged in a cirde in (5 - 1)! = 24 ways. Now
p , p can be arranged in 2! ways. Thus the total number of ways = 24 x 2 = 48.
1 2

Self Practice Problems :


(1) In how many ways letters of the word 'MONDAY' can be written around a circle. if vowels
are .to be separated in any arrangement ?

(2) In how many ways we can form a gar1and using 3 different red flowers,5 different yellow flowers
and 4 different blue flowers, if flowers of same colour must be together?
Ans. (1) 72 (2) 17280

Method of flctlous partition :


Number of ways in which n identical things may be distributed among p persons if each person may
receive none, one or more things is ....,c•.

Example# 26 : Find the number of solutions of the equation x + y + z = 6, where x, y, z e W.


Solution : Number of solutions= coefficient of x• in (1 + x + x2 + ....... x8 ) 3
= coefficient of x• In (1 - x1 )3 (1 - x)..,,
= coefficient of x' in (1 - x)-3
= 3 ~•c, =8 C2 =28.

nesonance® C0<porale Office: CG Tower, A-46 & 52. IPIA. Near Clly Mall. Jhalawar Road. Kola (Raj.)-324005
r"{ Websila : www.resonanca.ac.ln E-mail: contact resonanca.ac.ln
Educating for better tomorrow -::1:-::F=-
T o1
1-= : u.=.a"'o.c.c30"'"2'"'R=-=J2"'-0""0"'1p"'L"'c"'o2.=:4.c.co""29==.:._-
re~e-:..,.1a::-:o:-=o:-::2-=sa::-::-:ss"'s"""s',--c"'1""N-'-
Combinatorics/,-----------------------------~•

Example# 27 : In a bakery four types of biscuits are available. In how many ways a person can buy 10
biscuits if he decide to take atleast one biscuit of each variety?
Solution : Let the person select x biscuits from first variety, y from the second, z from the third and w
from the fourth variety. Then the number of ways = number of solutions of the equation
X + y + Z + W = 10.
where x = 1, 2, ......... ,7
y= 1, 2, ......... ,7
z=1,2, ........ ,7
w=1,2, .........,7
So, number of ways = coefficient of x• 0 in (x + x 2 + ..... + x1 )•
= coefficient of x' in (1 + x + ...... + x1 )•
= coefficient ofx' in (1 - x7 )' (1 - x,.
= coefficient x• in (1 -
= •-•c, •c
= 3 = 84
x,.
Self Practice Problems:
(1) Three distinguishable dice are rolled. In how many ways we can get a total 15?
(2) In how many ways we can give 5 apples, 4 mangoes and 3 oranges (fruits of same species are
similar) to three persons if each may receive none, one or more?
Ans. (1) 10 (2) 3-150

Formation of Groups :
Number of ways in which (m + n + p) different thin,9s can be divided into three different groups
. h. (m+n+p) !
containing m, n & pt mgs respectively i s - - - - ,
m !n !p !
If m = n = p and the groups have identical qualitative characteristic then the number of groups
= (3n)! •
n! nl n! 3!
3
Note : If 3n different things are to be distributed equally among three people then the number of ways= < n~ .
. (n!)
Example #28 : 12 different toys are to be distributed to three children equally. In how many ways this can be
done?
Solution: The problem is·to divide 12 different things into three different groups.
121
Number of ways= = 34650.
41 41 41

Example# 29 : In how many ways 10 persons can be.divided into 5 pairs?


Solution: We have each group having2 persons and the qualitative characteristic are same (Since there
is no purpose mentioned or names for each pair).
. . 101
Thus the number of ways = - - - = 945.
(2!)5 5!

Self Practice Problems :


(1) 9 persons enter a lift from ground floor of a building which stops in 10 floors (excluding ground
floor), if it is known that persons will leave the lift in groups of 2, 3, & 4 in different floors. In how
many ways this can happen?
(2) In how many ways one can make four equal heaps using a pack of 52 playing cards?
Ans. 521
( 1) 907200 (2)
(131)4 41

Resonance® Corporate Office: CG Tower, A-48 & 52, IPIA, Near City Mail, Jhatawar Road, Kota (Raj.}-324005
Website : www.ras011anca.ac.ln E-mail : contact res011ance.ac.in
Educating for better lomorrow r-T~ol:;;l:.F::-:re='=e"""'.":':"18;;-;00;:::'-;::2::::58="'5;:':55~57'.C::-:l~N""
: u:':':e::':0""30:':2c.::Rc-cJ2:':oc=o:c1P~L"=c:-:'02~4:=.;0'='29;::==.:-
Combinatorics /r----------------------------1!-

Inclusion Exclusion Principle:


If A, B, C are finite sets and U be the finite universal set then
(i) n(A v B) = n(A) + n(B)- n(A n B)
(II) n(A v B v C) = n(A) + n(B) + n(C)- n(A n 8)- n(B n C) - n(A n C) + n(A n B n C)
AJ ... A,, are finite sets then
If A1, A2,
1
n(A1 v A2 v ... v A,,)= :E n(A;) - :En(A; n A;)+ :En(A; n A; n At) - .... (-1 ),,_ n (A, n A2 n A3 .... n A,,)
Example# 30 : Amongst first hundred natural numbers how many are divisible by 2, 3 or 5
Solution: n(A v B v C) = n(A) + n(B) + n{C)- n{A n 6)- n(B n C)- n(A n C) + n(A n B n C)
= 50 + 33 + 20-16- 6-10 + 3 = 74
Self Practice Problems :
(1) A,A,B,B,C,C,D,E,F are arranged in a row so that no two alike alphabets are together. Find
number of such arrangement
Ans. ' (1) 21960 .
Derrangements : '
Number of ways in which 'n' letters can be put in 'n' corresponding envelopes such that no letter goes
to
. (
correct envelope Is n! 1- l 1
+ -
1 1 2 1 3
1 1
< 1
4 1
-·:········+(-1)" n
1
1)

Example# 31 :· In how many ways we can put 5 writings into 5 corresponding envelopes so that no writing go
to the corresponding envelope?
Solution : The problem is the number of dearragements of 5 digits.
1 1 1
This is equal to 5! ( -- - - - + - - - -·1-) = 44.
2 I 3 ! 4 ! 5 !

ExamplP. # 32 : Four slip of papers with the numbers 1, 2, 3, 4 written on them are put in a box. They are drawn
· one by one (without replacement) at random. In how many ways it can happen that the ordinal
number of atleast one slip coincide with its own number?
Solution: · Total number of ways= 4 ! = 24.
The number of ways in which ordinal number of any slip does not coincide with its own number
1 1 1
is the number of dearrangements of 4 objects = 4 ! ( _ - - --+-
l2 ! 3 ! 4 I
-IJ ,, 9
Thus the required number 9f ways. = 24 - 9 = 15

Self Practice Problems:

(1) In a match the column question, Column I contain 10 questions and Column II contain 10
answers written in some arbitrary order. In how many ways a student can answer this question
so that exactly 6 of his matching are correct ?
(2) In how many ways we can put 5 letters into 5 corresponding envelopes so that atleast one
letter go to wrong envelope ?
Ans. ( 1) 1890 (2) 119
Geometrical & Grid Problems :

Example# 33 : Let each side of smallest square of chess board is one unit in length. Find the sum of area of all
possible squares whose side parallel to side of chess board.
2
Solution: (82 " 12 ) + (72 x 22 ) + (62 x 32 ) + ........... + (1 x 82 ) = 1968

Aesonance® Co,porate Office: CG Tower. A-46 & 52. IPIA. Near City Mall, Jhalawar Road Kola (Raj.}-324005
PRERMO-80
Website: www.resonance.ac.ln E-mail : conlacl resonance.ac.ln
f;C~IN;;::';'U;.80~3;;;'0:;;2R:;'-;J';;;20~0~7P/!:fL";;c'='o2;;::,4~02:;;9==--
Eduating for better tomorrow rr:t.o1:fi1~F,::e='e:-":
: 1iiia;;;oot:2~si"e"t;555eti'5
Combinatorics/ - - - - - - - - - - - - - , -_,----
-
Example# 34 : A person is to walk from A to B. However, he is restricted to walk only to the right of A or
upwards of A. but not necessarily in the order shown In the figure. Then find the number of
paths from A to B.

·~ B

.
A
Solution: No matter w1-,:ch path the person choses, he must walk 9 steps in total, 4 in the right direction
and 5 in the upwards direction. So we have to arrange 9 steps (or which 4 are of one kind and

5 of the other), which can be done in ways.


4!5!
. d
Hence, th e require b f
num er o ways
91 : ----
=-- 9.8.7.6.51 =126
415! 4.3.2.1.51

Self Practice Problems :

(1) Let each side of smallest square of chess board is one unit in length. Find the total number of
rectangles (including squares) whose side parallel to side of chess board.

(2) Find the number of ways of selecting pair of black squares in chessboard such that they have
exactly one common corner
Ans. 11) 1296 (2) 49

Pigeon hole principle: If there are 'n' pigeons and r holes (n r) then there is atleast one hole having more
than one pigeon

Example# 35 Prove that given 13 points with integer coordinates, one can always find 4 of them such that
their center of gravity has integer coordinates. -
Sol. When we divide 13 integers by number 4, we get 4 possibilities of remainders (0, 1, 2,3). By PHP,
atleast 4 numbers have same remainder. If we find centre of gravity of these numbers , it also an
integer.

Self Practice Problems:

(1) 16 composite integers are chosen from 1 to 2800. Prove that the selection indudes at least two
integers which are not co-prime

~esonance® Corporate Office: CG Tower, A-46


I I EducaUng for better tomorrow
& 52, IPIA, Near City Mall, Jhalawar Road, Kota (Raj.)-324005
Webslle : www.resonance.ac.tn E-mail : contact resonance.ac.ln
r:.Tot::;:t;;:F,:"'.re:-:e~:-:c,a;;-;oc;;o""2"'saccs"'s~ss:-'f':c::;1N:-;-: ~u:::a':co3:':0~2=RJ7.:2:':'00="1:'P""Lc'
:! =-o;,2"'40c"2:::;9= =- -
Combinatorics /,--- ---- ---- ---- ---- ---- ---- ---- 1•

• Exercise-1
PART - I : PRE RMO
of the same seven
1. There are seven even greeting cards each of a different colour and seven envelopes
so that exacUy four of
colours .Find the number of ways In which the cards can be put in the envelopes
the cards go into the envelopes of the right colour
h, i into three non-empty
2. In how many ways is it possible to separate the nir.e letters a, b, c, d, e, f, g,
batches?
with everybody except
3. There an n persons sitting around a circular table. Each person shakes hands
him. Tne total number of hand shakes are 90. Then find n.
the person sitting on both sides of
and 11 pass through the
4. In a plane there are 37 straight lines of which 13 pass through the point A
no lines passes through both points A and 8,
point 8 . Besides, no three lines pass through one point,
and no two are parallel. Then find the number of intersectio n points the lines have.

5. Find the number of selections of six letters from the letters of the word
AK A'
' KAR NAT
in a row such
Find the number of ways by which 4 green, 3 red and 2 while balls can be arranged
6. All balls of the same colour are identical
that no two balls of the same colour are together.
the no of ways is
There are 10 stations enroute. A train has to be stopped at 3 of them. Lei N be
can be stopped if aUeasl two of the stopping stations are consecutiv e. Find N.
7. which the train
is formed . Find the total
Number·n is selected from the set {1, 2, ........ 200}. and the number 2n + 3n + Sn
number of ways of selecting n so that the formed number is divisible by 4
8.
Find of the total number of words which are formed by using the all the letters
of the word
"SUCCES S" such the no two alike letters are together
9.
Let N be the number of integer solutions of the equation n,+ n2 + n3 + n, = 17 when
n 1, n 2 -1, n 3 3, n, 0. If N =
10. 17
CK ,then find K.
TION such that any two
11. The number of different permutatio ns of all the letters of the word PERMUTA
neither both vowels nor both identical is K . 5! . 5! . Find K
consecuti ve letters in the arrangeme nt are
10. If the number of ways
12. There are ten boxes numbered from 1 to 10 and 10 balls numbered from 1 to
numbered balls go to odd
of putting all balls in given boxes such that no box remains empty, odd 2
number which is written on the ball is K , then
numbered box and no ball goes to the box having same
find K.
elements. if the sum of
Let set A = {1, 2, 3 ......... 50}. Set 8 is a subset of A and B has exacUy 20
13. elements of all possible subsets of 8 is ' 9C,s . x 25 x K

her first husband. They


14. Mr. john has x children by his first wife and Ms. Bashu has x + 1 children by
The whole family has 10 children. Assuming that two children of
marry and have children of their own.
Find the maximum number of fight that can take place among children
the same parents do not fight,
the two sets containing 6
15. What is the total number of ways of selecting atleast one item from each of
identical items each?

5
Corporate Office: CG Tower, A-46 & 52, IPtA, Near City Mall, Jhalawar Road. Kola (Raj.}-32400
r"'I: Website : www.resonance.ac.ln E-mail : contact resonance.ac.in
Educating for better tomorrow : ;';ua;;';o:::3'='02;:;;R:c-:J'=20='=0:::1~PL:":C:c::oc::2':'40:'-::2~9==----
1-;To1'-';1:"::F:-"re"'-e-':'-::-1'::-:aoo~2;;:s~a~555~57';::-c:-:-rN=-'
Comb inator ics/
_ __ _ _ :_ -- -- -- -- •
PAR T- II: RMO
1. All the 7-digit numbers containing each of the digits
1,2,3,4,5,6,7 exactly once, and not divisible by 5,
are arranged in increasing order. Find the 32001h numbe
r in the list
2. Determine tht: number of subsets of {1,2,3 ...... 70} whose sum is larger than 1243.
3. Find the numbe r Of 4 >< 4 array where entries are from
the set {O, 1,2,3} and which are such that the
sum of the number's in eact'i of the four rows and in
each of the four ·columns is divisible by 4.
4. Find the number of-9 digit natural numbers in which
each digit appears at least thrice.

5. Find all 6-digit natural numbers a a a a,a ae formed


1 2 3 5 by using the digits 1,2,3,4 ,5,6
once each such that number a a a •• ...... ak is divisibl
1 2 3 e by k for 1s k s 6.
6, In a group of 10 people, the su~ of the age of the membe
rs is 542 years. Prove that four memb ers can
be chosen so that their ages is not less than 216 years.

7. Find numbe r of three digit numbers equal to the sum


of the factorials of their digits.
8. Find the sum of all four digit numbers formed by the
digits {1,2,3 ......9} in which exactly two digits are
prime and repetition of digits is not ai19Wed

9. If 11 distinct integers are chosen from among 1,2,3


..... 20 then show that selection include s at least
one pair of integers which are relatively prime.

10. Let n be -a positive integer. Find the maximum numbe


r of non congruent triangles whose side length s
are integers less than or equal to n.

11. Find all the three digit numbers for which one obtains
when dividing the numbe r by 11, the sum of the
· squares of the digits of the initials .

12. Find all 8-digit numbers such that the sum of their digit~
is 14 and each of the digits 0, 1,2,3,4 occurs at
least once in them.

13. Find·all J..digit nombers formed by using only the digits 5 and 7 and divisible by both 5 and 7.
14. Two boxes contain 65 balls of several different sizes.
Each ball is white, black, red or yellow. If you take
any 5 balls of the same colour at least two of them
will always be of the same size (radius) Prove that
there are at least 3 balls which lie in the same box,
have same colour and have same size (radius)

15. of
A set 10 positive i~tegers in given such that decima
l expansion of each of them has two digits.
Prove that there are two disjoint subsets of the set
with equal sum of their eleme nts.

50 1
16. Find the numbe r of solutions of the equation L x = 1501 , where x,'s are odd natural numbers:
1
1-1

17. Find the number of different seven digit numbers that


can be written using only the three digits 1, 2, 3
with the condition that the digit 2 occurs atleast 2 times
in each numbe r.
18. Find the numbe r of permutations of the letters of
the word CONT RADIC TORY such that neithe r the
pattern "CON'' nor "RAD" nor "ORY" appears

Resonance® Corporate Office: CG Tower, & 52. tPIA, Near City Mall, Jhalawar Road. Kola (Ral.)-32
4005
Educadng for tomorrow !-¥W;;l;ebs'F.:lte::':'7-wwwffif
're~•~on7an;!;c~e.ac~.~lnJ.,E~-c::-m~•l~I:~c~on~t•~c~t!l!r~es~o!!!n•~n~ce!:J.&e!!£:!!.ln!.,__
7
Toll Free : 1800 258 5555 CIN: U80302RJ2007PLC0240 __
29
Combinatorics /,-- ---" ---- ---- ---- ---- ----
---- ---1 •
both equation s
19. Find the number of quintuples (x, y, z, u, v) of positive integers satisfying
=
x + y + z + u 100 and x + y + z + v 70 =
In how many ways can 10 persons take seats in a row of 24 seats
so that no two persons take
20.
consecutive seats

in 10 different boxes if each


21. In how many we can put 6 identical white balls and 5 identical black balls
box must contain atleast one ball.

• Exercise-2 )
PART - I : PREVIOUS ASKED QUESTION FOR PRE RMO
PRMO (2012)
usly, he posts one letter through
1. A postman has to deliver five letters to five different houses. Mischievo
if it is the correct address: In how _many different ways could he do this
each door without looking to see
so that Exactly two of the five houses receive the correct letters ?

PRMO (2013)
number of ways of choosing
2. There are n - 1 red balls, n green balls and n + 1 blue balls in a bag. The
colours is 299. What is the value of n?
two balls from the bag that have different
. Suppose that for any. two
3. To each element of the set S = {1, 2, ..... , 1000} a colour is assigned
if 15 divides a + b then they are both assigned the same colour. What is
elements a, b of S,
the maximum possible number of distinct colours used ?

PRMO (2015) WB
4. 4, 5, 6, 7 exactly once, and not
Consider all the 7-digit numbers containing each of the digits 1, 2, 3,
g order. What is the 2015-th number (from the beginning) in
divisible by 5. Arrange them in decreasin
this list?

200) .
Determine the largest 2-digit prime factor of the integer , i.e., 200c 100•
5. ( 100

Find the sum of all the distinct prime divisors of L


2015 2
r
(2015) i.e., L r 201 5 2 2015
• c,
6. ,-o r r-0

of the integers 1, 2 , ...... n. In


Let n!, the factorial of a positive integer n, be defined as the product
at the end of the integer
7. other words, n! = 1 " 2 " ........... " n. What is the number of zeros
102 2 2 2
! + 11 ! + 12 ! + - - - + 99 1?
b = 1000 and neither a nor b
8. Find the number of ordered pairs of positive integers (a, b) such that a+
should be counted as two distinct solution s.
has a zero digit. Note that (2,998) and (998,2)
4098
Find the largest positive integer n such that 2" divides 3 - 1.
9.

PRMO (2015)
of a given cube?
10. How many line segments have both their endpoints located at the vertices
danced with exactly. three men.
11. At a party, each man danced with exactly four women and each woman
Nine men attended the party. How many women attended the party?

nesonance® Website
Corporate Office: CG Tower, A-46 & 52, IPIA, Near Chy Mall, Jhalawar Road,
: www.resona nce.ac.ln E-mail: contact reaonance.ac.ln
Kola (ReJ.>-324005
r"(
Educating tor better tomorrow : 1;;;8;;;00"2"58;;-;;55~5;--5"C;;;IN:;::'.':u~eo;;:;3~02a;:R:'7J2::';;0~07:';P~L~C0""2:":'40=::2~9=:;.:.:.:..---
r-r:ro1:;;1"F,::ee~
Combinatorics/,-----------------------------,•

PRMO (2016) Chd


12. On a stormy night ten guests came to dinner party and left their shoes outside the room in order to
keep the carpet clean. After the dinner there was a blackout, and the guests leaving one by one, put on
at random, any pair of shoes big enough for their feet,(Each pair of shoes stays together). Any
guest who could not find a pair big enough spent the night there. What is the largest number of
guests who
might have had to spend the night there ?

PRMO (2016) Delhi


13. There are three kinds of fruits in the market. How many ways are there to· purchase 25 fruits from
among them if each kind has at least 25 of its fruit available?

PRMO (2016) WB
5 5 6
14. Find the coefficient of a5b c d in the expansion of the following expression.
(bed + acd + abd + abc)7

15. Find the total number of times the digit '2' appears in the set of integers {1, 2, ........2016}
For example, the digit '2' appears twice in the integer 229. ·

16. Find the number of· pairs of positive inte·gers (m, n), with m n, such that the 'least common multiple'
(LCM) of ,n and n equals 600.

PRMO (2017)
How many positive integers less than 1000 have the property that the sum of the digits of each such
17. number is divisible by 7 and the number itself is divisible by 3 ?

18. There are five cities A, B, C, D;'E on a certain island. Each city is connected to every other city by road.
In how many ways can a person starting from city A come back lo A after visiting some cities without
visiting a city more than once and without taking the same road more than once ? {The order in which
he visits the cities also matters. e.g., the routes B C A and C B A are different.)

19. There are eight rooms on the first floor of a hotel, with four rooms on each side of the corridor,
symmetrically situated (that is each room is exactly opposite lo one other room). Four guests have to
be accommodated in four of the eight rooms (that is, one in each) such -that no two guests are in
adjacent rooms or in opposite rooms. In how many ways can the guests be accommodated?

20. Find the number of ordered triples (a, b, c) of po~itive integers such that abc = 108.

21. Suppose in the plane 10 pair wise nonparallel lines intersect one another. What is the maximum
possible number of polygons (with finite areas) that can be formed?

PRMO (2018)

22. There are several tea cups in the kitchen, some with handle and the others without handles. The
number of ways of selecting two cups without a handle and three with a handle is exactly 1200. What is
the maximum possible number of cups in the kitchen ?

23. Determine the number of a-tuples ( e!,e2 , ... ,e8 ) such that e 1,e2 , ... ,e8 e {1,-1} and
e1 +2 e 2 +3 e 3 + ... + 8 e 8 is a multiple of 3.

nesonance®
I""(
Corporate'Office: CG Tower, A--46 & 52, IPIA, Near City Mall, Jhalawar R*, Kola (Rll].)--324005
Website : www.resonance.ac.ln E-mail : contact resonance.ac.ln
Educating for better tomorrow 1-;T01'°'1:=:F"",e"--e':"'7.1e=-=00~2s='::e:"::s°=ss:c::s~c'i':1N~:-::;ue'='=o;:;:3'='02:':R:c;J2='=0;;0='=1p!,"'L'=c:;,02;:,4:;:02~9==---
Comhinatorics /

- - -- - -- - - - --
________,

PART - II : PREVIOUSLY ASKED QUESTION OF RMO


·- - - - - - -
, --•
1. Find the number of an 6-digit natural numbers such that the sum of their digits is 10 and each of the
digits 0, 1, 2. 3 occurs at least once in them. [RMO-2008]

2. Find the number of all integer-sided isosceles obtuse-angled triangles with perimeter 2008.
[RMO-2008]

3. Find the sum of all 3-digit natural numbers which contain at least one odd digit and at least one even
digit. [RMO-2009]

4. Let (a,, a 2, a3 •••• a,011 ) be a permutation (that is a rearrangement) of the numbers 1, 2, 3


........... , 2011. Show that there e:,cist two numb~rs j, k. such that 1 j <k - ii = la, -kl.
2011 and la[RMO-2011]
1

5. Consider a 20-sided convex polygon K, with vertices A,, A;•. . .. , A;0 in that order. Find the number of
ways in which three sides of K can be chosen so that every pair among them has at least two sides of
K between them. (For example (A,A;, A.As, A11A, 2 ) is an admissible triple while (A,A;, A.As, A, 9A;.) is
not). · [RMO-2011]

6. A finite non-empty set S of integers is called 3-good if the the sum of the elements of S is
divisible by 3. Find the number of 3-good non-empty subsets of {O, 1, 2, . . . , 9}. [RMO-2013)

7. Suppose ·for some positive integers r and s, the digits of 2' is obtained by pe_nnuting the digits of 2• in
decimal expansion. Prove that r = s. · [RMO-2014)
·• .
8. Is it possible to write the numbers 17, 18, 19, ..... 32 in a 4 "f grid of unit squares, with one number in
each square, such that the product of the numbers in each 2 " 2 sub-grids AMRG, GRND, MBHR and
RHCN is divisible by 16? [RMO-2014]

g:
B H C

g_ Find the number of all ·6-digit natural numbers having exactly three odd digits and three even digits.
[RMO-2016]

10. Consider a chessboard of size 8 units " 8 units (i.e. each small square on the board has a side length
of 1 unit). Let S be the set of all the 81 vertices of all the squares on the board. Whal is number of line
segments whose vertices are in S1 and whose length is a positive integer ? (The segments need not be
parallel to the sides of the board.) [RMO-2017]

nesonance® Corporate Office: CG Tower, A-46 & 52, IPIA, Near City Mall, Jhalawar Road,K~ta (Raj.)-3240-J~
l""t Website : www.resonance.ac.ln E-mail : contact resonance.ac.ln
Educating ·tor better tomorrow r-To1'7.1:'::F'"",e"-e":-':=:':1a""o'if-o=:;2sa:':-':s'='ss:'::s~c:':c1Nc1,
: U:!:':8:!:0~30:!:2'::R:;J2,!.::0;:;07~P~LC~0~2~40~2~9=~---
Combinatorics/,---------------------------•

• -Answers
• Exercise-1
PART - I : PRE RMO

1. 70 2. 3025 3. n = 15 4. 535 5. 19

6. 79 7. 64 8. 99 9. 96 10. 3

11. 57 12. 44 13. 51 14. 33 15. 36

PART - II : RMO

1. 6172453 2. 2s9 3. 4. 1,98,207

5. 123654, 321654 7• .

l
n(n + 2)(2n + 5)
n is even
24 ' 11. 550, 803
10.
(n + 1)(n + 3)(2n + 1)
24 ,. n is odd

12.

13. 7 7 7 5 7 7 5, 7 7 5 7 5 7 5, 5 5 7 7 7 7 5, 7 5 7 5 5 7 5, 5 7 7 7 7 5 5, 7755755

5 7 5 5 5 7 5, 5 5 5 7 7 5 5, 7555555

131 111 111 11! 9! 91 9! 71


16. 17. --------- --+-+-+---
(2!)4 2!.2! . (21)3 2!.2! 2! 4! 2! 2!

15 10! 10! 10!


19. 20. C10 . 101 21. --+--+--
61.3! 51.-4! 51.4!

nesonance®
t'(
Corporate Office: CG Tower, A-46 l 52, IPIA, Near City Mall, Jhalawar Road, Kola (Raj.)-324005
Website : www.resonance.ac.ln E-mail : contact resonanee.ac.ln
EducaUng for better tomorrow 1--,T=-o1-=-1,e-Fr~ee~:..,.1e""'o""o-=-25="e"'=5=55"=5':"c="1""°N:~uc.,.so""'3-=-02""R""J2""'0""01""P""Lc""o""'24'"0""29"-==-'----
"
Combinatorics /
--
• Exercise-2

PART - I : PREVIOUS ASKED QUESTION FOR PRE RMO


2

1. 20 2. 10 3. 8 4. 471 2536

5. 61 6. 61 7. 24 8. 738

9. 14 10. 28 11 . 12 12. 5

13. 351

14. 630 15. 619 16. 53 17. 28

18. 60 19. 48 20. 60 21 . 36

22. 29 23. 88

PART - II : PREVIOUSLY ASKED QUESTION OF. RMO

1. 490 2. 86 3. 370775 5. 520

6. 351 8. Not possible 9. 281250 10. 780


iI
I

nesonance®
r"(
Corporate Otrlce: CG Tower,
Website : www.resonance.ac.ln
A-46 & 52, IPIA, Near City Mall, Jhalawar Road, Kota (Raj.}-324005
E-mail : contact resonance.ac.ln
Educating for better tomorrow r-t.ro1;f,1't.F,;::ee:'::~1i"iso~o-;:25~5~5'i.55~57.c~1~N:~u;iso;;:;3:';;; o7~p~LC;f;0~24~0~2 9 ~ ~ - --
02;;;,R;'-;:J2:ao~

You might also like